100% found this document useful (1 vote)
2K views173 pages

Cirtoaje Vasile Can Vo Quoc Ba Ahn Tran Quoc Inequalities Wi PDF

Uploaded by

Maria Popescu
Copyright
© © All Rights Reserved
We take content rights seriously. If you suspect this is your content, claim it here.
Available Formats
Download as PDF or read online on Scribd
100% found this document useful (1 vote)
2K views173 pages

Cirtoaje Vasile Can Vo Quoc Ba Ahn Tran Quoc Inequalities Wi PDF

Uploaded by

Maria Popescu
Copyright
© © All Rights Reserved
We take content rights seriously. If you suspect this is your content, claim it here.
Available Formats
Download as PDF or read online on Scribd
You are on page 1/ 173
VASILE CIRTOAJE VOQUOCBACAN TRAN QUOC ANH Inequakrties with Beautiful Solutions SB om About the Authors Vasile Cirtoaje is a Professor at the Department of Au tomatic Control and Computers from Petroleum-Gas Uni- versity of Ploiesti, Romania, where he teaches university courses such as Control System Theory and Digital Control Systems. He has obtained original scientific research results in the fields of monotonic systems, control algorithms based on process compensation, control system design by limiting the magnitude ratio, and, of course, mathematical inequali- ties. Vasile Cirtoaje became interested in mathematical inequalities at an early age, in high school. Since 1970, he published many solutions, problems and ar- ticles in the Romanian journals Gazeta Matematica-B, Gazeta Matematica-A and Mathematical Review of Timisoara. During the last years, Vasile Cirtoaje has published many interesting problems and articles in Mathlinks Forum, Math- ematical Reflections, Crux with Mayhem, Journal of Inequalities in Pure and Applied Mathematics, A.M.M. He has collaborated with Titu Andreescu, Gabriel Dospinescu and Mircea Lascu in writing the book Old and New Inequalities, and he wrote on his own Algebraic Inequalities - Old and New Methods. Notice that Vasile Cirtoaje is the author of some well-known strong methods for proving and creating inequalities, such as Side-Convex Function Method, Equal Variable Method, Arithmetic Compensation Method. Vo Quoc Ba Can is a student at the Can Tho University of Medicine and Pharmacy. As a high-school student, he par- ticipated in many national contests obtaining several prizes. ‘Though at the moment he is not studying mathematics, his activity in Inequalities has proved to be quite wide lately. Some of his problems were published in specialized jour- nals, but the biggest part of them became popular on the worldwide known MathLinks forum. On the same theme, he (co)authored several manuscripts, which were published in Vietnamese. Since 2008, he was invited to lecture and seminar on inequalities at many high schools for gifted students in Vietnam such as Le Hong Phong high school, Nguyen Binh Khiem high school, Ly Tu Trong high school, and also others. Tran Quoc Anh jis a student at Hanoi University of Tech- nology. As a young student, but he has many experiences ig in writing books and teaching inequalities. He takes part in Vietnam and international mathematical forums as Collab- orator or Active member. Particularly, on the world mathe- matical website MathLinks, his creative inequalities have be- come brand which is known by many international friends for their mysterious beauty attached to impressive solutions. Soars j Vasile Cirtoaje Vo Quoc Ba Can Tran Quoc Anh Inequalities with Beautiful Solutions GIL Publishing House *\U SE IMPRUMUTA ACASA © GIL Publishing House 2009 Title: INEQUALITIES WITH BEAUTIFUL SOLUTIONS Author: Vasile Cirtoaje, Vo Quoc Ba Can, Tran Quoc Anh ISBN 978-606-500-014-8 Copyright © 2009 by Gil. All rights reserved. National Library of Romania CIP Description CIRTOAJE, VASILE Inequalities with Beautiful Solutions / Cirtoaje Vasile, Vo Quoc Ba Can, Tran Quoc Anh = Zalgu ; Gil, 2009 ISBN 978-605-500-014-8 I. Girtoaje, Vasile TL. Quoc, Vo Ba Can Hil. Quoc, Tran Anh 512 HU "727077U* GIL Publishing House P.O. Box 44, Post Office 3, 450200, Zaliu, Romania, tel. (+4) 0260/616314 fax.: (+4) 0260/616414 e-mail: [email protected] www.gil.ro Foreword “Let solutions say the method!” is the way this book is written. Readers don’t find here the entire theory, strong theorems as well as detailed explanation of the methods. But you can find here a lot of beautiful problems with beautiful solutions. Most of these solutions are simple and elementary, the authors try to avoid as much as possible of using advanced methods of proving inequalities, The main weapons here are skilful technics of handling with algebraic expres- sions and virtuous applications of classical inequalities. It makes the book more romantic rather than academic. And even a student of 8th, 9th grade can read most of the content of this book. Reading the book, you sometimes are surprising with the way the authors solve the problems. “How simple! Why didn’t I think about this?”, you ask. If you read this book for fun only, it is OK. But if you want to learn something from this book? Don't be surprised only. You should ask more “Where does the solution come from? Why and how the authors think about the way?”. The answer is not simple, and you may not to find it immediately in single solution. Try to gather the answer from several solutions. If you succeed, you are going on the right way, the way that the authors of the book want you to go. Tran Nam Dung Ho Chi Minh city University of Science Preface “The only way to learn Mathematics is to do Mathematics.” Paul Halmos The inequalities appeared in Mathematics a long time ago, have developed and evolved stably in course of time, and even more in our days. As stated by Richard Bellman in 1978, “... there are at least three reasons for the study of inequal- ities: practical, theoretical, and aesthetic; ... beauty is in the eyes of the in- equality beholder; ... it is generally agreed that certain pieces of music, att or mathematics are beautiful; there is an elegance to inequalities that makes them very attractive”. We add two new reasons to the three ones already formu- lated by Bellman: fascination to create a new strong and beautiful inequality, and happiness to prove such an inequality by an original and nice way. For all these reasons, the inequalities became very popular in advanced and elemen- tary Mathematics, being very useful in level-transfer tests, in university entrance tests, and especially in national and international contests for excellent students, This explains why a large number of people are so concerned with mathematical inequalities. Nowadays, many clever people find out a lot of new ideas and methods to deal with inequalities, and a lot of “modern style” reference books are published In our viewpoint, the methods for solving inequalities are very important, but above all, learning how to think for creating or solving an inequality is even more important. We wrote the book “Inequalities with beautiful solutions” having in view these things, as well as our desire to make known to the inequality lovers some new inequalities of the authors. With more than 200 problems, which are carefully and logically arranged, the book will help the readers form a general overview on the inequality field, as well as learn the secret of “finding way” to deal with inequalities and other mathematical problems. We look forward to receiving heart-felt comments from the readers to improve the book in the next republication. vii Finally, we would like to thank Dr. Tran Nam Dung, Ho Chi Minh University of Nature, for his helpful suggestions, and our friends, Nguyen Van Dung, Tran Quang Hung, for their creative solutions and contribution in solving many puz- ales. Contents The authors October 2009 Foreword v Preface vii 1 Two-variable inequalities 1 1.1 Statements . . 1 1.2 Solutions . 5 2. Three-variable inequalities 27 21 Statements ............... 27 2.2 Solutions ............00..000.. 44 3 Four-variable inequalities 161 3.1 Statements . . . + 161 3.2 Solutions .... 4 Five and six-variable inequalities 181 4.1. Statements . . 4.2 Solutions ... 181 185 5 n-variable inequalities 213 5.1 Statements ............. . 213 5.2 Solutions .............. 218 6 More difficult inequalities 6.1 Statements . 6.2 Solutions . . 7 Some open inequalities 317 Glossary 325 References 331 Chapter 1 Two-variable inequalities “Truth is ever to be found in the simplicity, and not in the multiplicity and confusion of things Isaac Newton 1.1 Statements 1.1. Let a and b be real numbers such that 9a? + 8ab + 7b? < 6. Prove that 7a+Sb+12ab <9. 1.2. Let a and b be positive real numbers. Prove that 1,3. Let a and b be real numbers such that 3(a +b) > 2|ab + 1). Prove that 9(a3 + b) > |a3b3 +1) 1.4. Let a and b be nonnegative real numbers such that a2 + b? = 1. Prove the following inequality 3v3 ab+maxfa, b} << ~, 2 Vasile Cirtoaje - Vo Quoc Ba Can - Tran Quoc Anh 1.5. Let a and b be nonnegative real numbers satisfying a? + b? = 2. Prove that 3(a* + b*) + 2a*b4 <8. 1.6. Let a and b be nonnegative real numbers satisfying 2a? + b? = 2a + b 5 arb 1-ab> 1.7. Let a and b be positive real numbers. Prove that a+2b b+2a 8 son tas V3. +a? +B). a ‘Two-variable inequalities 3 1.12. Let @ and b be nonnegative real numbers such that Vab =r. Ifr > =, then @ pe then 1 1.2 lta+ 14+b4+b?~ 1+r+¢r?’ (b) fO 0, Prove that a 2ab 3 b SS + ——— £-. va?+b? V9a2+b? Va?+b?-Voa?+H? 2 1.15. Let a and b be nonnegative real numbers. Prove that Vita+V1+b%+ ¥(—a?+(1—b? > (1+ V5) (1b). 1.16. Let a and b be nonnegative real numbers such that a + b = 2. Prove that by? ot soe (54) <2. 1.17. Let a and b be positive real numbers such that a+b = a4 +b’. Prove that ath’ <1 0 and y(y +1) < (x +1) Prove that yy -1)sx*, 1.22. Ifa and b are positive real numbers satisfying a? + b* = 2, then Two-variable inequalities S 1.2 Solutions Problem 1.1. Let a and b be real numbers such that 9a” + 8ab + 7b? < 6. Prove that 7a+5Sb+12ab <9. (Titu Andreescu) Proof. It is easy to check that the desired inequality becomes equality for a = b= > ‘Therefore, in order to have only second order homogeneous expressions 1 1 in a and b, we use the inequalities a 0 and (2b — 1)? > 0, respectively. Thus, we have 1 1 7a+Sb-+120b-97(a? +3) +5 (0847) 412089 = 7a? + 5b? + 12ab —6 = (9a? + 8ab + 7b? — 6) — 2(a? +b? — 2ab) S~-2(a—b) <0. Problem 1.2. Let a and b be positive real numbers. Prove thar 1 4 1 4 32(a? + b?) ab? a?+b?~ (a+b)i © (Tran Quoc Anh) First proof. Applying the AM-GM Inequality, we have 1,1), 4 4,721) _4 4 eR) ete Ve Bae Therefore, it suffices to prove that 4, 32{a? +b?) ab~ (a+b)* © This inequality is equivalent to (a+b)* > 8ab(a? +b), which is true according to the identity (a+ b)* — 8ab(a? + b?) = (a—b)* >0. Equality holds if and only if a = b. o Vasile Cirtoaje - Vo Quoc Ba Can - Tran Quoc Anh Second proof. Using the well-known inequality for x, y > . i 0, we get oe. (x+y? 2 8 8 (a? +B?” (aby ~ (a? +6? 4206) ~ (ab) Multiplying each side of this inequality by a? and b2 , respectively, we obtain a? 7 a? > _ 8a? b b? (+67 aby * aby + ba * Gaby and by adding these two inequalities, we get 8(a? +b?) (a+b)* * This is equivalent to a 24. p2 ape 4, 32a? +52) 2 be +b (at by” which is just the desired inequality. Problem 1.3, Let a and b be real numbers such that 3(a + b) > 2|ab + 1|. Prove that (a> +b) > [a3b3 +4), (Titu Andreescu) Proof. Since a*b? — ab +1 > 0, we have |a9b° +1) = |(ab +1)(a2b*~ ab +1)| =|ab + 1\(a?b? — ab +1) = lab + 1\[(ab +1)? — 3a], and from the given hypothesis, we get lab +1|[(ab +1)? - 3ab] < 3 +b) [Ze +byP- 2ab] 9 = gat b)(3a? + 2ab +362). It follows that a [a3b? +1] < g(t +b)(3a + 2ab + 36%), ‘Two-variable inequalities Thus, it is enough to check the following inequality 8(a* + b8) > (a+b)(3a? + 2ab + 357). This is equivalent to 5(a + b)(a — b)* > 0, which is obviously true because a+ 6 > 0. The proof is completed. Equality 345 3-v5 holds if and olny if a = b = ,ora=b= Qo Problem 1.4. Let a and b be nonnegative real numbers such that a? +b? =1 Prove the following inequality 3v3 ab +max{a, b} < (Pham Van Thuan) Proof. Without loss of generality, we may assume that a = maxfa, b}. Using the AM-GM Inequality, we have pee +30? a ac ttt abS———,, and ax es 2Vv3 4v3 It follows that 2(a?+3b?)+(4a? +3) 6(a?+b7)+3 _ 3V3 ab+max{a, b} =ab+a $ ~~ = OY : 4v3 4v3 4 This proves the desired inequality. Equality holds if and only if (a, b) is a per- 1 mutation of | —, =}. Db 2°.2 Problem 1.5. Let a and b be nonnegative real numbers satisfying a3 + b3 = Prove that 3(a* + b*) + 20*b* <8 Wasile Cirtoaje) Vasile Cirtoaje - Vo Quoc Ba Can - Tran Quoc Anh Proof. By the AM-GM Inequality, we have a3 + b? +1 > 3ab, a? +2 > 3a and b?+2> 3b. Since a® + b = 2, from these inequalities we get ab + 2a°b? -ab <$(4—b)a3 + (4 — a°)b5 + 2036? =4(a° +b°)=8, which yields 3(a* +b) +.2a°b* < 8, Equality holds if and only if Problem 1.6. Let a and b be nonnegative real numbers satisfying 2a + b? = 2a + b. Prove that (Wasile Cirtoaje) First proof. We see that equality holds for a = b = 1. Using this fact, we get (6a-2N3b+1) 3 - (6a-2+3b+1" _, _ 3(2a+b-3)(6a+3b+7) 8 7 8 a—b~3(1—ab) =(3a—1)(3b+1)-8= Therefore, it suffices to show that 2a + b —3 < 0. Indeed, from 2a+b = 207+ b? > (2a + b)? 3 Ps eanciaon follows. Thus, the proof is completed. Equality holds if and only ifa=b=1. | Second proof. For a= b = 0, the inequality is trivial. Otherwise, we write the inequality in the homogeneous form (2a? + b?)? > (a= b)2a? + b?) @atby “°* 3@2a+b) (2a? +07)? (a b)(4a3 — b) Qatoe “ @atbye Two-variable inequalities 7 9 we need to show that (a — b)[3(4a? — b3) — (2a + b)(2a" + b7)] 2 0. ‘This is equivalent to the obvious inequality (a— b)*(4a? +3ab + 2b?) > 0. a Problem 1.7. Let a and b be positive real numbers. Prove that a+2b | f b+2a - [8 aap \ eae ~ Vat (Tran Quoc Luat) Proof. Using the known inequality /¥+ V¥ S V2(x + y) (which can be easily proved by squaring), we see that it suffices to prove that the stronger inequality holds a+2b b+2a 4 a? 4+2b? © b? 4202 ~ atb ‘This inequality is equivalent to (a+ b)a+2b) | (a+b)(b+2a) _ 4 a? + 2b? a? + 2b? Applying the AM-GM Inequality, we have (a+ b\a+2b) 3ab 3ab 3a lt oo 21 + Yt a+ 2b? (a? +b?) +b? 2ab + b? 2a+b Therefore, we need to show that a, b 2a+b 2b+a a which is equivalent to (a—b)? > 0. Also, we can prove the last inequality without expanding, as follows. By the Cauchy-Schwarz Inequality, a 2a 2a 1 1 1, 4a = s + + =st+ > Qa+b 2a+(a+b)+(at+b)~ 9 (2a atb a) 9° a+b) and hence a b 1 4a 1 4b oe fot ott ein 2a+b b+a~ 9 Aatb) 9 MAat+b) Equality holds if and only ifa = b a 727077 Nae Vasile Cirtoaje - Vo Quoc Ba Can - Tran Quoc Anh Problem 1.8. Ifa and b are positive real numbers, then 1 1 +20 * Tab b+2 First proof. Applying the Cauchy-Schwarz Inequality, we have 1 were ge, _ ee)? b+1 2a+1 * 20h +a? ~ 2a 4a? + OqaT? and thus, it suffices to prove that gy gd (a4 17> 22eb+a? +2041) ab+2 . This inequality is equivalent to (a-12> 2(20?b +a? + 2a +1) ab +2 aa. or 2(a 1)? @-is ae 2 Since B13 * 1 and (a ~ 1)? > 0, the last one is true and our proof is com pleted. Equality holds if and only ifa = b = 1 2 oa Second proof. By replacing a and b with ~ ana 4 equality es fi and i ig th — and pr the inequality becomes a b 2ab ag a+2° b+2> 2ab+1 Lets = = ts=a+b and p= ab. By the Cauchy-Schwarz Inequality, we have a b (a+b)? st 7 a+2" b+2 a(a+2)+b(b +2) 3? 425—2p Therefore, it suffices to show that s2 2p 3a5, 5,23 Ss? 4252p ~ p+ Since S*(2p +1) — 2p(s? + 25 ~ 2p) = (s— 2p? > 0, the proof is completed. 1 ‘Two-variable inequalities Third proof. Setting s =a + b and p = ab, we can write the inequality as lts 1 1+2s+4p 2+p’ 1+ ps 2 3p This inequality follows immediately from the AM-GM Inequality, since 1+ps—3p=1+a°b+ab’—3ab > 0. Problem 1.9. [fa and b are nonnegative real numbers, then 1 ot 1 (+a)? * (14+b)? ~ 1+ab° First proof. If one of a and b is zero, then the inequality is obvious. For positive a and b, by applying the Cauchy-Schwarz Inequality, we have » fa 2 fb (+n? s ($41) @o+0, and camps (F41)r40, Therefore 1 1 1 1 1 i > i = : Gsaretarbe= 7a b Q+ayP (+b) (E41) @o+1) (2+3) aon abt a Equality holds if and only ifa = b =1 a Second proof. 1f one of a and b is zero, then the inequality is obvious. For - 1 positive @ and b, by replacing « wish =, and b with =, the inequality becomes a e ab (a+1)? (b+1)? ~ ab+1 +b Let us denote s = and p = Vab, s > p > 0. By the Cauchy-Schwarz Inequality, we have oy (a+b? (a+1? © (b+1)? © (a+1P+(b+ 12 Vasile Cittoaje - Vo Quoc Ba Can - Tran Quoc Anh Therefore, it suffices to show that 292 ie 5 F. 2s? +2s+1—p2 = pez]? which is equivalent to (2s —p?)? + p°(p? 2) > 0, For the nontrivial case p < V2, we have 2s —p? > 2p— p? = p(2-p) >0, and hence (2s— p?)? + pp? ~ 2) > (2p p?)? + p2(p? — 2) = 2p%(p - 1)? > 0 Third proof. Using the notations above, we can write the inequality as 4s? + 4s +2 2p? eri (25+1+p?}? > 1 4p?’ or 4p?s? — 3p4—2p? 41 0. (1+p?)(2s+1+p2) =" This inequality is true, since 4p*s? — 3p* — 2p? +1 = 4p?(s? — p®) + (p?— 1) > 0, Problem 1.10. Let a and b be positive real numbers satisfying the condition a? + BS 562, 13 Two-variable inequalities Thus i. spt-3 2 +3 Combining this with the given hypothesis, we have in succession 3a’ a +b>> 3a7-1 5b°-3 24 52> 2a ot a +b°2—— 4G a’ +3b? <4, a? +b? $4— 2b? <4—2(2b - 1)—2(b - 1) $6 — 4b, 1 1 ———— a? +b? ~ 6—4b Therefore, i 1 >b-1? eT > = +b 2 6-4b 2 3-2) Equality holds if and only if a= Problem 1.11. if a and b are positive real numbers, then sf eases Vad re +B). a5 First proof. Write the inequality as ar 5+) > ¥3(1 +a? +b?). a “\b By squaring, we get 1 I sai? (ot 420-3455 Since 1 1 bi +ab—-34 > 2b- 3+ G=b+bt 77-320, we may apply the AM-GM Inequality to get I 1 a (o42p-34 442b-34—5. ate (0 +2b-34 5) 224 bt +2 B 4 Vasile Cirtoaje - Vo Quoc Ba Can - ‘Tran Quoc Anh ‘Thus, it suffices to prove that 24 DA +21 b 1 4(b*+2b-34 5) > (5243 Be This inequality is equivalent to 2 b?4+3-= +e; 3): which holds if b> —2b3+4b?-75+4>0, or b(b? 1)? +4(b- 1)? > 0. The last inequality is clearly true, and the proof is completed. Equality holds if and only if a = =1, a Second proof. For any positive real number m, applying the AM-GM Inequality, we have 7 1l+a? +b? 2V3(1 +a? +b?) <3amp ote th (an am Therefore, the desired inequality is proved if we have 2 1 4 2 ata(5 +) =(m+2)ar tt? i a b nm ma 1+b? 1 a+—— moa or : 1 1+b? 2 5 1 a) 27] ta( =420' ~am-=) =o. a m b mi 263 +1 Choosing m = ‘oosing m = —>—, we get 146? 3b(7 +1) _ B+ T4+1~35 m "2641 ops and 2 2b +1 3b > +2b? -3m—— =~ 4252_ 7 b b 2b +1 1 3b D8 +3 41-35? b 2841 Babs +1) so that the above inequality is true and our proof is completed 20, ao ‘Two-variable inequalities Problem 1.12. Let a and b be nonnegative real numbers such that Vab = r. 3 (a) ifr 2G, then 1 to 2 l+at+a? 1+b+b7~ 14r+r?’ @fo0ands 2, wehave 71 2? +r -1)s+2r34r?—r—-12 Ar? 4r—-Drt2sre—r-1 =4r?43r? -3r-1 1 =(4r—3)(r24 2843) 42 50, 273/73 Equality holds if and only ifa = (b) Using the result above, we have to prove that Ar? +r—-Ust2r+r?-r-1<0. ‘This is true since Per-1=5(art1- V5) (2r+14-¥5) <0, and a+r? —p-1aar(r?-Dtrtr—-1 2ab, or (a> + b°)> > 8ab*. Multiplying each side of this inequality by 3(a? + b*) > 0, we can write it in the following equivalent form 3(a? + b3)* > 2403b3(a3 + b%), Since 3a°b3(a + b3) = (a? + b9)3 — (a? +b) = (a? + 63)? — 2, it is equivalent to 3(a? + b°)* > 8[(a? + b3)3 — 2], or 3(a? + b*)* + 16 > B(a3 +b). By the AM-GM Inequality, we have 3(a3 + B3)* + 16 = (a3 +3)* + (a3 + 53)" + (a? + D3) + 24 > 4V (2 + 224 = 803 + 55), Equality holds if and only if a= Second proof. After homogenizing, we can write the inequality as =P a° +b? > 2ab\/* = (a? + b>)? > 28a? b%(a? + b°). or equivalently, Since a + b° = (a3 + b5)(a° — ab? + B®) and a? + b? > 0, it is equivalent to (a3 + 3) > 28a9b9(a°— ab? + b°), ‘Two-variable inequalities 17 Applying the AM-GM Inequality, we have a b9(a5 — 0363 + b9) = V/(a3b3)5(a% — 3b? + b°) 3b + (0% ab? + dS Therefore (a3 +b°) ] (a3 + B88 °49(a® —a2h? + B® aba on+6s[ "i a The proof is completed. Problem 1.14. Let a and b be real numbers such that a? + b? > 0. Prove that b 2ab 3 tt 5G Vato? Voar+b? var+b?- Voa2+b? 2 (ack Garfunkel) Proof. Since a <|a| and b < |b}, it suffices to prove that lal 1b) 2lallb) me ae Va2+b? 9a? +b? Va? +b? 9a? +b? Setting now a? = x and b? = 3y?. The inequality is equivalent to x y XY 2 +2)|—— +4,] —____ «3. x+3y y+3x (e+ 3y)y +3x) Applying the AM-GM Inequality, we have ct 2 BY, yf Fg oy x+y x+3y y+ax xty yt3x 8xy qi (e+ 3y)(y +3x) © 2° and xy (x+3y)(y + 3x) Adding these three inequalities, we can see that the left hand side of the above inequality does not exceed xty | x+y x+3y | y+3x 8xy (e+ 3y)(y +3x)’ 18 Vasile Cirtoaje - Vo Quoc Ba Can - Tran Quoc Anh and thus, it is enough to prove that BY | ty | 8xy 3 x+3y " y43x * @43yyys3n <2 But this is true because 3 xty xty bry 3 ty +8xy 2 xt3y yt3x (x+3y) 43x) 2° &+3y)G+3x) (x-y? © 26643y)(y +3x) ~ ° The proof is completed. Equality holds if and only if b= Y3a > 0. a Problem 1.15. Let a and b be nonnegative real numbers. Prove that Vita? + V1+b?+ ¥(1— a)? +(1—b) > (14 V5) (1 ab). Proof. We claim that Vita?+V1+b?+ab>14+ V1+(a+by, q@ and V(l-a?+(1—b)? + V5ab > 1+ —a— by? @ Proof of (1). Let A=1+V1t+(atbP+V¥1+4+a24+ V1+b2, B=V1+(a+b)+ V(1+02)(1 +62), and Since 1+V1+(at+b?-Vite?~V14+be= 2ab +2 | Vis @ro?- (A+ AATHD| a A aad + Ab —ab) 4 40d 2ab A = A” it suffices to show that ‘Two-variable inequalities _ 19 But this is true because A> 4 and B > 2. Proof of (2). The inequality is equivalent to ¥Sab > 1+(1-a-b- V(1-a??+(1— by, °r 2ab V1+(-a-bY+ YO -a?t+(1— bP Since ab > 0, it suffices to prove that Vi+(1-a-b)?+V(1-a)?+(1- bP > But this is true because V5ab> 2 / 2 oe 1+(1 by 21> Te Now, by combining the inequalities (1) and (2), we see that it suffices to prove that the following inequality holds Vit(a+bP+ V14+0—a—bP = v5 However, this is clearly true according to Minkowski’s Inequality Vit(a+by+V1+(-a-bP > VA+1P +(atb+1 1 The proof is completed. Equality holds if and only if a= > and b=0, ora =0 a—bP=V5. 1 a =r a and b a Remark, Actually, the stronger inequality Vita?+ V1i+b?+ VO -aP+(1- by + (1+ v2) ab 2145 holds for any nonnegative real numbers a, b. Problem 1.16. Let a and b be nonnegative real numbers such that a + b Prove that , a® 4 py SS) £3 2 (Vasile Cirtoaje) 20 Vasile Cirtoaje - Vo Quoc Ba Can - Tran Quoc Anh Proof. Without loss of generality, assume that a > b. Since 0 0. We have In(a*b?) = alna + bInb 0. To prove the right inequality, we use the inequality Iny > 1-4, y > 0, which y 1 follows from Inx 1 is true if a+b? >a? +b Using the hypothesis a + b = a‘ + b*, we can write this inequality in the homo- geneous form , ° (a + b)(a? + b3)9 > (a4 + b*)(a? + b7)°, which is equivalent to A-3B>0, where A=(a+b)(a? +b°) — (a? + b*)(a° +b), B =a?b*(a4 + b*)(a? +b?) — a3b*%(a + b)(a? + b*). Since A= ab(a? — b3)(a5 — b§) and B = a?b(a — b)(a® — b5), we get A-3B =ab(a—b)*(a5 — b*) > 0. This completes the proof. Both inequalities turn into equalities if and only if : o a=b=1 Problem 1.18. Ifa and b are positive real numbers such that a + b = 2, then at4p <2, (Vasile Cirtoaje) Proof. For a = b, equality holds. Without loss of generality, assume that 0 < b <1 aH(a—1), and b’-1>b'F(b-D. Thus, it suffices to show that av (a-1)+b 7? (b-1)20. Since ¢— 1=1-b > 0, we only need to prove that 22 Vasile Girtoaje - Vo Quoc Ba Can - Tran Quoc Anh 4 . Lt which is equivalent to (ab)? < 1, This inequality is true since from a + b > 2Vab, we get Vab < 1. Equality occurs for a = b = 1. Lemma. Let p and c be positive real numbers. If either p > 1 and c > 1, or Ope'F (c~1), Proof. We need to show that f(c) > 0, where F()= cP -1—pe@(c~1) 1 We have f/(c) = 5 ni pe‘ g(c), where i g(c)=2c? —(p+1)e+p-1, sO=@0+Y(cF -1). Since g’(c) 2 0, g(c) is strictly increasing. In the first case (namely ¢ > 1), we have g(c) > g(1) = 0, f'(c) > 0, f(c) is strictly increasing, and hence f(c) > £(1) = 0. In the second case (namely 0 < c < 1), we have g(c) < g(1) =0, J) SO, f(c) is strictly decreasing, and hence fc) > f(1) = 0. Qo Problem 1.19. Let a and b be Positive real numbers. Prove that (@+¥8) (SS 1 ) <2, a+3b Vb+3a (Michael Rozenberg) Proof. By the AM-GM Inequality, we have and From these two inequalities, we deduce that vat+vb_1f a 3 Vva+3b 2(as5*3) ‘Two-variable inequalities 23 Similarly, we also have faeVb 21/_) 3) b+3a 2\atb 2 Therefore 1 1 1 (2 +b et Se) 55 + (var v5) (x 7m) 2\a+b Equality holds if and only ifa = b. a Problem 1.20. Let x and y be positive real numbers such that xty + ¥2x? + Ixy +3y? =4. Determine the largest value of x*y. (Michael Rozenberg) Proof. From the given condition, we see that x + y <4. Therefore, it can be written as > 2x? +2xy t3y?=(4-x-y), which is equivalent to (x +4)? +2(y +2) = 40, ar xy? 2 a(2+5) +@+yP=20 By the AM-GM Inequality and Hélder's Inequality, we have “ 2 (2+3)'+(2+3) +ente3 [(2+ “4 »| “33 (2+ i=) From this, we get ; z a/x?y > xy\ zoza(2+ 5 5) 2y< =-1). wysaa( (3 we 24 Vasile Girtoaje - Vo Quoc Ba Can - Tran Quoc Anh 5 Since we have equality for x =2y = 4 (5 - :) , this means that 3 | max{x*y} = 32 (V1) . Oo Problem 1.21. Let x and y be real numbers such that y > 0 and y(y +1) < (x+1)*. Prove that y(y-1) x?. Then, we have y> land ¥Y-D+2x41>(xr41). Combining this with the inequality (x +1)? > y(y +1), we get YO -1)42x+1> yy 4D), or 2x>2y—-1>0. It follows that 400? - y(y - 1)] > (2y - 1)? -4y(y-1) =150, which is a contradiction. So we must have yy-Dsx, as desired. Equality holds for x = y =0. a Problem 1.22. if a and b are positive real numbers satisfying a4-+ b4 =2, then (Vo Quoc Ba Can) Two-variable inequalities Proof. The inequality is equivalent to 5a‘ +3b4 > 8a°b. Since Sa‘ + 3b4 = (a4 + b*) + 4a* + 2b* = 2(2a’ + b* + 1), this inequality can be written in the form ai+a*+b*+1>4a°b, which is obviously true according to the AM-GM Inequality. Equality holds i and only ifa = Remark. Actually, the sharper inequality holds: Ifa and b are positive real numbers satisfying a! + b'? = 2, then 26 sile Cirtoaj Vasile Cirtoaje - Vo Quoc Ba Can - Tran Quoc Anh Chapter 2 Three-variable inequalities “As to inequalities, the simpler and sharper, the more beautiful.” Vasile Cirtoaje 2.1 Statements 2.1. If a, b,c are positive real numbers such that ab + be + ca = 3, then 44 2(1+a?)(1 + b?)(1 +c?) > (1+4a)(1+b)(1 +c). 2.2. Ifa, b, ¢ are positive real numbers, then 1 1 4 _s > +— Sera) 1 elarby =? c2 +a? a+b? 2.3. Let a,b,c be nonnegative real numbers such that a + b +c > 0. Prove the inequality 1 a b oe 1 ooo oo se ss 3~ 3a? +(b+c)* 3b2+(c+a?? 3c? +(at+b)?~ 2 2.4. Let a, b, c be positive real numbers. Prove that 8(a? + b?)(b? + e)(c? +a? atb+e)? > Ha+b)*(b+e)c+a)*(a2+b 27 EEE 3 28 Vasile Cirtoaje - Vo Quoc Ba Can - Tran Quoc Anh 2.5. Let a, b, c be nonnegative real numbers such that a+b+c=3. Prove that a+ab+2abe < 3. 2.6. Leta, b,c be positive real numbers satisfying a+b +¢=3, Prove that be ca ab 3 + —— + —— Ve+3 Vb243 243 = 2.7. Let a, b,c be real numbers, Prove that Lbtena? | e+a-bP | @tb—e 1 2a? + (b+)? 2b + Hay * 224 (aa bye 2 2.8. Let a, b, ¢ be nonnegative real numbers such that (a+ b\(b+c)(c+a)=2, Prove that (a? + deb? + cafe? +b) <1, 2.9. Leta, b, c be nonnegative real numbers such that P+ + =a4b4e, Prove that 2 ab? + 6c? + e292 < ab + be +ca 2.10. Let a, b, ¢ be positive real numbers. Prove that 2.11. Let a, b, c be positive real numbers. Prove that atbtcrt > abt be+cat1)? abe ~ (a+b)(b+c\e+a)" Three-variable inequalities : 29 2.12, Let a, b, ¢ be nonnegative real numbers. Prove that be ca ab atbt+c ———$—— ¢§ ——— ¢§ 2a+b+c b+e+a 2+a+b 4 2.13. Let a, b,c be positive real numbers satisfying a + b +c = 3. Prove that 1 1 1 1 rete apa Fee OD 2.14. Let a, b, c be nonnegative real numbers such that a? + b? +c? = 1. Prove that be ca ab 3 a a a+1° b?+1 e417 4 2.15, Let a, b,c be positive real numbers. Prove that a? +b? + ee a b ¢ ab+be+ca~ a?+ab+be b*+be+ca c*+ca+ab™ 2.16. Let a, b, c be nonnegative real numbers such that ab + be +ca = 3. Prove that 1 1 + + <1. a?+2 be+2 C242 2.17. Let a, b, c be positive real numbers. Prove that a*—be be 2 4a? +4b2 +02 © 4b* + 4c? +a ae a6 4c? +4? +b? ~ 2.18, Let a, b,c be nonnegative real numbers. Prove that a b c —* _,__° _4__* 4at+4bt+c! 4b+4c+a 4c4+4a4b 30 Vasile Cirtoaje - Vo Quoc Ba Can - Tran Quoc Anh 2.19. Let a, b,c be nonnegative real numbers, no two of which are zero. Prove that 1 1 1 9 So t = SS ——_.. a@+abt+b?” b+be+c? | +eata® ~ (a+b +ep 2.20. Let a, b, ¢ be nonnegative real numbers, no two of them are zero. Prove that Be 2 - b+ be + +cata? ~ 2.21. Let a, b, c be positive real numbers. Prove that b c Hab + be +ca) tab+be Bb +be+ca c+catab~ (atb4+ep 2.22. Let a, b,c be the side-lengths of a triangle. Prove that Saeed BB+: re 2atc ° 2b+a 2W+b— 2.23. Let a, b, c be positive real numbers. Prove that Betiby bie aera ate 2b+a’ 2%W+b— 2.24. Let a, b, c be positive real numbers. Prove that a? +b? +c? sa tebe ca ab+be+ca~ b? + be +e Pteata | sabe be 2.25. Ifa, b, c are nonnegative real numbers such that 7(a? +b? +c?) = 11(ab + be+ca), then Sl oa b c < + +—— 52. 28 b+e eta at Three-variable inequalities 2.26. 2.27. 2.28. 31 Let a, b, c be positive real numbers such that ab + be + ca = 3. Prove that 1 1,343 @a1* rl +1 Let a, b, be positive real numbers. Prove that a B 2 QatbQatc)” (2b+0(2b+a) © Qc+aN2e+b) 1. Prove that Let a, b, c be positive real numbers such that a + b ab be, ea Bab+2b4e * 3be+2e+a Xeat2atb 2.29. Let a, b, ¢ be positive real numbers. Prove that 3 3 tee, B+3+abe 8 +a5+abe a? + b3 +abe 2.30. Let a, b, c be positive real numbers. Prove that 2.31 2.32, 1 1 1 3 + + = : avatb b¥b+e cv¥e+a~ Y2abe | . Let a, b, ¢ be positive real numbers. Prove that ab c_at+b, bte ~ tat > + +1 boc a b+e a+b . Let a, b, c be positive real numbers. Prove that a,b ely, @-F bret a7 abtbetca™ Vasile Cirtoaje - Vo Quoc Ba Can - Tran Quoc Anh 2.34, that 2.35. that 2.36. 2.37. . Let a, b,c be positive real numbers such that abc = 1. Prove that 2.38. 81(a +.1)(b? + 1c? +1) < B(a+b+c)*. Let a, b, ¢ be nonnegative real numbers satisfying a + b +c = 2. Prove (a? + ab + b*)(b? + be +c? (c? +ca +a") <3. . Let a, b, c be nonnegative real numbers, no two of which are zero. Prove (b+cP (eta (a+b). a@+be b*+ca Ifa>b>c>d>Osuch that abcd = 1, then 1 1 1 3 l+a > 1+b 1+¢~ 14 Yabe Let a, b, c be positive real numbers. Prove that a+b? +c? ab + be + ca ab+be+ca~ a?+be+ca b?+ca+ab c?+ab+be Let a, b, c be positive real numbers. Prove that a a J reer; x V3(a? +ab + be) . Let a, b, c be nonnegative real numbers such that a +b +¢ = 3. Prove @ 5 c a ff ee fog a Teprte Vinereat Vivara =”? Three-variable inequalities 33, 2.40. If a, b,c are positive real numbers, then [ 11.1 we fiereto(e+gee) ert 1+ 4] (a? +b? +c?) pia 2.41. Let a, b,c be nonnegative real numbers such that ab + be +ea = 3. Prove that —I a(b? +c? , bee +a) cla? +b) @tbe | be+ca ch+ab 2.42. Let a, b, ¢ be nonnegative real numbers satisfying a +b +¢ = 1. Prove that TES ae av'4b? +c? + bv 4c? +a? +cV 4a? +b? < z 2.43. Let a, b, ¢ be nonnegative real numbers satisfying @ + b +¢ = 3. Prove ie that i 1 ee corp Es ab? +8 bc? +8 ca? +8 2.44, Let a, b,c be nonnegative real numbers. Prove that +b? +3 +3abce > ab 2(a? + b?) + be 2(b? +c?) + cay 2(c? +0), 2.45. Let a, b, ¢ be positive real numbers. Prove that 4abe(a? — b*)? Dita Wa-O2 EEF aa rb tera 2.46. Let a, b, ¢ be nonnegative real numbers satisfying a + b +c = 3. Prove that 5 [= +c) b(c+a) clat+b) 2 4(ab+ be +3abe. GEL ree FFL |= (ab + be +.ca) + 3abe. 34 Vasile Cirtoaje - Vo Quoc Ba Can - Tran Quoc Anh 2.47. Let a, b, c be positive real numbers. Prove that a "i 2 2 atbte Vato? Vbr+e2 Verte? v2 2.48, Let a, b,c be nonnegative real numbers such that a® + b°+c® — 3a?b?c? Prove that a+ b> +4¢9—3abe < V2. 2.49, Let a, b, c be positive real numbers such that Prove that 2.50. Let a, b,c be nonnegative real numbers. Prove that (a? +ab + be)(b? + be +ca)(c? +a + ab) > (ab + be + ca) 2.51. Ifa, b, c are positive real numbers, then 2a mre a+b 2b bret 2c = cta 2.52. Let a, b, c be real numbers. Prove that (a? +b? +07)? > 3(a3b + b3c +c3a) 2.53. Let a, b, c be nonnegative real numbers, no two of which are zero. Prove that c 7a-c? = 4. = 16(ab + be +ca) a a+b” 2 Three-variable inequalities Let a, b, c be positive real numbers such that abc = 1. Prove that 2.54. 1 1 ee eel Goa? * aby (@Faja+ by +e) ~ Let a, b,c be nonnegative real numbers, no two of which are zero. Prove 2.55. that 1 Ly 9 (are ten) (at gray ara |24 . Let a, b, ¢ be positive real numbers such that ab + be+ca = 1. Prove that 1+¢%a? (c+aP 1+ab? a + bc? ‘ (a+b (b+c)? Let a, b, ¢ be positive real numbers. Prove that 2.57. a+b24+c2 3(a3b+ bee +c%a) =a, ab+be+ea a2b? + bc? +c7a? 2.58. Let a, b, ¢ be nonnegative real numbers such that a + b +c = 3. Prove that : a (2+ab* (2 + be?)3(2 + ca”)? < 3456. 2.59. Let a, b, ¢ be positive real numbers. Prove that a+ b+? =. ab + be+ca 2.60. 36 Vasile Cirtoaje - Vo Quoc Ba Can - Tran Quoc Anh 2.61. Let a, b, c be positive real numbers, Prove that 242 gab? > (a+ bc) b+e—alle+a—bYla2 +b? +c? +abc). 2.62. Let a, b,c be positive real numbers such that ab-+ be +ca = 1. Prove that (+17 (bh? 41? (2 +1)? bee "ee ge G24 52 7% 2.63. Let a, b, c be real numbers such that a+ b + ¢ = 3, Prove that @—be b?-ca —ab @+3 B43) 243 7% 2.64. Let a, b, c be positive real numbers such that a +b +c =3. Prove that ab oy te 9-2 9 o-b 8 2.65. Prove that, for any positive real numbers a, b, c, the following inequality holds a be 2 (wrest set |e se P+(etae | rare | Prbsee = Ve 2.66. Let a, b, c be nonnegative real nu: 2b, mbers such that a > 0, a a? +b? +c? = 1. Prove that wei te> oan B+c3 a 2 v2. 2.67. Let a, b, ¢ be positive real numbers, Prove that (22 : x \i #)'()'+(Sa)'= Three-variable inequalities 37 2.68. Let a, b, c be nonnegative numbers such that ab+be+ca = 3. Prove that a b? 4. Ga s4 st ot or? @+tb+c be+ct+a chtatb 2.69. Let a, b, c be nonnegative numbers. Prove that (a2 +b? +29 > (a+b +cNad + be +ca)(a? + b* +c*) 2.70. Let a, b, c be positive real numbers. Prove that = .2 1 2.71. Ifa, b,c are positive real numbers such that a+ b+¢ = z + b 4 e then (i-abe) (« +b" +c" for any positive integer n. 2.72. Ifa, b, c are positive real numbers such that abc = 1, then bs + e 3 @ 1+b4 1+ct be 6 bd) ett 2 ®) 2a 14205 142° 2.73. If a, b, c are nonnegative real numbers, then a +4abe b? +4abe c3 + 4abe oe + oo ooo ee? @t(b+ep+babe B+ (c+aP+babe cP +(a+b)> + 6abc 2.74. Ifa, b, c are positive real numbers such that abc = 1, then Tat? b+? , We+7 2a?+1° 2b?4+1 2c?+1~ 38 Vasile Cirtoaje - Vo Quoc Ba Can - Tran Quoc Anh. 2.75. Let a, b, c be positive real numbers such that abe = 1. Prove that 1 1 1 4 sta += + > (+a? a+oy G+? @+atbtee 2.76. Leta, b,c be nonnegative numbers such that a? + b? +c? = 3. Prove that 1+4abe > 5min{a, b, c}. 2.77. Let a, b,c be nonnegative real numbers, no two of which are zero. Prove that 1, 44 1 2a?+bc 2b? +ca ' 2c?+ab~ ab+be+ca a bie 2.78. Let a, b, c be positive real numbers such that . +—+-=5. Prove that ca Wobic a SZ sr ttt Sab + be tea) o (=) +(4) +(3) +12 yer 2.83. Let a, b, c be positive real numbers such that abc = 1. Prove that wo ab be 6 z=, Tab oath! * pcs abe | P+ar toca 4° 2.84. Ifa, b,c €[0, 1], then a(1— b?) + b(1—c?) +e(1 - a Ala 2.85. Let a, b, ¢ be positive real numbers. Prove that 3(a? — ab + b2)(b? — be + cc — ca +a”) 2 abc(a +b? +3). 2.86. Let a, b, c be distinct real numbers such that a +b +¢ = 3, Prove that a? + bc? b?+c?a? c? +a7b? = (ce (c-a | (a—by 2.87. Let a, b,c be nonnegative real numbers such thata <1 abe +2; (b) If ab+ be +ca =3, then ab + bc +c7a > 3, 2.88. Let a, b,c and k be positive real numbers. Prove that a dle, kate kota, ke+b bic a~kbte kota kato 40 Vasile Cirtoaje - Vo Quoc Ba Can - Tran Quoc Anh 2.89. Let a, b, c be distinct real numbers such that a+b +c = 0. Prove that the following inequality holds (ab+be+ca?* [ 1 | 33 Soe (b a)* . (a—by* | — 16° 2.90. If a, b, c are real numbers, then 4abe + ¥ 2(a? + b?)(b? + c?)(c? +a) > (a + b\(b+c)(c +a). 2.91. Ifa, b, c are real numbers, then (a? +ab + b*)(b? + be +e7)(c? +ca+.a*) > 3(a2b+b2c-+c7a)(ab?+be* +ca2). 2.92. Ifa, b, c are nonnegative real numbers, then 3a*— be 3b? —ca 3c2—ab 3 So +t eo ss ts ES 2a? +b? +c? ° 2b? +e2 +2 2c? +a? +5 2.93. Ifa, b, c are the side-lengths of a triangle, then (a+ b)*+(b+c)*+(c+a)* > 9(a4+b4 +4). 2.94. Let a, b, c be nonnegative real numbers such that a + b +c = 6. Prove that a? + 2b* + 3c? +.abe > 24, 2.95. Let a, b, c be positive real numbers such that a + b +¢ = 3. Prove that 8 — slab + be + b+ bi > i wae 1c +ca)Vab + be +ca > (a+ be)(b +ca)(c + ab). 2.96. Let a, b, c be positive real numbers. Prove that (Lta+b +e) tab+ be +ca) > 4/2(a+ be)(b +ca)(c + ab). Three-variable inequalities 4 2.97. Let a, b, c be positive real numbers such that a + b +c = 1, Prove that roils a’) bc 14+48abe" 2.98. Ifa, b,c are the side-lengths of a triangle, then (a2b + bc +-c2a)? > abc(a tb +c)(a? +b? +c”). 2.99. Ifa, b, ¢ are positive real numbers satisfying abc = 1, then fe Eg A i S34 tre * Va a+26 ~ 2.100. Let a,b,c be nonnegative real numbers, no all are zero. Prove that 2 a b c 2 4 et 1+v2 a+ Y2xe4+2) b+ Valer+a%) c+ V2(a? +b?) 2.101. Let a, b,c be real numbers such that 26. a+b b+c cta + + c a b Prove that 3 B4g3 —+ a b a +b3 te + 6(ab + be +ca) > 8(a? +b? +c”). 2.102. Let a, b, c be nonnegative real numbers such that a+ b +c = 3. Prove that 2.103. Let a, b, c be positive real numbers such that abc = 1, Prove that b c fl, Vb?+20 Vct+2a Va?+2b cee 42 Vasile Cirtoaje - Vo Quoc Ba Can - Tran Quoc Anh Three-variable inequalities | 43 2.104. Let a, b, c be positive real numbers. Prove that 2.110. Let x, y, 2 be real numbers. Prove that 4 } 2 _J ye? 28 _ SS fie \> dtd B46" Reta~ Yarrow ra 4(Sx-D#) (Dy? -te) 23(D 2.111. Leta, b, c be nonnegative real numbers, no two of which are zero, Prove that _ 2.105. Let x, y, 2 be positive real numbers satisfying x+y +2 = ve Vab+be+ca 1 1 1 Vb? +ca — Va? +be | Prove that (xy +ys+ax) (Vey + V¥E + Vex)’ 227. 2.112. Let a, b,c be nonnegative real numbers. Prove that fo-a4 4. 2 fae2 2.106. Let a, b,c be nonnegative real numbers. Schur’s inequalities of third 5(a+b+c) 22 (v 4a? + be+ V4b? +ca+ V 4c? + ab) . and fourth degree state that (a) a(a~b)(a~c) + b(b—c)(b — a) + c(c - a)(c— b) = 0, (b) (a b)(a~c) + b(b —c)(b-—a) +c%(c —ay(e— b) 0 2.113. Let a, b,c be real numbers such that a? + b? +e? = 2(ab + be+ca) > 0. Prove that (b) is sharper than (a) if Prove that lb-d_ | le~al_ la) va+ b+ ez 2max{ va, Vo, veh. Va2+2be ee Vc? +206 2.114, Let a, b, c be nonnegative real numbers such that a +2b + 3c = 4. Prove 2.107. Let a, b, c be positive real numbers. Prove that for any n > 1, the that , inequality holds (2b + be +c2a + abc)(ab? + bc? + ca” +abe) <8. ave 4. batt ght ae ct) fat tbr tch seh pee ora eue? (beet stefan) Yo 2.115. Let a, b,c be real numbers satisfying a + b +c =2. Prove that (a+b -—ab)(b +e —be)(c +a—ca) <1-abe. 2.108. Let a, b, ¢ be distinct real numbers. Prove that the following inequality . holds . 2.116. Leta, b, c be nonnegative real numbers, no two of which are zero. Prove ab?+1 bc? +1 ca? +1 fat (a-bP * (b=cP (ae [oe —be+e® tbe 3 a a?—ab+b? 2(ab+be+ca) ce +ab +bt+c8 24 2.109. Let x, y, z be real numbers. Prove that 4(S- Dv) (Ty2- Vy) = [Peo 9-222 ys] gi . a+b +c? > So at+b+e 44 Vasile Cirtoaje - Vo Quoc Ba Can - Tran Quoc Anh 2.2 Solutions Problem 2.1. Ifa, b, c are positive real numbers such that ab + be +ca = 3, then 44 ¥21+a7)(1 4 b*)(1 +2) > (1+a)(1+b)(1 +0). (Wolfgang Berndt) Proof. By the AM-GM Inequality, we have 3=ab+be+ca > 3Va2b%2, and hence abc < 1. Since (14a?) + b? a+b)?+(1-aby, and Wl +e)=(1 +e +(c-1)%, we can apply the Cauchy-Schwarz Inequality to get V2(1 +07)(1 + b2)(1 +2) > (a+ b)(1 +c)+(1-ab\(e-1). Then, it follows that 4+ V2 +07)(14 b1+c2) > 44+ (a tb) +c)+(1-ab)(c—1) =2(1-abe)+(a+1)(b + 1c +1) 2(a+1)(b+1)(c+1), which is just the desired result. Equality holds if ab + be + ca =3, abc = 1 and (a + be — 1) = (1 +c)(1—ab). The last condition reduces to a+b +c = 3. From 0 = 2(a +b +c)? — 6(ab + be + ca) = (a — b)? +(b —c)? + (c —a)?, we get ne 5 =c, Therefore, the original inequality becomes equality if and only if a=b=c=1. Qo Problem 2.2. If a, b,c are positive real numbers, then 1 +} 1 1 atb +e) beta) + clatby 22 b? +c? a? +b? Wasile Cirtoaje) Three-variable inequalities 45 Proof. Since (b +c)* < 2(b? +c”), we get abe) 25, 2a _Marbte) be+c2 ~" b+e b+e and hence, it follows that 1 bt+e 4 OF9) =2a+b+c)" b? +7 Similarly, we have 1 cta 1 a+b —era = Hate y = a+b +c) 24 2 +a? a+b? Adding up these three inequalities, we get the desired result. It is easy to see that equality holds if and only if a = b =c. a Problem 2.3. Let a,b,c be nonnegative real numbers such that a+ b+c¢ > 0 Prove the inequality at 1 = So ¢§ SST 3 3a2+(b+cy? 3b? +(c+a)> 3c? +(a+by (Vo Quoc Ba Can) Proof. It is easy to see that 3a? +(b+¢)* < 3a? +2(b? +c) < 3(a2 +b? +c), and hence 2 a a’ 3a? +(b-+c)* 2 3(a? + b? +c?) Therefore, a b? uo? a+b? +c7 BET see 4 etay BH ETE TPA ‘The left inequality is proved. Equality holds when two of a, b, ¢ are zero. For the right inequality, we will prove that a? a a ee 3a2+(b+c)?~ 2(a+b+c) Then, a b? @ at+btc 1 ara (bbe * BB+ (eral +324 (at by = arbre) 2! || le 46 Vasile Cirtoaje - Vo Quoc Ba Can - Tran Quoc Anh which proves our inequality. Now, we see that the above claimed inequality is true if . 30? + (b+c)? > 2a(a +b +0). Since 3a +(b+c)*—2a(a+b+c) = (b+c—a)?, this is clearly true. The proof is completed, Equality holds if and only if a = b, ¢ = 0, or any cyclic permutation thereof. a Problem 2.4. Let a, b,c be positive real numbers. Prove that 80? +b*V(b7 +07)? +a2)at-b-+cP? > 3(a+b)*(b+c)P*(c+a)*(a2 +b? +2). (Vo Quoc Ba Can) Proof. The desired inequality can be written as 2(a? +b?) 2(b? +2) 2c? +a?) 3(a2 +b? +c ~“(atb+e (a+b? (b+ceP (e+aye 2a? +b?) (@— by? Since Tepe 73 Gey pyin ts equivalent o a4 [ (b-c)? (c-a)?] _ 3(a2 +b? 4¢2) 14+ 1 C a* + +c*) [ (a+by +o] p+ES] (@tb+ey Now, notice that for any nonnegative real numbers x, y, 2, we have Q4+x\1+y)\d+2)> l+x+y+z, so that Te] (a-by (b-c (c-a? | (a+b) (a+b)? ° (b+? * (e+ayP 414 (a- bP +(b=c)? +(c—a)? _3@? +b? +0?) (at+b+c? (a+b+c? ” as desired. The proof is completed. Note that equality holds if and only if a c. Q Problem 2.5. Let a, b, c be nonnegative real numbers such thata+b+c Prove that a+ab+2abe < - Three-variable inequalities 17 First proof. Applying the AM-GM Inequality, we get 1 1 b+e+> 7 ab + 2abe = 2ab (+5) $2a = =2a ( 7 Therefore, it suffices to prove that 7-2a\?_9 a+2a(———]} <<. 4 2 After some simple computations, we can write this inequality as (4-a)(2a-3)?>0, 3 which is obviously true. Equality holds if and only if (a, b,c) = & Second proof. Replacing b = 3 —a~c, the inequality can be written as 9 ata(3-a~c)+2ac(3-a-c) <5, or equivalently, 9 F(a) = (2c + 1a? + (2c? - Se 4a + 5 2 0. We see that f(a) is a quadratic polynomial of a having its highest coefficient ‘riminant is positive. Moreover, its di Ay = (2c? — Sc — 4)? — 18(2c + 1) = (2e — 1)*(c? - 4c - 2) <0. This allows us to conclude that f(a) > 0. a Problem 2.6. Let a, b, ¢ be positive real numbers satisfying a+b +¢=3. Prove that be ca ab 3 + S53. va+3 Vb?4+3 Ve24+3° 2 (Vo Quoc Ba Can) 48 Vasile Cirtoaje - Vo Quoc Ba Can - Tran Quoc Anh Three-variable inequalities 49 Proof. From the well-known inequality (a + b +c)? > 3(ab + be + ca) and the given hypothesis, we deduce that ab + be + ca < 3. Using this in combination with the AM-GM Inequality, we get be be be 1 ( be be ) < = ab+be+ca, which is a well-known inequality. Equality holds if and only if a = b = Problem 2.8. Let a, b,c be nonnegative real numbers such that (a+ b)(b +ele+a)=2. Prove that (a? + be)(b? + ca)(c? +ab) <1. (Wasile Cirtoaje) Proof. Without loss of generality, we may assume that a > b > c. Then, we have 4(a? + be)(b? + ca)(c? +ab) < 4(a? +.ac)(b? +ca)(be + ab) = 4ab(b? + ca)(atc}? <(b? +ca+ab)(a+c)? <(b? +ca+ab+ be(a+c)?* (a+ bP(b+e)*(a +c) From this inequality, the conclusion follows immediately. Equality holds if and only if (a, b, c) equals (1, 1, 0), or (0, 1, 1), or (1, 0, 1). a Problem 2.9. Let a, b,c be nonnegative real numbers such that a+b? +c? =a+bte Prove that a?b? + bc? +.c2a? < ab + be +a, (Wasile Cirtoaje) Proof. By squaring, from the hypothesis condition, we get 2 2 at+bt4ct— = 2(ab + be +a — ab? — b°c? — ca). Therefore, the required inequality is equivalent to at+bttct za? +b te? This is true, since its homogeneous form, (a+b+c)(a4 + bt +e4) > (a? +b? +07), follows immediately from Hélder’s Inequality. Equality occurs for (a, b, ¢) = (1, 1, 1), for (a, b, ¢) = (@, 0, 0), for (a, b, c) = (0, 1, 1) or any cyclic permutation, and also for (a, b, c)=(1, 0, 0) or any cyclic permutation. Vasile Cirtoaje - Vo Quoc Ba Can - Tran Quoc Anh Problem 2.10. Let a, b, c be positive real numbers. Prove that (Vasile Cirtoaje) ae mu ato * Applying the AM-GM Inequality, we have b(a+b)+e(e +a) b(a +b) a= 3) a(b+c) x - peta pyran a DuneEE) aera = Re Thus, the proof is completed. Note that equality holds if and only if a = b c. Problem 2.11. Let a, b, c be positive real numbers. Prove that 2(ab + be +ca +1)* (a+b) +e +a)" (Tran Quoc Anh) Proof. Applying the Cauchy-Schwarz Inequality, we have 1 4 (arbres a) (ab44+be-+e0" babe) > (ab-+ boca 1 1 (arbrer J tact +ba*+cb? +abe)>(ca+ab+be+1 Tt follows that (ab+be+ca+1)?* = ab? + be + ca Fabe’ » lab be+eat 1? a?b+b?c-+c2a-+abc Three-variable inequalities 51 Adding these two inequalities, we get 1 a(ars+e + =) = (ab+bet+ca+1)? abe zt Diab? +abe 4(ab + be +ca +1)? (a+b\b+ceeo+a 1 Mab +abe . 4{ab+betca+1y? © Sa?b + abe + Dab? +abc Dividing each side by 2, the conclusion follows. Equality holds if and only if 1. a=b Problem 2.12. Let a, b, c be nonnegative real numbers. Prove that be a+b+c ps ear 2a+b 4 ile Cirtoaje) Proof. Applying the Cauchy-Schwarz Inequality, we have be be 1 1 1 ee 1 + ae Lqentera =e =) 1 be be 1 ca -{(Sasth =a) vss Us) 1 cat+b) at+tbt+e au atb 40 Equality holds for a = b =c, and fora = O and b=c, b=Oandc=a,c=0 and a=b Qo Problem 2.13. Let a, b, ¢ be positive real numbers satisfying a+ b-+c = 3. Prove that B+” aby 2 Proof. From the Cauchy-Schwarz Inequality, we have 1 PY 9 _ (a+b+eP < a? ‘ b? ee 4a2 +b? +c2 a2 +(a? +b2)+(a2 +02) ~ 2a? a? +b? a? +c? Therefore, be + (= mt from which the desired result follows. Note that equality holds if and only if a=b 1. o 52 Vasile Cirtoaje - Vo Quoc Ba Can - Tran Quoc Anh Three-variable inequalities - Problem 2.14. Let a, b, c be nonnegative real numbers such that The first inequality can be written as Prove that 2 a : Lasers ste bya Do. a? +2be ~ Under the assumption that a > b > c, it suffices to prove that be | ca, ab a?+1 b?41 C241 a? + 2be (Pham Kim Hung) Proof. We apply first the AM-GM Inequality and then the Cauchy-Schwarz In- equality to get be (b+c)* (b+c)* Pal 4+) Mer) +@+2)] a*(a— bya —c) a? + 2be 2(b — e)(b — B(b- b= a) , b?4+2ca Similarly, we have ca 1 (_ 2 @ e af @ which is equivalent to the obvious inequality P41 4 e+e ate)’ &+1~4\ e+e (a — b)?[a2b? + 2a°c + 2c(b — ca? + ab + b?)] 20. Adding up these three inequalities gives us the desired result. It is easy to see rhe eeenad inequality: equivilenten that equality holds if and only ifa = b =c a a atabtee - he) ab+be+ca a+2b~ Problem 2.15. Let a, b, ¢ be positive real numbers. Prove that Applying again the Cauchy-Schwarz Inequality, we get Po pR ee 2 2 a +b? +c @ b (b+c+a)* hacen areeemnare: b : _* eter Paheaarad Liaame 2 Kara) +b p20) Fale (Tran Quoc Anh) > HH > ab+be+ca~ a+abt+be b? +be+ca Thus, it is enough to prove that Proof. Using the Cauchy-Schwarz Inequality, we have a? +b? +0? 2(a+b+c) 2 2 4 ee ab+be+ca" 2(a?+b? +c?) +(ab+be+ca) @ 2a? a @ = < Faabtbe ~ G+ abe) + (a+ ab) ~ Da? + abc) | Aa?+2ab) . . a ‘ which is equivalent to the obvious inequality Using this inequality and the two similar ones, it suffices to prove that a quality 2 3 (a? +b? +c? — ab — be - ca)(2a? + 2b? + 2c? — ab — be — ca) 2 0. = +> x 2 + 2be Lage 2(a? + b? +c?) ab+be+ca The proof is completed. Equality holds for a = Moreover, it suffices to prove that 2 a? +b? +c? yr @ ab+be+ca a?+2bc’ Problem 2.16. Let a, b, c be nonnegative real numbers such that ab+be+ca =3. Prove that 1 it 1 a?+2 0 b?+2 a 2 ch 42 24. p24 (2 a+b +c Tr a ab+be+ca a+2b Vasile Cirtoaje - Vo Quoc Ba Can Proof. The desired inequality can be written as G 1 : 1 1 ' 1 1 2 a?+2 2 +2)” x aa) | or equivalently, a + ef + ; 1 i >is a?4+2° b?4+2 0° c24+27~ This is true since by the Cauchy-Schwarz, Inequality, we have ee 2 (at+b+c? @+2 b+2 +2 (a2 +2) +(b? +2) +(e +2) (a+b+cyP 1, 1). Problem 2.17. Let a, b, ¢ be positive real numbers. Prove that @-be | bemca -ab 4a? 44b2 402 | 4b? 442402 | 4c2 4402 +P Proof. Since 4(@?—be) (2b +e) 4a? +4b? +02 402+ 4b? +c?’ we can write the inequality in the form (b+ (2¢-+a)? (2a+ by? | 4a? +4b2 +c? © 4b?4+ 4c? +07 4c2+ 4a? +b? ~ By the Cauchy-Schwarz Inequality, we have - Tran Quoc Anh (2b+¢)2 (2b +c? 2b2 | aa tab +e 2a? +20) +2420) ~ +2 A and, similarly, (2e+a)* 2c? a Bo 4 a? = B22 * POD? @a+bP 2a? be? 4c? +4a? +b? ~ c?+2a? b?+2c% “(+b +e) 4 2ab + beta) The proof is completed. It is easy to see that equality holds when (a, b, ¢) = ao (Vasile Cirtoaje) + Three-variable inequalities 55 ‘Adding these inequalities, the desired inequality follows. Equality occurs if and only if : , a(b? +2c?) = b(c? + 2a”) = c(a? + 2b?). It is easy to show that these conditions are equivalent to a = b = c, or da = 2b =c, or 4b = 2c =a, or 4c = 2a=b. Oo Remark 1. Using the identities associated to the Cauchy-Schwarz inequalities above, we have 5 (2b+cP 4024 4b? +2 2 2(a? — be)?(2b —c}* 2b? ¢ = lz $abe Fa ea® PFI + 2a \(4a? + 402 + 325 (a= be}*(2b=eF_ (a? + 2b?)(c? + 2a)(4a? + 4b? +c? and (a? = bc)*(2b — )? a —be yr (a? + 2b?)(c? + 2a2)(4a? + 4b? +c?) Ligeaee Thus, the original inequality is equivalent to (b? + 2c2)(a? — be)?(2b — c)? yee 4a? + 4b? +c? ~ Remark 2. Similarly, we can prove that a*—be b? -ca 4 c?—ab So 2ka2 + kb? +c? " 2kb?+k2c2 +a? © 2ke? + k2a*+ b? ~ for any positive real number k. Indeed, this inequality is equivalently to ys (kb +c)? a 2ka? + kb? +c? ~~” and, by the Cauchy-Schwarz, Inequality, Ee (kb +c}? zy (kb +c)? 2ka? + k*b? +c! k(a? + kb?) + (c? + ka”) kb? 2 0, applying the Cauchy-Schwarz, Inequality, we have ca ca caf_l_ 2 4at+4b+c (2b+c)+2(2a+b)~ 9 \2b+e maa Adding this and its analogous inequalities, we get ca 1 ca 2ca 1 ca 2ca <5 & =e Laver ssl iat) a met les) 1 ca 2ab atbt+c =-(yY— + ae 3(Dapes Dae) 9? as desired, Note that equality holds if and only if a = b = c, or a = 2b and ¢ =0, or any cyclic permutation. a ‘Three-variable inequalities 87 Remark. ‘The desired inequality has the following equivalent form 2 (atb+cP> Fs +b*c+c2a +abc). Problem 2.19. Let a, b,c be nonnegative real numbers, no two of which are zero. Prove that 1 1 1 9 Arabakt * Pabete + Decata! — Geb +ee @+ab+b? b?+bet+c? c2+cata?~ (a+b+cy (Vasile Girtoaje) Proof. Multiplying each side of the desired inequality by a® + b? + c+ab+ be +ca, we can write it as o(Se+ De) a+b? +c2+ab+betca ae (atbter b? + be+c? or equivalently, (a+b+c)}* a 3+(a+b+e)) ae By the Cauchy-Schwarz Inequality, we have that a Ss. (a+b+c) atb+e x Be bere~ Sate? +be+e4) abt be tea Therefore, it suffices to prove that (a+b+e) _ %a?+b? +c? +ab+be +a) ab+be+ca (a+b+eP . Since a2 +b? +c? +ab+be+ca =(a+b +c)*—(ab + be +ca), this inequality is equivalent to 4 +b+eP abt be+ca) ab+be+ca— (a+b+c)? ’ or (atb+c)? %ab+be+ca) ab+be+ca (a+b+c)P which is obviously true according to the AM-GM Inequality. Note that equality holds if and only ifa = b =c. a 58 Vasile Cirtoaje - Vo Quoc Ba Can - Tran Quoc Anh Three-variable inequalities 59 Problem 2.20. Let a, b, c be nonnegative real numbers, no two of them are zero. Prove that 2 tao tt a 2h B+bet+c? ch+cata? (Vo Quoc Ba Can, Vasile Cirtoaje) Proof. Multiplying each side of the desired inequality by a? + b? +c? +ab+ be +ca, we can write it as @[a? +ab+b?+cla+b+c)] a rae 2 at, or equivalently, (=) (Uae)? Ze By the Cauchy-Schwarz Inequality, we have ae (ie)) Se x Pot SYie(a?+ab+b4) Sa’ and hence, it follows that (Xs) (Saar =a): dua ‘The proof is completed. Note that equality holds if and only if a= Problem 2.21. Let a, b, ¢ be positive real numbers. Prove that a 4 b + c 9(ab + be +ca) @tabtbe | be+be+ea ce+catab (a+b+c)* (Tran Quoc Anh) Proof. Multiplying each side of the desired inequality by a+b+c, we can write itas x a a(b+c) 5, Hab + be tea) ab+be P+ab+ be (atb+c)? By the Cauchy-Schwarz Inequality, we have 2 a (a+b+c? Sacre ab +be Sie +ab + be) = and y a(b+c) @Hab+be SY alb+ oat +ab+be) ia + dlab Therefore, it suffices to prove that 4(ab + be +b +c? +ab+bet+ca (a+ a) 9ab + be +ca) at b+c)* This inequality is equivalent to ve 2 = (+27 oy > = ab+be+ca a®+b? +c%+ab+be+ca Yes (Se?+ Yas) In this form, we can see immediately that it is a direct consequence of the Cauchy Schwarz Inequality. Equality holds if and only ifa = b = ¢. a Problem 2.22. Let a, b,c be the side-lengths of a triangle. Prove that Bab Bb+ce Seta. 2a+e 2b+a 2W+b~ (Tran Quoc Anh) Proof. The desired inequality is equivalent to 3a+b 3b+e Bc+a a\+ -1)+(=—-1)21, Qa+e 2b+a 2c+b atb-e b+ema | Qate | 2b+a — A%+b Since a, b,c are the side-lengths of a triangle, we have at+b—c > 0, b+c—a>0 and ¢ +a ~— b > 0. Therefore, we may apply the Cauchy-Schwarz Inequality to get sratice [Kero] (Da) 2ate > Sate =oQate) S1a? +25 ab Equality holds if and only ifa = b =c. o or a cta—b aS 60 Vasile Cirtoaje - Vo Quoc Ba Can - Tran Quoc Anh Problem 2.23. Let a, b, c be positive real numbers. Prove that 2atb 2b+c 2+a 2at+c 2b+a > iene =? (Pham Kim Hung) Proof. Without loss of generality, we may assume that ¢ = max{a, b, c}. Then, there are two cases to consider Case 1. 2b + 2c > a. Write the inequality as 2atb 1 x 2b+e 1 + 2ce+a 2atc 2 2bt+a 2 2c+b It is equivalent to Qa+2b—c Ab+2c-a | W+2a—b 2a+e 2b+a 2c+b Since 2a + 2b —c > 0, 2b + 2c —a > O and 2c + 2a — b > 0, we can apply the Cauchy-Schwarz Inequality to get 2 9(Xs) yrmet2d [Zea+26-]” | - date ~ S'Qat2b-c)2ate) 3910746 ab Case 2, a > 2b-+2c. In this case, we have 2b+a < (a—2c)+a = 2a—2c < 2a Therefore, 2atb | 2b+e_ 2a+b 2at+c 2b+a™ 2at+c In addition, we have Bre _ 5, 3b 2at+e "date >. 2c+a c+ (2b+2c) _, 2c+b Qc+b Adding these two inequalities, we get 2urieb) » Qbshi 2c+a Date Bb+a* 2+b” ‘The proof is completed. Equality holds if and only if a = a Problem 2.24. Let a, b, c be positive real numbers. Prove that a+b? +c? abe ca abt+be+ca~ b?+be+c? c2+cat+a* a?+ab+b>" (Tran Quoc Anh) Three-variable inequalities 61 Proof. Since a ab ab+betea BPH be+e? ca(ca ~ b?) (b? + bc + c2)(ab + be + ca)’ the inequality can be written as a(ca-b7) B+be+e2 ca(ca — b?) > (sso +a > Vic. This can be simplified to or c’a(atb+c) = 7 2 ca, b? + be +c? 22 or equivalently, ab+be+ca ca laa ee be+be+c?~ atbte Applying now the Cauchy-Schwarz Inequality, we get ea (Se)! Lager) * Fautab ea) Sa so the above inequality is true and our proof is completed. Note that equality holds if and only ifa=b =c. a ca Dupree Problem 2.25. Ifa, b, ¢ are nonnegative real numbers such that 7(a? +b? +c?) = 11(ab + be + ca), then 51 28 (Vasile Cértoaje) First proof. Due to homogeneity, we may assume that b +c = 2. Let us denote x = be, x < 1. From the hypothesis, we get _ 7a? — 22a +28 “* 25 é C0 LU 62 Vasile Cirtoaje - Vo Quoc Ba Can - Tran Quoc Anh 1 and then x < 1 yields 7 Sa <3. On the other hand, ay b ~*-_# a(b+c)+(b +c)? — 2be bt+e cta' a+b b+e a +a(b+c)+be =, Aa+2~x) _ 4a? +27a+11 2° @+2a¢x 8a +7a+7” and the desired inequalities become 51 _ 4a3+27a+11 287 8a7+7a+7 — These inequalities are true, since 4a? +27a+11 St Ga-Ta-™ 6 8a? +7a+7 28 © 28(8a2+7a+7) ~~” and 4a3+27a+11 _ (3-a)(2a-1)? 8a? +7a+7 Ba? +7047 — For the left inequality, equality holds for 7a = b = c or any cyclic permuta- a tion, while for the right inequality, equality holds for 5 = b = or any cyclic permutation. a Second proof. Notice that a? + 6? +c? > a? + (b+eP s , So from the given hypothesis, we deduce that 2 2 7[ee Cee) su [aosror Oe), and then, it follows that b+c)? Gxt 2 i and ab+be+ca < a(b +e) + 1 3 (b+) 0, we can apply the Cauchy-Schwarz Inequality to get that : ; 9 ye ygtto-s, [Z[ero~e]] 2 bt+e bte D[for0-e]o+0 - ay 3300? + Sab Furthermore, we have 4(Se)” _5_ uyab-7)@ 3+ ab 2 2 (302+ Yas) so from the above inequality, we deduce that a 9° 5 1 Lape S273 a 1 1 1 Similarly, since a 7 (b +c) 2 0, b~ =4(c-+a) = 0 and ¢~ +; (a+b) > 0, the Cauchy-Schwarz Inequality gives us e—oro [=[e-Ze+a]] Also, we have (Xe) 5 3 (11 fab-7 5a?) 13,ab— Sia? 16 16(13}))ab— 7a’) Combining this with the above inequality gives us that ot 2 SS e b+e7 14° 7 16 28° ? 64 Vasile Cirtoaje - Vo Quoc Ba Can - Tran Quoc Anh From (1) and (2), we deduce that 51_ a b ¢ 28 *b4c cra a+b as desired. The proof is completed. a #3, Third proof. Due to the homogeneity, we may assume that a+ b+c = 5. Then, we have ab + be +ca = 7. Let p = abc. The numbers a, b, ¢ are the roots of the equation f (x) = 0, where f(x) = x9 —5x2+7x-p. From f/(x) = 3x? - 10x +7 = (x — 1)(3x — 7), it follows that f has a local 7 maximum f(1)=3—p and a local minimum f (§ —p. All the roots of 3) 27 7 the equation f(x) =0 are real if and only if (1) > 0 and f (2) $0; that is, 49 5p

1. On the other hand, from the known inequality t, 2,2 (be +ca+ab)| —+—+—] 29, be ca ab we obtain a+ b-+c > 3abe. The desired inequality follows now by summing up the following inequalities 66 Vasile Cirtoaje - Vo Quoc Ba Can - Tran Quoc Anh We have ryt 2 eb) efb=0) b?+10 +10 bet+1 (b?+1)be+1) (c2+1)(be +1) (b= be-1) © (BF FINE + D(be +1) - and 1 4. 2 —be+3-3a7b atat+b+ce~3abc) | a+1 0 be+1l 2 a?+1be+1) Aa? +1/(be+1) ~ The proof is completed. Qo Problem 2.27. Let a, b, c be positive real numbers. Prove that a b? 2 Gat byaate) * @b+clab+a) * Getalactb) = 3 (Tigran Sloyan) Proof. Applying the Cauchy-Schwarz Inequality, we have a a a! (2a+ bate) 4a? +2ab+2act be (2a? 4+ bc) + Qala tb +c) a 1 2 <2 | + 9 | 2a? + be qarets| 1 2a a =c|— +> }- 9latbte 2a?+be Adding this and its analogous inequalities, we get a it a Learers <5 (+Tax) and thus, it suffices to prove that Be soa tea £1 2b? +ca © 2c? +ab~’ 2a? + be which is equivalent to be ca ab rage Oy bet 2a? * ca 2b% * ab+ 20 = Three-variable inequalities _ 67 Using the AM-GM Inequality, we have be br bec? _ bc? be+2a? bc? + 2a7be — bc? +a%(b? +07) b2c? + c7a2 +a7b? Adding this and its analogous inequalities, we get the desired result. It is easy to see that equality holds if and only ifa = b =c. a Problem 2.28. Let a, b, c be positive real numbers such that a+ b +c = 1. Prove that ab be ca soot oo st ooo Ss 3ab+2b+e Bbe+2+a 3cat+2a+b 4 Proof. Note that 3ab+2b+c=3ab+c(at+b+c)+2b=(ab+be+ca)+(2ab+c?)+b+b. Therefore, by the Cauchy-Schwarz Inequality, we have 1 é 1 1 ie 1 pig 3ab+2b+e~ 16\ab+bet+ca 2ab+e2 bb)” It follows that ab 4. ab ab SS i Deas eh (arise 2ab +2 ) 31 ab 315 = é 16° 1642ab+c2 16° 32 2ab+c? J’ In addition, from the AM-GM Inequality, a e ss 2 a= 2ab +c? a+b? +c? Combining this with the above inequality, we get y ab » 3 + 13 tas: 3ab+2b+c~ 16° 32 4 as desired. Equality holds if and only if a = b 68 Vasile Cirtoaje - Vo Quoc Ba Can - Tran Quoc Anh Problem 2.29. Let a, b, c be positive real numbers. Prove that B3 Cs @+ abe +O babe * Spal pabe 2h (Nguyen Van Thach) c : . a First proof. Setting x=, y= 5 and ==, we have xys=1, and a 1 1 xyz ye @B+bitabe yy ya, % l4x8+x2y xystittxty tay tye z So, we have to prove that ye 2x xy Stayt Pane a 2 xU4xy tye y?tystex "®t extxy 1, which is true since by the Cauchy-Schwarz Inequality, we have Y yz > Oztextxy? wet xy tye” Sys? txy bys) Equality holds if and only if x 2, ie. if and only ifa =b =c o Second proof. Multiplying each side of the desired inequality by a3 +b? +c3 + abc, we can rewrite it as follows ye (a3 +b? +c? +abc) - >a +b? 402 tabe a+b? +abe 2a° +b? +c? tabe, 93 Larera 2 c2q? pre tips Lies rag 2 Applying the Cauchy-Schwarz Inequality, we have ge 2 2a + 53 2 [= eas [Dove (a? +b3 +abc)] = (Se a). Since Soca? +b +abe) = (Mea)’, the conclusion follows. oa Three-variable inequalities Problem 2.30. Let a, b, c be positive real numbers. Prove that 1 1 1 3 esata — + : ava+b b¥b+e cvera~ Y2abe (Phan Thanh Nam) First proof. Due to homogeneity, we may assume that abc = 1. Then, there ,b=7,¢=~, Making were exist some positive real numbers x, y, # such that a= y n, we can rewrite the desired inequality as Fg EE, Vx0+y2) Vy? +2x) Vale? +xy) Now, using twice the Cauchy-Schwarz Inequality and then the AM-GM Inequal- ity, we get y? . Qa) Vey? +yz) V7 Vey? +y2) (ay) (Dey) [Le?+] ava (Si2)" : 2[/2de) (Sey) Ls and thus, it suffices to prove that this substitu ee 2v2(Sox)° +3) xy A(x ty +2)? 23x? +y? +27) +9(xy + ye +2x), which is equivalent to the well-known inequality eP+y +s? >xy+yetex Equality holds if and only ifa=b =c a si d a 2be _ 2ca _ 2ab ie 4 ‘econd proof. Let x = aaa Vege = Vara fe nee to prove that x + y +z > 3. Since (x+y +2)? > 3(xy + yz +2zx), it suffices to prove the stronger inequality 2s 2a 2b SS §-— ——— + ———— (a +b\b+aQ /(b+eeta) W(c+a)(at+b) 3Sxytye+ex= 70 Vasile Cirtoaje - Vo Quoc Ba Can - Tran Quoc Anh By the AM-GM Inequality and the Cauchy-Schwarz Inequality, we have 2a a a Lyra Lise “Lay At b+eP_ = Sat+3y ab +3) 7 ab" so it is enough to check that Aa +b +c)? > 3(a? +b? +c?) +9(ab + be +ca), or equivalently, a+b? +c? >ab+be+ca. Of course, this is true and so our proof is completed Oo Problem 2.31. Let a, b, c be positive real numbers. Prove that a+b a bree boc a” b+e be: a+b (Belarus 1998) bt+e 2); (atbtc)(a-c)? First proof. Note that the inequality is equivalent to a+b fero+e($ +3+8-2) @+i+a(¢ and thus it can be rewritten as cab, bc S+S-2a+b+0)2 c (a+ b\b+e) 2 2 —by eer -ay? ve etbteyn Oo by + Oo 2 _ feo ay a) 3 ca b ¢ a bc +b%c— a)? +-e%a—b}? 8 gap gee Pere + be~ a)? + 2a- by = b 2abe it suffices to prove that (a—b)* —= +t (c-a? _ (a+b+c)a-cy a (a+bb+e) Three-variable inequalities 71 By the Cauchy-Schwarz Inequality, we get (ab)? 4 (b-c)? _ (a-cP? b c b+e’ and thus, it remains to show that 1 1, atb+e bte a” (a+b)b+c)’ This is equivalent to bla+b+c) aE 5G) a(a+b)(b+c) ~ which is obviously true. Equality holds if and te ifa=b=c. o Second proof, After writing x = © ; and y = —, we get y a+b x+1 ‘Nam_opg ax’ be It+y’ bea xt1" One may rewrite the inequality as yt txty ity? > xy + 2oxy +2xry? Now, we apply the AM-GM Inequality to obtain xty?+x_ 4 Sytxtyi ty? 2Bxy, 2 Adding up these three inequalities, we get the desired result. o > axy?, x? 4y?> dxy, Third proof. The original inequality is equivalent to mt a(t Eft je ( b+) tle B+ (5-zen) Bag or , b? ac be a+2b + 4 b+) bb+e) * alatb)~ atb By the Cauchy-Schwarz Inequality, we have wae 4 be MOH ales bre) * +e) * ala+b) Fe) h(a) = (E/E y¥). 72 Vasile Cirtoaje - Vo Quoc Ba Can - Tran Quoc Anh Thus, it suffices to prove that b a be S| b (2 (E+ +E) 2 [SR oro+e+n)] b+ b pecaize SOTO eanienre) +(b+c)+(a+b)= (/2+/E+ which is equivalent to , this inequality becomes &)'s = aaa (a+ 2b\2b +e) ab | ac a +4 Spe bebeze> 2a b4b-+2¢ pe = cb b° or ab. be —+—22b Of course, this is true and our proof is thus completed, a Fourth proof. Multiplying each side of the desired inequality by b + > 0, we can rewrite it as atb+e) | b(b+e) | elb-+e) (b+c) a at ab tet ; b a +b which is equivalent to b 2 2 ee oe eS pe OEP b a ae a+b Now, applying the AM-GM Inequality and the Cauchy-Schwarz Inequality, we get ac BP? | (b+)? = en b boa a+b Adding up these three inequalities, the conclusion follows. o Problem 2.32. Let a, b, c be positive real numbers. Prove that a bie (a-c? >+24+-234+——— _ bt eta? 3* tbe tea (Yo Quoc Ba Can, Vasile Cirtoaje) Three-variable inequalities __ 2B First proof. Expanding and combining like terms, the inequality can be written as ab? ate be® b? + — + — + — 3 2ab + 2be. cb a It is equivalent to be? — +ab—2be a which is true since by the AM-GM Inequality, we have be? — +ab > 2be, a ab? ae ta +2 30d. ¢ The proof is completed. Equality holds if and only if a = b Second proof. We will consider two cases The first case is when c > 3a. In this case, we write the inequality as follows ab c-3a (c-a? £8 Set bv ea ~ab+be+ca Applying the Cauchy Schwarz Inequality, we have a c~3a_ (c— 3a)? s 4a? ‘ b* a 4ab * (e—3aa 7 ab+be * ca (2ate~3a)?__(e-ay? ““ab+be+ca ab+be+ca’ The second case is when 3a > c. Since ab+ be +ca > b(a +6), it suffices to prove that which is equivalent to id c ( +22 (3-£)b+ ; c a atc By the AM-GM Inequality, we have 74 Vasile Cirtoaje - Vo Quoc Ba Can - Tran Quoc Anh and thus, it is enough to prove that (a-c) _ c(3a—e)? ate > 42 | or equivalently, e(3a~c) , e(3a~c? ato ~ 4a? This is true since, by the AM-GM Inequality, we have 4a? > (3a—c)(a+c). Third proof. As in the first proof, we need to prove that 2, ab? ae, be? B+ 4+ +—— 2 dab + 2be, This inequality is equivalent to bbe? ca? Cad tenet (FM a pen ea) B(a- cf a or oe a ee Fla bP+(b-oP (ao OEE ar, ea. Heo >(a—cP. z = 2(a-cy. This rewrites as (a= bM(b+2c) | (b= cP e+2a) . (a e)a~2h) b ¢ = b . By the Cauchy-Schwarz Inequality, we get (a—b)°(b + 2c) cP(¢+2a)_ [(a—b)+(b-c)]}? b ¢ Bb c b42¢ * o42a _ (a~cP(b+2c)(e+2a) ~— Qab+2be+2c2—” and thus it is sufficient to show that (b+2c)(c + 2a) 5 222d 2ab+2be+2c7~ a ” Three-variable inequalities or equivalently, A(ab? + bc? + ca” — abc) +4b7c + 1labe > 0. This is true since, by the AM-GM Inequality, ab? + be? +ca® > 3abe. Remark. This inequality is stronger than Belarus 1998, which states that (a-c? a b b?+ab+be+ca™ bic +a4- 2334+ bic'a Problem 2.33. Let a, b,c be positive real numbers such that abe = 1. Prove that 81(a? +1)(b? + 1)(c? +1) <8(a+b+c)*. (Pham Kim Hung) First proof. Since a+) + Vewa = Ve (Vas Ve), we can write the inequality in the homogeneous form 81 (Var +¥V vet) (Vor Vera?) (e+ V0?) <8(a+b+e)*. Since the constraint abe = 1 became superfluous, we can now assume that a+b+c=3. Then, we may write our inequality as (Var+ Ve) ( bet By the AM-GM Inequality, it suffices to show that x (Vai+ Vee) <6. Applying the AM-GM Inequality again, we have D (ves Vee) <> (Se) _ (Se) +2a+3 3 This is what we want to prove. Note that equality holds if and only if a = 6 Vasile Cirtoaje - Vo Quoc Ba Can - Tran Quoc Anh Second proof. Without loss of generality, we may assume that c = minfa, b, ¢}, and hence ab > 1 > c > 0. Using the AM-GM Inequality, we have (a? + 1)(b? +1) = (a? + abe)(b? + abc) = abla + be)(b +ca) < zoote +be+b+ca)*= aC +b) (e417. ‘Thus, it suffices to prove that gg | arbre * by ae aap | Z8tabee? +e +, or equivalently, 2 c a+b 2 32 («++ +2) > 8lab(c? +1)(c +1) Since ab > 1>c > 0, it follows that atet oS -avab- = (va- vo)" [:- In addition, the AM-GM Inequality gives | 20. 2Vab(a +b) 1 2 A 2 @+1=s 241s Fy From these two inequalities, we see that it is enough to check the following inequality 2 2 (c2 +3)? 2 32( 2Vab+ +2c}) >8lab-———— -(c +1)”. (avai sp) eetar 28 tery ‘Taking square root of each side, this can be written as 2 4v3( 2Vab+ —— 4. 2¢ ) > 2 fable? +3 + D, ( a aan t* zy pyvae Me or equivalently, 8 (400 +4cVab+ a) > 9ab(c? + 3)c+ 1). i, 1 Setting t = Vab > 1, we have c = =, and the last inequality becomes v 4 1 1 \ 1 \ o(ar+ts >or? (+3 (a+1)- t ¢ oe eh Three-variable inequalities Since t > 1, we get 41 1 1 s+ 15t?+3¢+1)(t-1)% 8(4+-+— }-9 | >+3){ 541 = GEIS SET UET WY 9, tet e t and the proof is completed. Note that equality holds if and only if a = b L Problem 2.34. Let a, b, c be nonnegative real numbers satisfying a+b +¢ Prove that (a? +ab+b?)(b? + be +e7)(c? +ca +a?) <3. (Murray Klamkin) Proof. Without loss of generality, we may assume that a = max{a, b, c}. For b =c =0, the inequality is trivial, and so it suffices to consider that b +c > 0 In this case, we may write our inequality as E(a, 6, c) <0, where 3 E(a, b, c)=(a? +ab + b*)(a? +.ac +c?) - = ——.,.. ( =O mt b?+be +c? We shall prove that Ea, b, c)S E(a, b+c, 0) <0. The left hand side inequality is equivalent to (a?+ab+b?)(a?-+ac+c?)—a"[a?-+a(b+c)+(b+c)"] < or 3be be(ab + be +ea— a) < ——~___. (b? + be +¢7)(b +c)? Because be > 0, it suffices to prove that (ab + be +ca—a")(b? + be +07)(b +c)? <3. We have (a+b+c? (aoe 8 3 L 3 ~ (at+tb+cP? 4 ab+be+ca-a?< b+be+c? Multiplying term by term these inequalities, we get the above desired inequality. This proves the left hand side inequality E(a, b, c) < E(a, b +c, 0). The right hand side inequality, E(a, b +c, 0) <0, can be written as a°(b +c)*[a? +a(b +c) +(b+c)7] <3. From the AM-GM Inequality, we deduce that morose erry [ee] 2a(b+e)[a?-+(b+e)*] < [ 2 3 and . obras (A Combining these two inequalities, we obtain @(b+c)*[a? +a(b+c)+(b+e)"] = =a(b +c)-a(b+c)[a? +(b+c)*] +a°(b +c)? <1-241=3, as desired. This ends our proof. Note that equality holds if and only if (a, b, c) is a permutation of (1, 1, 0). o Problem 2.35. Let a, b, c be nonnegative real numbers, no two of which are zero. Prove that (b+eP? (ta? (a+b)? > ts 26. @+be b+ca bab @arij Grinberg) Proof. Without loss of generality, we may assume that a > b > c. In this case, we will prove that the following inequalities hold (ate)V¥a+(b+o)Vb> (a+b +2c)4] = q@ 2 ba? + be) +a(b? +a) < eather tte) 2 Me Slats +20) /2a+b)- va- Vo] 2 (@+b+2c)(va- vb)" (va+ v6) (va- vo) = V2a+b)+Va+Vvb This is true because (va+vo) [v2ta+b}+va+ Vo] >2a+b)Da+b+2. For (2), from the AM-GM Inequality, we have b(a® + be) + a(b? + ca) = abla +b —2c) + cla +b)? a+b)? 4 a + bP(a+b+2c) a . (a+ b-2c)+c(a+b)? Now, using the Cauchy-Schwarz Inequality in combination with (1) and (2), we get 2 (atb+2c(a+b) (ote? , +a, [(+e)vb+(a+e)va] a @+be b> +ca~ dla? +be)+a(b?+ca) ~ (a+b (a+b + 2c) 4 a Matb+2)_ 4 ~ a+b ath Thus, it suffices to prove that 4c (a+b)? + eer 2 a+b P+ab By the Cauchy-Schwarz Inequality and the AM-GM Inequality, we have 4c (a+b Qctatby? _ [@t+e)+(b+0P + 2 eg, a+b c?+ab ~ cla+b)+c?+ab (a+ce)(b+c) Therefore, the proof is completed. Note that equality holds if and only if a = b=c, ora=Oand b=c, or b=0anda=c, orc =O anda=b. o 80 Vasile Cirtoaje - Vo Quoc Ba Can - Tran Quoc Anh Problem 2.36. If a > b >c > d > 0 such that abed = 1, then tet 8 lta 1+b) 1+¢~ 14 ¥abe™ (Vasile Cirtoaje) Proof. It suffices to show that A 2 lta’ 1+b~ 1+Vab™ and 1 2 3 oe ite 1+V¥ab~ 1+ Vabe The conditions a > b > ¢ > d > 0 and abed = 1 imply Vab > Vat abc? > 1, The first inequality above is true because > land 4. 1 2 =( 1 3 1 \e(- 1 ) Trati+b 1pvab (ita 1¢vab) ‘\1+b 1+vab 2 _ (a=) (VaB=1) (1+a)(1+b)(1+Vab) © ‘To prove the second inequality, we denote x = Vab and y = Yabc. We have 3 ¢=75,x 2 y 21, and from abc? > 1 we get abc > Vab, and hence y* 2 x x Since 1 2 3 2 3 | $+ oo | l+e 14+vab 1+Vabe x*+y? ltx 1t+y x? 1 ( =| =—,-—— } +2 ! xe+y) lty _ Ge yP Ly = 2)x + 2y G+x)0+ yx? +y9) we still have to show that (y — 2)x + 2y? — y > 0. This inequality is clearly true for y = 2. For1< y <2, since y? > x, we have (y-2)e+2y? -y 2 (y-Dy> +2y?—-y=y(y- DO? -y + DZ Equality holds if and only ifa=b = a Three-variable inequalities Problem 2.37. Let a, b, c be positive real numbers. Prove that ab be ea b?+ca+ab cc? +ab+be™ a+b? ab+bc+ca ~ a? + be +ca Proof. From the Cauchy-Schwarz Inequality, we find that ab ab(b? + be +ca) ab(b? + be + ca) @+bet+ca (a? +bce+cab?+be+ca)~ (ab+be+ca) Using this and the two analogous inequalities, it suffices to prove that (a? +b? +0?)(ab + be + ca) >} ab(b? + be +ca). After some simple computations, we find that this is equivalent to ab +b8c+c8a > abc(at+b+c), which is a direct consequence of the Cauchy-Schwarz Inequality (a5b + b'e+c3alle +a+b) = abela t+ b+e)”. Note that equality holds if and only if a Problem 2.38. Ler a, b, c be positive real numbers. Prove that (Tran Quoc Anh) First proof. By applying the Cauchy-Schwarz Inequality, we have r a(a+2b) ) reread (= ates) (Ors So it is enough to check the following inequality zy a a? +2ab a+2b @ tab+be" This is equivalent to 2 a+2b Vasile Cirtoaje - Vo Quoc Ba Can - Tran Quoc Anh 2 a a be 3)'— soo t2 a= arab Deere rer Now, by using again the Cauchy-Schwarz Inequality, we get yr a (at+b+c)? a +ab+ be ~ 5 (a? +ab + be) 2 zy a, (atb+eP a+2b © S'a(a +26) and 5 (abtbetea® a +ab+be~ S*be(a?+ab+bc) From these three inequalities, the conclusion follows. Equality holds if and only ifa=b=c. Oo Second proof. Applying the AM-GM Inequality, we have 2Vv3a 3a? SS S > Gh /@+ab+be U+ab+be Therefore, it suffices to show that a 3a? 6 > eee Daw hese). or 2 a a 2. Dias? epee tt ac SE this can be written x b+bce+ca = Since a ac so ood 2 @+ab+be ab+be+ca) ~ which is true because 2a a? ac _ _ _ @b(a~c)? a a+2b a?+ab+be ab+be+ca (a+2b)(ab+be+ca)a?+ab+be) ~ Oo Three-variable inequalities 83 Problem 2.39. Let a, b, c be nonnegative real numbers such thata+b+c =3. Prove that a c aot et Vitb+be Vite+e I+a+ab (Phan Thanh Viet) Proof. According to Hélder’s Inequality, we have 2 (= (=) [Siac +0 + 60] > (a2? +679 4629), It is thus sufficient to prove that (a2/3 + 57/3 +c7/3)3 > 3a(1 +b + be) + 3b(1 +¢+ca)+3c(1+a+ab), or equivalently, (a?/3 + b/3 + 07/3)8 > 9 4.3(ab + bc +.ca) + 9abe By the AM-GM Inequality, we have 3 = a+b-+c > 3 Vabc, so that a2/3p2/3¢2/3 > abc. Therefore, (02/9 479 4.299 = Sad 43S a/b (a2! +B) + 602/95275-2/9 > YieP? +6 ab + babe, and it remains to prove that a? +b? +c? + 6(ab + be +ca) + babe > 9+ 3(ab + be +ca) + abe. This is equivalent to ab + be +ca > 3abe, which is true because ab + be + ca > 3Va2b%c? > 3abc. Our proof is now completed. Equality holds if and only if (a,b,c) equals (1, 1, 1), or (3, 0, 0), or (0, 3, 0), of (0, 0, 3). o Problem 2.40. Ifa, b,c are positive real numbers, then 11 3 nfi,1_1 (a+b+c)(-+— 21+ 4/1+4/ (a? +b? +c?) ats | a'b be (Vasile Cirtoaje) — 84 Vasile Cirtoaje - Vo Quoc Ba Can - Tran Quoc Anh Proof. Using the Cauchy-Schwarz Inequality, we have 2/(L#) (Xa) and hence 1 a =)}-1 [fese)-] - From this inequality, the conclusion immediately follows. Equality holds if and (ie)(S8)= (24) oe). which is equivalent to (a? — be)(b? — ca)(c? — ab) = Consequently, equality occurs for a? = be, or b® = ca, or c2 = ab a Problem 2.41. Let a, b, c be nonnegative real numbers such that ab+be+ca =3. Prove that a(b?+c7) bc? +a?) cla? +b?) _ @+be bP +ca ce +ab (Pham Huu Duc) Proof. Taking into account the known inequality (x-++y +2)2 2 3(xy +y2+2x) it suffices to prove the stronger inequality x be(a? + b*)\(a? +c) S (OF calle? tab) ~~ is order to homogenize this inequality, we replace the right hand side by be. ince be(a? + b*)(a? +c) bela’ + b*\(a? +62) be(a3 — b3 —c3 +. ab? + ac? — abc) (b? +ca)(c?+ab) fe OP mead ee ae (+ caer +a ec ere Three-variable inequalities we have to show that Sita? + bej(a? — b3 - 3 +ab? +c — abe) > 0. This inequality is equivalent to Yak + 2abe } 1a? =F be(b? +7) +abe } Ve, Tarla—Hla~)+ Zabe Ho ~eP Zo. or 3(q — b)(a—c) > 0 is the special fifth degree Schur’s Inequality, the =I Qo Since 7 inequality is clearly true. Equality occurs if and only if a = b Problem 2.42. Let a, b,c be nonnegative real numbers satisfying a + Prove that iad 33 ava? +e?+ bV ac? +a? eV 4a? +b? <5. (Vo Quoc Ba Can) Proof. Consider the nontrivial case when two of a, b, ¢ are nonzero. Then, with the help of the Cauchy-Schwarz Inequality, we get (Xe Vaeee) < [Maas+o] [= | 2b+c (de) [ES5"]. hence it suffices to prove that b(4c? +.’ 2c+a 3 a(4b? +0?) c(4a? + b?) 16(ab+be+ca)”~ 2b+c . 2a+b Since a(4b*+c?) a(2b+c)* —4abe 4abe ee 2 +c) - =, 2b+e 2b+e 2b+e this inequality becomes +4abi + 1) > a(ab+ be +a) — ¢(——— + . abt beteay Bbc Weta dard ON ONG Using again the Cauchy-Schwarz Inequality, we see that 1 1 1 9 ——+-— +. > 8 2b+c 2+a 2a+b~ (2b+c)+(2c+a)+(2a+b) 86 Vasile Cirtoaje - Vo Quoc Ba Can - Tran Quoc Anh and so, we need to show that 1 T6lab + beta) * *bc 2 ab+ be + ca. Let us denote q = ab +be+ca,0 ab(a? + b*) + be(b? +c?) + calc? +42), and thus, in terms of q, we get abe > =A ~a) 6 Therefore, +4abe — ab — be -ca= + q+ 4abe T6(ab + be +ca) 16q > 44 2(4q- 0-0) = 164 qd 3 q @ eae _ G=8a)0~40? 48q - Equality holds if and only if a= b =0or any cyclic permutation. 0 2 Remark. Actually, the slighly more general inequality {apna 2 1 aVEP +E + VK +a eV Bat 4 bP < max | VE t1 R+T 3 4 holds for all positive reals a, b, ¢ satisfying the condition a+b +c = 1. However, the proof is quite similar to the above one we presented. We shall leave it as an exercise for the readers. Problem 2.43. Let a, b, ¢ be nonnegative real numbers satisfying a+ b-+¢ = Prove that 1 oe ab +8” be +8 cal +8 > (Vasile Cirtoaje) Three-variable inequalities 87 Proof. Expanding and combining like terms, the inequality turns into 64> 13 +(16-SrJA+5r(A+B), where r=abe, A=ab?+be?+ca®, B=a"b+b’c+c7a. From AM-GM inequality, we get r < 1. Also, by Schur’s Inequality (a+b+c)? > 3abe + 4ab(a + b) + 4be(b +0) + 4eale+a), we get 3099-1) A+B0,A<4-randA+B< 39 — ets , it suffices to show that 5r(o— 64> Pe a6—sya—n+ Oo This inequality is equivalent to r(r—1)(4r +9) $0, which is clearly true. Equality occurs for (a, b, c) = (1, 1, 1), and also for (a, b, c)=(0, 1, 2) or any cyclic permutation thereof. a Remark. We can prove the inequality ab? + be? + ca? + abe < 4 using the rearrangement inequality, Let (x, y, z) be a permutation of (a, b, c) such that x2 y >z, Since xy > zx > yz, by the rearrangement inequality, we have ab? + be? +ca? =b-ab+e-be+a-casx-xy+y-2xtz-yz = y(x +2) —xyz=y(x +2)? —abe. Consequently, it suffices to show that Warts? <4, which follows immediately by the AM-GM Inequality priest ctessi 2y(x+z)P < [ z 88 Vasile Cirtoaje - Vo Quoc Ba Can - Tran Quoe Anh ‘Three-variable inequalities 89 Problem 2.44. Let a, b, c be nonnegative real numbers. Prove that Second proof. The case (a + b)(b + ¢)(¢ + a) = 0 is trivial, so let us condider j _ now that (a + b)(b +c)(c +a) > 0. Due to symmetry, we may assume that a? +b? +c3 + Babe > aby 2a? + b) + be 2b? +2) + caV/2 +a). a> b>c, From the AM-GM Inequality, we have (Nguyen Anh Cuong, MOSP 2004) 2abV2(a? + b?) < aa? + b*) + 2ab?, First proof. Without loss of generality, we may assume that c= min{a, b, ¢} Then, by the AM-GM Inequality, we have : 2b? 4 bee? oa? eo® ec? bVAP+A +aVAera < eee eet £ 2 2bey 2(b? +c?) < c(b? +c?) + 2cb?, ED Cale? +a? 2eay 2(c? +a?) <

abV 202+ b2) +03 + seta? +b), 2a5 + 2b3 +.2c3 + babe > a3 +c3 +3b%(a +c) -+2abe + a or equivalently, or equivalently, 20° +265 — 2ab/2(a? + b2) > 3c(a — by? . ent? de) a +03 +269 + dabe > 36% +c) +———. We have b 2a° + 2b? - 2ab (a? +b?) = After some simple computations, we can rewrite this inequality as 2 a? +b? > 24. gd = 2(a° +69) —(a + b)(a? + b2) + 5 [Cat 6) V2(a2 + 6%) - 4a] (a b)(b-c) (= = ~ab+a+0) >0, b = a? +b? ——_ 4 =(a+b\Xa-bP +4 . 23 by - 2 2 [e+ yas 4ab | which is true because (a — b)(b—c) > 0, and Since (a+ b)(a~b)? = 2c(a—b)*, (a+b) /2(a2 + b2)—4ab > (a-+b)?—4ab = a +c? a? +b? (a—b)? >0, and 7 26 it follows that ° 2 + b? 4 (a+bya-byr+ 4/5 5 [e+ bv20@? 05 ~4ab] > Bea ~ by, as desired. Note that equality holds if or b=Oandc=a,orc=Oanda aw abtatez — +a-2b20. The proof is completed. a Third proof. As in the previous proof, we see that it suffices to consider the and only ifa=b =c, ora =O and b=c, inequality in the nontrivial case when two of a, b, c are nonzero. Without loss b. o of generality, we may assume that a > b > c. Then, by the AM-GM Inequality, S 2ab(a+ b)+ blab)’, ae coi be (20? +e) < be [ores 2 =2be(b +c) + Aor bte +e S2be(b +c) +c(b—c), 2 cay 2(c* +a?) 3a7b + 3a°c + 3b7c + b? + 2c3. After some easy computations, we can rewrite this inequality as (a — bP*(2a+ b-3c) > 0, which is obviously true. The proof is completed. o Problem 2.45. Let a, b, c be positive real numbers. Prove that _ pe Trace byfa—ey2 ———Aabel =F G@+b+ola+ byb+oe +a) (Nguyen Dinh Thi) Proof. First, we note that 2+ Lieber +sare= Fiala byes =a “(sect — Fez) tole) =P AELMOHH , adler Ket) bt+e b+e _ yr ab(a t+ bb +c) ab(a+b)(c+a) x c+a +2 b+e ~ Leow (St) 90 Vasile Cirtoaje - Vo Quoe Ba Can - Tran Quoc Anh we have 2(a? + b? 2 2abs/ 2a? +b) < ab [« +b+ wee) =2ab(a +b) + amen or Three-variable inequalities Therefore, Mata - b)(a-c) = 1a? + 3abc - } habla +b) = +P abla + b)+3abe~ 2) ad(a +b) b+ce cta = Liaba+ o (e+ be ) =P abla+b)(a— by a (a+c)(b+e) and hence, the desired inequality can be written as > 4abe(a? — b?)* ~(atb+clatbb+ci(ct+a) | ab(a+ b)(a— by? arobey- (a+c)(b+c) (a+b)(b +c)(c +a) abc 2_ p22 5 ea eP atb+c Multiplying each side of this inequality by , we get (ab? (2B (a? 2? € a b Since 4(a?—b?? _ [(a? — b?) + (c? — b*) + (a? — c?)}? atbt+c — c+atb ° the last inequality is a consequence of the Cauchy-Schwarz Inequality. Note that equality holds if and only ifa = b =c. a Problem 2.46. Let a, b, c be nonnegative real numbers satisfying a+b+¢ Prove that a(b+c cta 5 (b+c) li (c +a) atl b+1 c(a+b) c+1 ] = A(ab + be +ca)+3abe. (Vo Quoc Ba Gan) Proof. By the Cauchy-Schwarz Inequality, we have a(b-+c) 14 4ab + \s57_ a be(St+ a) Therefore, it suffices to prove that x 20ab 5—¢ Y4ab + 3abc, 92 Vasile Cirtoaje - Vo Quoc Ba Can - Tran Quoe Anh: Three-variable inequalities which is equivalent to Since + A+be 1 Veep Pete Lae Lew 9 this inequality is equivalent to ~ab) 2 3abe. This can be written as 1 4abe Y) — > 3abe 5-¢ Now, using the Cauchy-Schwarz Inequality again, we get 2at+b4) 4(ab+be+ca)* Sip Lge taper PLU HPN +4 Dab, or aoe Mo-o 15 so that the last inequality is true and our proof is thus completed, Equality holds 2¢a" +b) . ab+be+ca > [er -e-e'] =4(De) (- She) This rewrites as 2_ p2)2 yo b?); a OEE ee Shi, err thee if and only if (a, b, c) is one of the triples (1, 1, 1), pH): (5. 0, 3) ang 3 (0, ? i} (3, 0, 0), (0, 3, 0), and (0, 0, 3). Oo which is true because —b?)? 2ab+ be +ca) a? +b? a? +b? +c (a — b)?(a? + b? — ac — bc)? (+b? +b? +2) (a- b= Problem 2.47, Let a, b, c be positive real numbers. Prove that The proof is completed. Note that equality holds if and only if a = b a b e atbte —+§ —— +$ —__ Problem 2.48. Let a, b, c be nonnegative real numbers such that (Walther Janous) s 6 4b 4c — 3022? = Proof. Squaring both sides, the original inequality can be written as ah + DoE eO 3a°biel = 1 Prove that a? +b3 +3 — 3abe < V2. Eee RE La 5 (Xe +2Ne0). Applying the Cauchy-Schwarz Inequality together with the AM-GM Inequality, we get (Wo Quoc Ba Can) Proof. Due to symmetry, we may assume that a = maxa, b, c}. Write the desired inequality in the homogeneous form eb? b+b 2 ; = CAD DEE ae (a3 + b3 +c3— Babe}? < 2(a® + b§ +c — 3a2b%c?), V@? + bb? + 62) VV ee +570 +c) a? +b? +b? +c i Using the Cauchy-Schwarz Inequality, we have _ (ab + be +ca)? (a? +b? +c3 —3abc)* = [(a? — abc) + (b> +3 — 2abc)]? 2a? +b? +02)" . “ 2[(a? - abe)® + (6? +9 — 2abe}*]. So it is enough to prove that ‘Thus, it suffices to show that @ x a’ (abt be +ca?® fon @apeg = 5 (Xe+25a0) —abc)® +(b +3 — 2abe)? < a® + b§ +.c5 — 3a7b7c? 94 Vasile Cirtoaje - Vo Quoc Ba Can - Tran Quoc Anh. After some simple manipulations, we can write this inequality as 2be(a* + 2ab? + 2ac? — bc? ~ 4a2be) > 0. Since 2be > 0 and 2ab + 2ac? > 4abc VIc, it is enough to check that a + 4abceV/be — bc? — 4a2be > 0. ‘This inequality is equivalent to (a- Voc)’ (a? +24 bebe) > which is obviously true. @no=(% va 7° , or any cyclic permutation. Problem 2.49. Let a, b, c be positive real numbers such that Prove that 1 (at + bt +04) (z+ @ Proof. Without loss of generality, assume that a > b. Let a+b-o(5 +E-2 @+o-0(2 +5-3) 161 boc 1 +4 Ft ae) 22304 (Vasile Cirtoaje) r)=@+n(2+ +i)- [erme+(5+ ie] 7. On the other u hand, using the Cauchy-Schwarz Inequality, we get 1 Gt tuteet (4 a 1 pt 2)2 [ rn (Sep) 1) =(#+34) = [(«+3) 3] > (72-1)? = 2304, It is easy to see that equality holds if and only if é > 1. Si S =u,u 21. Since b 2 Three-variable inequalities 95 as desired, Equality occurs when ab = 7. For a > b, equality a conditions are equivalent to — = o c Problem 2.50. Let a, b, c be nonnegative real numbers. Prove that (a? +ab + be)(b? + be +ca)(c? +ea + ab) 2 (ab + be ca)’. Proof. Due to cyclicity, we may assume that b is between a and c. Applying Hélder’s Inequality, we get 3 (@ +ab + be)(b? +cat bead +ca+c2) > (ab+a¥ pet +cVb%) , so that it suffices to show that Since be? +c V b%e— be - ca = a( Vee? bc? — c)+e V bec — be = Ve (a- Vere) (Vo- 2) =¥2[(e-»(¥o- ve) + VP (ve-¥)'] the inequality is equivalent to 2 (a-»)(Vo- ve) + Ve (Vo- ve) 20, which is true since ¥/b? (7B — Ye)” > 0 and (a - b) (¥B— Ye) > 0 (from our assumption). The proof is completed. Note that equality holds if and only if Problem 2.51. Ifa, b,c are positive real numbers, then 2a 4 2b ~ 2c z Vath Vore cta™ (Vasile Cirtoaje) 96 Vasile Cirtoaje - Vo Quoc Ba Can - Tran Quoe Anh First proof. By the AM-GM Inequality, we have x => 3a(b+c) —Sath+e) _, Mab besten) a+b [aos 3(a+b)(b +c) 3(ab + be +ca) iL = qa Let a 3 Lerhere 3, Mab+be+caatb+e) 34 4abe 21" 3@4byb+cKeta) 2°3" Harb\brelera) By4, abe 23 3(2vab) (2Vbc) (2 Vea) Equality holds if and only ifa = 6 =c. a Second proof. By the Cauchy-Schwarz Inequality and the AM-GM Inequality, we have 2a 2a Dares (Lera)] [Casnera] _ /8latb+e)ab+be+ea) _ Babe “V(arbb+oneray — V8* Grbyb rene ta) 8abe E (2¥ab) (2vbc) (2Vea) Problem 2.52. Let a, b, ¢ be real numbers. Prove that (a? +b? +c}? > 3(a3b + bc + c2a). (Vasile Cirtoaje) Proof. Applying the well-known inequality (x+y +2)? > 3(xy +y2+2x) with x=a?+be—ab, y=b?+ca—be and z=c? +ab —ca, we get 2 [ie + be ab)]" 23 Ya? + be ab\(b? +ea— be) = 3(a°b + 63 +00). On the other hand, itis clear that [he +be -ab)|” = (a2 +b 402), Three-variable inequalities _ 97 Therefore, (a? +b? +c?) > 3(a2b + bc +c a). Equality occurs for (a, b, ¢)~ (1, 1, 1). Besides, the inequality becomes again x . . equality for (a, b, c) ~ | sin 5 su? 5) or any cyclic permutation thereof. a Problem 2.53. Let a, b, c be nonnegative real numbers, no wo of which are zero. Prove that a b c 3 7a-c)* ——+—— + —— 2 5+ b+te cta a+b 2 16(ab+be+ca) (Pham Van Thuan, Vo Quoc Ba Can) Proof. It is easy to see that it suffices to consider the case a > b > c. We write the inequality as ys be +ca) bt+e a(ab+bet+ea) abe. Fh =a? +—, it is equivalent to b+e 3 7 ° Seah +L (a 2 Fab + be +ca) + Fac) Since b+e a+b +c Pavey > 5 Zeab+be+ca)+7 ao By the Cauchy-Schwarz Inequality, we have > a b+c™ 2(a+b+c) Therefore, it is enough to check that 9abe ————— egy ait + > flab be bea) 5 ga ey. ath eet Setting now a =c+x, b=c+y (x > y 2 0), after some easy computations, we can write this inequality in the following form (11x? — 32xy + 32y7)e + (x + y)(3x —4y)* > 0, which is true and our proof is thus completed. Under the assumption a > b > c, we have equality when a= b =e, and when a= 5b and c=0. a 98 Vasile Cirtoaje - Vo Quoc Ba Can - Tran Quoc Anh Problem 2.54. Let a, b,c be positive real numbers such that abc = 1. Prove that 1 2 (+c? + (+a) +b) +c) (Pham Van Thuan) First proof. Because a, b, c > 0 and abc = 1, we may put a = 2%, b = *%, wey xy c = “yy where x, y, 2 are some positive real numbers. Then, our inequality becomes 4 4 4 2y22 x z 2: 2 2 * 2 ~ 2 ot 2 2 + 2 —— 2h GP + ys? (tax? ” et xyP "(yey tex + xy) Using the Cauchy-Schwarz Inequality, we get x xe ¥ at _ Yox'07 +27) GP ty? © S08 ryAGF+2) OVA ey 2x2y2e2 ete (7 + yy? + 2?)(2? +2?) Therefore, it suffices to show that (2? + yy? +292? +27) > (x? + yey? + ex )(2? + xy). This inequality follows by multiplying the Cauchy-Schwarz inequalities (7 + y)P +24) (x? + yz), 7 +27)? +x) > (y? tax)’, @4+xe Ve +y2)>@+xy?. Equality holds if and only ifa=b=c=1 o Second proof. There are two of the numbers a, b, ¢ either greater than or equal to 1, or less than or equal to 1. Let @ and b be the numbers having this property, that is (1 — a)(1 — b) > 0. Using Cauchy-Schwarz. Inequality, we have 1 a—l_s 1 4 1 1 2 +o = > G+ay? * (1+) (a+ad) (1+2) aren(i+2) 1+ab a and thus it suffices to show that — +o Itab © (+e? " G+a)G +b) +e) ~ Three-variable inequalities 1 Substituting c by ab’ the inequality becomes 1 ap? + 2ab ip * Gripe * Graa + babe D ~ l+ab This inequality is equivalent to ab(1 —a)(1—b) 5 (+a) +b) +ab)* ~ 7 o which is clearly true. The proof is completed. a nd Lolite 1 lity) 1 tte ae EY ana = . tat is Tuedareel 88 7 21th” 2 ite 2 = 1s 2 | a ee 14x l+y T+2 that x+y ++xyz =0 implies 4x? +0 +y? +0 +240 4x) +y)+s) = 4, that is : x?4+y?+2742%x+yta2)txy+yz+2x 20. (ety +2? -2x?- a? 2 Since xy + yz +x = into , the inequality transforms ry ter taxtyts)t(xty ts 20. Substituting x + y +2 by —xyz, the inequality becomes x+y? 4224 x2y2e2 > axyz, By the AM-GM Inequality, we get x? + y? +27 +x2y22? > 4 V/xty424 = dlxyz| > days. The proof is completed. Problem 2.55. Let a, b, c be nonnegative real numbers, no two of which are zero. Prove that be bere) (ote te ]23 (abs betea)| Tae ree Grae 2a (iran 1996) 100 Vasile Cirtoaje - Vo Quoc Ba Can - Tran Quoc Anh. Proof. Without loss of generality, we may assume that a > b > c. Then, we will show that 1 i 1 (b+c? Ge +c)? c (a+b)? 1 a+c\b+c) 4ab Indeed, this inequality is equivalent to 1 1 2 ss 1 1 (b+cP" (atc (ate +e) ~ 4ab- (a+b or (a-b)? 5 _@ by (a+cP(b+cP ~ 4ab(a+b)?’ which is true because (a —b)? > 0, (a+b)? > (a+c)? and 4ab > 4b? > (b-+c)°. Now, using the inequality (1), we see that it suffices to prove that 2 1 9 b ———__+—_]>7 (b+ +0 aerate) 2s ab+be+ca 1 cla+db) | Aab+be+ca) ag Since => an =2- ; 4ab 4° 4ab (aFeyb+e) (a+c)(b+c) ja it can be written as c(a +b: 2 “gab = @FoQweHa’ % (|+b\Mb+e\c+a)> Babe By the AM-GM Inequality, we see that this inequality is true, and hence our proof is completed. Note that equality holds if and only if a = b =c, or a = b and ¢ =0, or any cyclic permutation. a Problem 2.56. Let a, b, c be positive real numbers such that ab + be +ca =1. Prove that +0? 5 ——7 25: eta)? ~2 1+a%b? 1+ bc? (a+b? (b+e? (Wasile Cirtoaje) Proof. Since 1¥a?b? _(ab+be+ca)+a7b? 2ab(ab+be tea) +cat b)? (@+bP (a+ bP ~ (a+b}* 2, _2ab 2 2ab — = YH (a+b)? ab+be+ca * (a+b)?” ‘Three-variable inequalities - 101 we can see that the original inequality is equivalent to +b? +c2 2ab ac ‘ 2be Ss ab+bet+ca © (a+b)? " (a+c)? (b+c?~ 2 Without loss of generality, we may assume that a > b > c. Then, we will show that 24 p2 5 a+b 2ab 2ab 1 2cla+b a ee _ gy tale) ab+be+ca (a+b? ~ ab+be+ca 2° (a+c)(b+c) a Indeed, this inequality is equivalent to (a—b (a-bP [ atb a b ] ——— - ae ab+be+ca (a+b ~ (a+clb+e) (atc)? (b+c? or by (a— bP 2c?(a— b)? ( ab+be+ca a+b)? ~ (b+ce2(atce Since (a — b)? = 0, it can be simplified to 1 1 2c* Barby * +oato" ab+be+ca 1 3 ich is true because ———[ < ——_~____, which is rue because 5 S grape 7a Sc? (b+ Plate? and 3 1 1 Sonn + eo 8(ab+be+ca) 2(ab+be+ca) ~ab+be+ca Now, using (1), it suffices to prove that 2ab +c? 2c(a +b) + >. ab+be+ca (a+e)(b+c) But it is true because 2ab +c? 2c(a + b) c%(a—c)(b-c) $6 ab+be+ca (a+e)(b+c) (ab+be+ca)(a+e)b+c)~ so our proof is completed. Note that equality holds if and only if @ 1 3 102 Vasile Cirtoaje - Vo Quoc Ba Can - Tran Quoc Anh Problem 2.57. Let a, b, c be positive real numbers. Prove that a+b +c? 3(a3b+ b3c+c3a) ab+betca * 2b +b +a = (Bach Ngoc Thanh Cong) Proof. Multiplying each side of the desired inequality by a?b? + b?c? +c2a?, we can write it as 2 242 (Xe Ue 5 ) +3024 ae. a Since Y)a*6? = (Sab) * _ 2abe Dia. itis equivalent to (ce) (La) aZer- Tors NE) By expanding and combining like terms, it can be written as 43 107b + Yab3— 4 a2b? > ane) abe )\a, av or equivalently, By the Cauchy-Schwarz Inequality, we have [Sav(2a- 0)" (Sea?) = [= Varbe(2a— | 7 =abe (2e?- Ya)? It follows that a Vavb(2a- 5)? > and so, we only need to prove that (eXe-Les) (Lea) (Le) (Tee). Three-variable inequalities 103 This inequality is equivalent to 23 a5b + Dab? - which is true because Yas? - abe a2 0, Ver = ore zaie ED =abe )oa, Died + Dab? = Yab(a? +b?) > 25 a?o?. The proof is completed. Note that equality holds ifand only ifa=b=c. 0 and Problem 2.58. Let a, b, ¢ be nonnegative real numbers such that a+b +c =3. Prove that (2+ab?)*(2+ be? )3(2 + ca”)? < 3456. (Tran Quoc Anh) Proof. Using the well-known inequalities ab? + bc? + ca? < 4 —abc and a2b + b?c+c%a < 4 —abc (see Remark from the proof of problem 2.43), we get (2+ab*)(2 + be?)(2+ca?) = =8+4(ab? + be? + ca”) + 2abe(a2b + be + c2a) + a3b3¢3 <8+4(4— abc) + 2abe(4— abc) +a3b%c3 = 4(6 + abc) + a7b?c*(abe — 2) < 4(6 + abc). It follows that (24 ab?)?°(2-+ be?)*(2 +.ca?)? < 16(6 + abe)’. Using this result in combination with the AM-GM Inequality, we obtain (2+ab? (2 + be?)9(2 +.ca?)? < 16(6 + abc)?2(2 + be?) 16 fat 213 < 37 [26 + abc) +2 + be’ ] = [14 + be(c +.2a)}*. Since bele +24) =2-2b-c-(c+20) < Cbtetet 2a? _ ee 3 = 34 ~ (by the AM-GM Inequality), the conclusion follows. Equality holds if and only if (a, b, c)=(0, 1, 2). a 104 Vasile Cirtoaje - Vo Quoc Ba Can - Tran Quoc Anh Problem 2.59. Let a, b, c be positive real numbers. Prove that 2a)? 2b 4 14.2 7 Hat b+e? 1+) +(144 a} ~abt+be+ea™ (Pham Kim Hung) Proof. Notice that “(20+ bY 2a\? b E(3)-2" so we may apply the Cauchy-Schwarz Inequality to get 2 a (oye Bees] bd} av , and thus, it is sufficient to prove that carn ft+arrof2rarrafezaars +c), Applying now the AM-GM Inequality, we get 22+ b[f =20+8)]¢- lab >2-2Vab- fe — 24? 40 Adding this and its analogous inequalities, we obtain the above inequality. Note that equality holds if and only if a= b=c. o Problem 2.60. Let a, b, c be positive real numbers. Prove that wee at beenalarsseecy(t4 242 Rt a a ce (+b +c) o+ote (Pham Huu Duc) Proof. Without loss of generality, we may assume that b is between a and ¢ that is, (b — a)(b —c) < 0. Now, applying the AM-GM Inequality, we get 2 2 2 (Pane (F4e42) a2 @HHe (b V boca b c 2 Three-variable inequalities 105 and thus, it suffices to prove that 2 be —+c2—4+—. a ab This is true because ct be c(b —ay(b—c) Le pp Cole eros ab ab = 0. ‘The proof is completed. Note that equality holds if and only ifa=b=c. 0 Problem 2.61. Let a, b,c be positive real numbers. Prove that 4a?b?c? > (a + b—c)(b +c -a)(c +a— b)(a> +b? +3 + abc). Proof. Due to symmetry, we may assume that a > b > c. Fora > b +c, the inequality is trivial so let us consider the case b +c > a. In this case, we write the inequality as 16a?b2c?(a +c)? > 4(a + c)*[b? — (a —c}*I(c +a — b)(a? +b? +c* + abc). By the AM-GM Inequality, we see that the right hand side of this inequality does not exceed [(a + c)*(b? —(a—c)*) + (c ta—b)(a? +b? +3 +abc)]?, and thus, it suffices to prove that 4abe(a +c) > (a +c)*[b? —(a—c)"] + (c +a — ba? + b? +c3 +abc). Since 4abc(a +c) = 4abc(a +c — b)+4acb?, we can write the last inequality as 4acb? — (a +c)*[b? - (a —c)*] > (a+e-b)(a? +b? +c3 — Babe), which is equivalent to (a—c}*((a +c)? — b?] > [(a +c)? — b?](a? + b? +c? — ab — be ~ ca). This is true because (a +c)? — b? > 0, and (a—c)* (a? +b? +c? — ab — be —ca) =(a— b)(b—-c) 20. ‘The proof is completed. Note that equality holds if and only if a = b =c. Qa 106 Vasile Cirtoaje - Vo Quoc Ba Can - Tr ‘an Quoc Anh Problem 2.62. Let a, b, ¢ be positive real numbers such that ab + be -+ca = 1. Prove that (@? +1)? (241? (2 +1? " bec? " cha? ” Grp? = (Tran Quoc Anh) First proof. By the AM-GM Inequality, we have (a? +1)? = [a? + be +a(b +c))]? > 4a(b + c)(a? + be) = 4[ab(c? +a?) +.ca(a? + b?)] It follows that yee ay able? +a?) ca(a? +5?) b+? bP 2 +e +a pte 43 at > yi (st Z ats) 8 lab =8. This is what we want to prove. Equality holds if and only ifa = : v3 Second proof. Since a? +1=(a+b)(a+c), the inequality can be written as (a+ bP(a+ey yee a 28a. Applying the Cauchy-Schwarz Inequality, we get yetitero [Sata+bye+0)]” wae See - [Net +3abe+ Yab(a+o)]” By On the other hand, by the third degree Schur's Inequality, we have Moe? +3abe > Nav(a +b). Combining this with the above inequality, we obtain yesterer | 2[Savca+s)]” b2 + c2 = Three-variable inequalities and thus, it suffices to prove that [Savee+s)] 24(Sav) (Se*s?). Without loss of generality, assume that b is between a and c. By the AM-GM Inequality, we have 4(Sjab) (Sab?) s | bab + ue So it suffices to show that 5 ab? + b2c? +:c2q? ab(a+b) + be(b +c)+ca(c +a) > b(ab + be+ca) + —{ This inequality is equivalent to ; ate? b(a? +c?) + b*(a +e) +acla+c)> ba +c) +abe + bla? +c?) + ? or acta=blb-e) . 4 b The last inequality is obviously true, and our proof is completed. Problem 2.63. Let a, b, c be real numbers such that a+b +c = 3. Prove that a@’—be b?—ca_ c?-ab + +> 20. @+3 0 b+3 6 0243 (Wasile Cirtoaje) Proof. By the known inequality (a+b+c? > 3(ab+be+ca), we get ab + be +ca $3. Then, (a a +3 “2 (@ wate) (b-a)(b +e) “as a+3 +h bP +3 atc bt+e =de-o(S5 +3 “Faa) = (a— by? =@-ab-be~ ca) ) aaa This completes the proof. Equality occurs if and only if a Vasile Cirtoaje - Vo Quoc Ba Can - Tran Quoc Anh Problem 2.64. Let a, b, ¢ be positive real numbers such that a+b +c =3. Prove that ab be ca 3 pT, oe 9-2 "9-a 9-B 8 (Vo Quoc Ba Can) Proof. Applying the AM-GM Inequality, we have ab ab ee (a+bYa+b+20) ~ 20+) /(atreKb +o) ab are 2Va+b- (a+ eek os (ab) pee 2vavab- a+ b\b+cleta) 2V2at+b\b+ and thus, it suffices to prove that (ab)¥* + (bc)°/4 + (cay/4 < Verne +cc+a), or equivalently, [(ab)¥* + (be)9/4 + (caj3/4]2 < za + b\(b+c)(c +a). Using now the Cauchy-Schwarz Inequality and the AM-GM Inequality, we get Uady°"4 + (be)** + (ca)? < (ab + be-+ca) ( Jab + V/be + ve) +b bte +a S(ab+be +a) (4 — ) atts =(ab+be+ca)a+b+c) = (a+ b\(b+e)(e+a)+abe (a+ bib +e)c +a) GO ea 5 = plat bb +e)(c+a) This completes our proof. Note t 1 hat equality holds if and only if a = b Three-variable inequalities 109 Problem 2.65. Prove that, for any positive real numbers a, b, c, the following inequality holds [ oa b2 a oe <3 (trereFt @+atbye \at(b+e ~ VE (Vo Quoc Ba Can) Proof. Due to homogeneity, we may assume that a+b+¢ = 1, Then, the inequality becomes = -2a+1 a V2b?-2b+1 Notice now that 3(3x +4) v2x?-ax+1 SVB for any positive real number x. Indeed, this inequality is equivalent to 9(3x + 4)*(2x? — 2x +1) > 125, which is true because 9(8x + 4)°(2x? — 2x + 1) — 125 = (18x? +.42x +19) — 3x)? >0, Using this result, we have a b © —— * ss —2b+1 vse 2-241 y2a?-2a+1 “ia [a(3b +4) + b(3c +4) +c(3a+4)] = azz lBlad + be +ca)+4] Since 3(ab + be +ca) < (a+ +c)? = 1, the conclusion follows. Equality holds if and only ifa=b =c. a Problem 2.66. Let a, b, c be nonnegative real numbers such that a > 0,b+e>0 and a? +b? +c? =1. Prove that # b?—be+e (Vo Quoc Ba Can) 727077 110 Vasile Cirtoaje - Vo Quoc Ba Can - ‘Tran Quoc Anh Proof. Applying the Cauchy-Schwarz Inequality and the AM-GM Inequality, we 3 @?+b2+2)P st+5 2 a? ~ a(b? — be +02) +.02b + ac 1 * @[b?— be +e? +alb+o)] 1 2 a? +(b+c) a [ —be+c?+ a 2 — 2 © a(a?+3b?43c2) a(3 — 2a? 4 ) ¥ 4a? - (3 — 2a?) - (3 — 2a?) 4 Pes (woe | 3 as desired. Equality holds if and only if a = and b=0. 1 = — andc=0, ora= v2 oal= Problem 2.67. Let a, b, c be positive real numbers. Prove that (=) 2b \% 2c bs bte (5) +() “~ (Michael Rozenberg, Tran Quoc Anh) Proof. Due to homogeneity, we may assume that a + b +c = 3. Then, by the AM-GM Inequality, we have (=) = 2a ss, 10a dtc} ~ Vfatb+o]?-(b +c) 22 2a(b+e)+(b He) +22 Thus, it suffices to prove that a 10>) =—_—____>3, Dan aaebtat -_ Three-variable inequalities 111 Using now the Cauchy-Schwarz Inequality, we get (hey a aR +c)tb+ce+4 Sarath +e)+b+e+4] 9 “2 [De@+o+)iab+6]’ and it is enough to check that orb +0)+ Nab <9. After homogenizing, this inequality becomes 330% +0)+ (Yad) (Na) < (Na), which is equivalent to the third degree Schur’s Inequality Dia? +3abe > Yi ab(a +b). Equality holds if and only ifa = b =c. a Remark, Generally, if a, b,c are nonnegative real numbers and if r > ry = In3 > — 18 0.585..., then In2 2a)’ ( 2b)" x Voy b+e cta) ‘\a+e) *=* Problem 2.68. Let a, b, c be nonnegative numbers such that ab + be + ca = 3. Prove that 2 42 2 Prbtc Meeta Gath (Vasile Cirtoaje) Proof. By the Gauchy-Schwarz Inequality, we have y a? eve y eve YiaP+2) aby"? @+b+e™ Yaa? +b+c) Mere - Therefore, we still have to show that Neary? =3. 2 Vasile Cirtoaje - Vo Quoc Ba Can - ‘Tran Quoc Anh This follows immediately by Hélder’s Inequality (141+ 1)[(ab)?? + (be)9!? + (ca)?") [(ab)¥? + (be)? + (ca)/2] > 2 (ab+be +ca)? The proof is completed. Equality occurs if and only if a= _b =¢ =1 Oo Problem 2.69. Let a, b, c be nonnegative numbers. Prove that W +H +c > (a+b+ cab + be +cala +b +c) (Vasile Cirtoaje) Proof. Letx=a+b+c and y =ab+ be +ca. By the well-known inequality (ab + be + ca)? > 3abe(a +b +c), it follows that y? > 3abcx. Then, we have (a+b +c)(a5 +b +c3) = x(x3 - 3xy + 3abc) < x43 Thus, it suffices to show that (x? -2y)9 > y(x4 —3x2y + 2), This inequality is equivalent to (?~3y P= y) 20, which is clearly true, Equality holds if and only ifa = b ~ ¢. iB Problem 2.70. Let a, b, ¢ be positive real numbers. Prove that 3 ae) (Tran Quoc Anh) (a6 +58 4 05y4 P+ Re +oa Three-variable inequalities 113 From this inequality, we get Therefore, we still have to prove that (a® + b° +05)? > 3(a°b5 + bc + 0°05), Setting x =a°, y = b° and z=’, it becomes (2 +y? +2°P > 3(3y ty%z 27x), which is true according to Cirtoaje’s Inequality (see problem 2.52). Equality holds if and only ifa=b =c. a 11,1 Problem 2.71. if a, b, c are positive real numbers such that a+b+c = ate +=, c then 1 1 1 (1 abe (ar +o" +e" Pease =)z0 a" bt c® ‘for any positive integer n Wasile Cirtoaje) Proof. Let us denote 3 1 1 E,=a"+b"$e"-—- =, Pe It suffices to show that E, <0 for abe > 1, since the case abe = 1 is trivial, 1 1 and the case abc <1 reduces to the first by replacing a, b, ¢ with =, >, — a’ bc b respectively. Let x = be,y =ca and z = ab, xyz > 1. We write the hypothesis E, =Oas (- Dy -D@-1)=xyz-1, and the desired inequality E, <0 as (x" = 1(y" — 1)@" - 1) =x"y"2"- 1. Since xyz > 1, we may write this inequality as (Si i 0 Aha ie eC ee Od D xP Lyt Agr y gr 2yn-2gnd gd, which is clearly true. For n > 2, equality holds if and only if a = b = 114 Vasile Cirtoaje - Vo Quoc Ba Can ~ Tran Quoc Anh Problem 2.72. If a, b,c are positive real numbers such that abc = 1, then 5 s 5 @ b c @ 1+a4 ieee Te: a' BI ¢ @) 1 tas t Tae * T4255 * T4208 > (Wasile Girtoaje) 5 4 _ a a(itat)— a Predf, @ since T= STO e OL ay write thedte 2f Teat pygt 8 pce: we may write the in equality as 3, a b c atb+c2 > +7 —+——_ 4 20 a4+1 0 bt41 ct] Since a* +1 > a(a? +1), it suffices to show that 3 atbtoe st a+ . “ x 2 Using the substitution a =~, b =~ and c= =, where x, y, z are positive real x numbers, the inequality becomes tit sts Yor x xt+y2" y2 ez’ By the Cauchy-Schwarz Inequality, we have 2 2h yey ge ZY, 2, Gtytet 4 ety te : You x xytys+ex xy +yz+ex and 2 x (xtytz? sty? (x? +2) +? +22) + @? + x2) (xtyt2) 1. xy+yzt+ex © 26847423) aye Thus, it suffices to prove that xtty?te? xy tyztex xyt+ystex x2+y2 422 = which is obvious. Equality holds if and only if a= b =¢ =1. Three-variable inequalities 115 2 ap? fe (b) Using the substitutions a = {/ —, b = {/ = and c = {/—, the inequality yz 2X xy becomes 4 4 at x y : + + 21 yet +2x5 yxye | 2x*+2y3 yxye x*y? +223 yxye By the Cauchy-Schwarz Inequality, we have Ta (2) (2) > = : ya? + 2x8 yaye ~ S\y2e? + 2x9 veya) Dly*s? +2yRVE DIX? Therefore, we need to show that (ney > Diy? +297 Ds. Since x+y +2 > 3 /XY%, it suffices to prove that (Se) ese +2(Ds) (E>). Mixt4+3 077222) xy(x? + y7). This inequality is equivalent to or DVe-y*20. Equality holds if and only if a= Problem 2.73. Ifa, b, c are nonnegative real numbers, then a) +4abe 3 +4abe c+ 4abe $ @+(b+cP+6abe bi +(ct+a)?+6abe c3+(a+b)?+6abe~ ” (Wasile Cirtoaje) Proof. If one of a, b, ¢ is zero, the inequality turns into equality. Next, consider that a, b, c are positive numbers and write the inequality as 1 a 4abc ¥ 1 =——_—__ 31 a,” “ pcre ren or re DiRT ED eae 116 Vasile Cirtoaje - Vo Quoc Ba Can - Tran Quoe Anh Since -EgEeEr are L va a? +(b+c)> + 6abc B+ b3 + a3 +(b +c)? + Gabe ___3abe z 2a5 +a?(b +c) OF +3 Oa +(b +c) + babe’ the inequality becomes as follows se +0)—~ 20° ~ 30%(b +c) @+(b+cP+6abe ~ x (b= a)(2b +a)? + (c—a(2c+ ays a +(b +c)? + 6abe y (c= b)(2c +b)? y (b=o)2b+eP 03 +(c+a)>+6abe +(a+b)>+6abe — Yo-0) [> ____ er by #0 S+at+bP+6abe b2+(c+a)> +6abe | ~ zr (b~c)*(b + ca? - Ba? - Ca + D) [c+ (a+b)? + 6abc][b3 + (c+ a)? + Gabe] be 2 where B= b-+¢-=——, C= (b~c}? and D = b3 +3. Since —“— +(b+e)2 2a>a,wehave ** bre 2b Ba? =a2(b+c)- 2% < 42 - -b-¢ c) bye ST Cb +e) bela — b—c). Using this result and Schur’s Inequality of third degree, we obtain 3 _ pa? a" — Ba" ~Ca+D2>a*~a"(b+c)+be(a—b—c)—Ca+D Ya +3abe - Nav(a +b) > 0. Equality holds for a = ¢, and also for a = 0, or b= 0, orc =0. a Problem 2.74. Ifa, b, ¢ are positive real numbers such that abc = 1, then 12a+ (Vasile Cirtoaje) Three-variable inequalities 117 Proof. Write the inequality as 12a+7 12b+7 12¢+7 9- +[9- +(9- 28 ( sat} (2 ea) ( eee (3a -1)? (3b=1" | Ge- 1? S 2a?+1 9 2b7+1 ° 2c?+1 ~ By the Cauchy-Schwarz Inequality, we have (30-1)? , (8Xe-3) Ma- 3)" _ 9a +18) ab-18 7a +9 Last? Saray aes ‘Thus, it suffices to show that f(a) + f(b) +F(c) 23, or where f(x) =x? +18 (= -x). Assume that a = max{a, b, c}, clearly a > 1 and be < 1. We will show that F(0)+ FE) 2 2F (vee), and s(a)+2F (voc) =3, which yield the desired inequality. The first inequality is true because s(0)+F(6)-2f (Vie) = cP +18 (Yo- ve) (4-2) 0 To prove the second inequality, we write it as f(x?) +2f (3) 2 z where x = /@. After some calculations, this inequality becomes 6 x°—18x5 + 36x4 — 3x? — 36x +202 0, or (x - 1)*(x — 2)"(x? + 6x +5) > 0, which is clearly true. Equality holds for (a, b, ¢) =(1, 1, 1), and for (a, b,c) = 11 (+ ie 3) or any cyclic permutation. o 118 Vasile Cirtoaje - Vo Quoc Ba Can - Tran Quoc Anh Problem 2.75. Let a, b, c be positive real numbers such that abe = 1. Prove that wi tN APT 4 (+a? ” (+672 G+e? *GsatbacR 2! (Vasile Cirtoaje) Proof. Let p=a+b+candq=ab + be +ca. We have (+a) +b) +e)=p+q+2, and oy 1 e+ a+ 6? aa (p+q+2) : [Lo +aa+0)]*-2+0+2 0a +a) (p+q+2? _ @p+4+3)?~20p +4 +2)(p +3) _ 49+8~(p-1? (p+q+27 (p+q+2) Thus, we may write the inequality as 4 49+8~(p-1)? (P+1P ~ (p+q42p This inequality reduces to (p?-24-3)?>0, of equivalently, (@? +b? +023)? >, Equality occurs if and only if a= b =¢ =1, Q Problem 2.76. Let a, b, ¢ be nonnegative numbers such that a? + b? +c? = 3, Prove that 1+4abe > Sminfa, b, c} (Vasile Cirtoaje) Proof. Without loss of generality, assume that a < b Sa. ‘Three-variable inequalities 119 From a?+b?+-c? = 3 anda < b 0, we obtain be >aVb? +0? — a? =aV'3— 202, Therefore, it suffices to show that a? V3 — 2a? > Sa To prove this inequality, we consider two cases. 2 S Case 1. 0Sa <5. Since V3-2a?> 37 itis enough to prove that 20 5 Ze -Sa+120, which is equivalent to the obvious inequality 5 2 5 sa-1} +0720. (2 ) i2 1 . Case 2. 5 0. ‘Therefore, it suffices to prove that 4a°(2-a)>5a-1, which is true because 4a°(2-a)—5a+1=(1-a)(2a—1)*>0. Equality occurs if and only if a= Problem 2.77. Let a, b,c be nonnegative real numbers, no two of which are zero. Prove that 1 2 alee * are (Vasile Cirtoaje) 120 Vasile Cirtoaje - Vo Quoc Ba Can - Tran Quoc Anh Proof: Assume that ¢ = min{a, b, c}. Applying the Cauchy-Schwarz Inequality we have } 1 1 4 = tS > 2a? + be * 2b?+ca ~ 2a? +262 tact be’ Thus, it suffices to prove that 4 1 1 2 St Te 2a? +26? +ac+be | 2c? +ab ~ ab+betca | abla ‘This inequality is equivalent to 14 2 2? +ab ab+be+ca~ a+b +e? 2a24 abo pact be’ cla+b 2c) S 2c(a + b ~ 2c) (ab + 2c?)(ab + be + ca) ~ (a? +b? +e2\(2a? + Ob? bac + be)" Since c(a + b — 2c) > 0, itis enough to show that (a? +b? +c?)(2a? +26? + ac + be) > 2(ab + 2c*)(ab + be +a). But this is obviously true because a? + b? +c? > ab + be +ca and 2a° + 2b? +ac+ be > 2ab +22), Equality holds if and only ifa = b = ¢, ora = b,c = 0, orany cyclic permutation. a Problem 2.78. Let a, b, c be positive real numbers such that {42 4.£ = 5 Prove that A 8 7b ca cee He a sqtptysitav2. (Vasile Cirtoaje) a b c Proof. Setting x=, y = — and s =<, we need to show that a 1 += S144V' z for xyz = land x+y+z=5. Dueto symmetry, assume that x < y 1, _ Three-variable inequalities 121 The left inequality is true since (2x -1)°(4—x) ae 1$17~1 +z 17 o ep 1 =+x(5—x)— % wl xy 4 x. ys 4 and g = 4. Therefore, equality in the original left Rie Equality holds for x = y = bc inequality holds for a= 5 = 3 or any cyclic permutation. The right inequality is also true since 1,1,1 ty 1 1 Sfp Spe 1-4/9 Sy aE asiSg) 4 = xy 2 xy z _ G1 v9) (B-2V8-2) ; = 1+ V2. Therefore, equality in b ¢ 34+2V2 14+Vv2 Equality holds for x = 3—2V2 and y the original right inequality holds for a = permutation. Remark. The following more general statement holds: Let a, b, c and ¢ be positive real numbers such that ae B +2 aaa boca. ee (a) IfO 1, then bie 2 cece lela? and (@tb+c) abc t b 7 =e orany cyclic permutation; bioic with equality for a= =“ or any cyclic permutation In the cases (a) and (b), we have bi c.a 2 1 =-2-P=(~-1?(—-1)> atote fo e-8 (a 1) 20, 122 Vasile Cirtoaje - Vo Quoc Ba Can - Tran Quoc Anh and respectively. Problem 2.79. Let a, b, c be positive real numbers. Prove that 3a(b +c) —2be 3b(c +a) — 2ca 3cla+b)—2ab _ 3 (2a+b+c)(b+c) " (2b+ce+ale+a Ei (2c-+a+ bat b) — (Vasile Cirtoaje) b+c c+a Proof. det = SS, y= S8 and = SS" We have Ba(b +e)—2be _ Gx(y +a~x)- 2x +y—s\ixt2-—y) Qa+b+celb+c) 4x(y +2) = 3x +2)4+(y~2)?— 4x? (y—a)?- 4x? 3 2x(y +2) “"Fxytay) 12 Therefore, the original inequality is equivalent to -s? Dagese4 Lys Since x ae eae 2D yte yrs “et x x-y¥ x-2 = Davart yas oS Ca GWG F2) it can be written as agit 2 do- ay- warn |? which is equivalent to Y & =2)Pr- ye -2) X@+)O4+2e+x) or (SS EE2) 5 0 (e+ y)O +2) +x) x Three-variable inequalities 123 On the other hand, it is easy to verify that -y¥ _@-wWy-2)@-*) Ue Therefore, the last inequality is equivalent to (e-y"-2%e-x) xya(x+yy t2)e+x) which is true. Equality holds for a = b, or b=c, ore =a. o Remark. Actually, the following identity holds x Ba(b+e)—2be 3 _ Qatbtc\b+c) 2 _ (a—b)*(b-c)*(c-a)? ~ 2a + bib + cc +a)(2a +b +c\(2b+c+a(2c+a+b) Problem 2.80. Let a, b, c be nonnegative real numbers. Prove that a” —be b?-ca atte B3atbt+e 3b+ce+a 3cet+atb™ (Wasile Cirtoaje) Proof. Since P=be _ wr (a+ca-b)+(at+ba-c) 205 ne 3at+b+e (b+a)(b—c) yet» “3b+e+a 3c+a+b -¥ (b-*(b+e-a) (3b+c+a)(3c+a+b)’ ‘we can write the inequality as (b-c)*S, + (c —a)*S, + (a— b)*S, > 0, where Sq =(b +c —a)(3a+b +c) =—3a? + (b+ cla + b? +c? + 2bc =-2(a— b)la-c)—a? +b? +07 +4be. 124 Vasile Cirtoaje - Vo Quoc Ba Can - Tran Quoc Anh Since }'(b ~c)°(a— bla -c) =0, the inequality is equivalent to (b-c)?Rq + (c -a)°R, + (a— BYR, > 0, where R, = —a? +b? +-c?-+4be. Assume now that a 2 b2c. Since R, > O and R. 2 0, we get No-crr, (b~ cPPRq + (a—c)?Ry > (b~c)(R, + Ry) 2c(2a + 2b+c)(b—c)? >0, Equality holds for a = b= c, and again for a=0 and b=c, b= andc =a, ¢=Oanda=b. ag Problem 2.81. If a, b, c are positive real numbers such that Vabe=r <1, then 1 1 1 3 Se ye a l+a+b “1+b+c° T+ce+a~ +27 Proof Since + ;__@+b_ the original inequality can be writte as l+a+b l+a+b’ ‘quality can be written atb bte cta a+b+1" b+c+1 chai or a+b bt+c cta a+b+1° b+ce4+1 c+at] Setting p = a+b +c, by the Cauchy-Schwarz Inequality and the AMLGM In- equality, we have by ato (Livers) 2420 Varner pa =e oe a+b+1~ Sat b41) 2p+3 - 2p+3 2p+3 ~ 2p+3 Therefore, it suffices to prove that 2p+3r 3r 2p+3 ~2r+1 This is equivalent to __3r_ 2p +3r ar+1~ 2p+3” Three-variable inequalities 125 or 3a-r) 2 > ar+1~ 2p+3 which is true because 1 —r > 0 and 2p +3>6r+3=3(2r +1). oO The proof is completed. Equality holds if and only ifa = b =c. Problem 2.82. Ifa, b, ¢ are distinct real numbers, then @ (s5)'+(a) + (Gye a)? a \? cy, 1, Slab+ be +ca) © (62) +(4) +) aaa (Tran Quoc Anh, Vasile Cirtoaje) Proof. (a) Since a ob Lia a we have Le a b a 2 Spt Lge sa (Lye) 2° ue (b) Let x = a? + b? +c? and y = ab + be +ca. Applying the Cauchy-Schwarz Inequality, we have 2 5 @tbteP _ xt2y @ Lane “Sone 2—y Thus, it is enough to show that xt2y 11, sao t12-- 2(x-y) x This inequality is equivalent to (x — 2y)* > 0, which is clearly true. a 126 Vasile Cirtoaje - Vo Quoc Ba Can - Tran Quoc Anh Problem 2.83. Let a, b, c be positive real numbers such that abe = 1. Prove that ab be a 3 a4 oy Dab Be Arabi t Dra paigt Sq (Tran Quoc Ank) Proof. Applying the AM-GM Inequality, we get ab ab abc? +b? +2044 2a°b? ae B%2 4 DaPp? 24 52 a*b eB abc? abe? = a Ss ee a°(b? +c) + Ba? +62) ~ > [Pbo*(a2 + Vb? +2) 2 1 Se ee 2V(@? +c7)(b2 402) 4 and similarly, be clf_@ b+c2 +2684 ~ 4 | G2apet z ca <1(_? +ar+aciat ~4\ ppt Adding up these three inequalities, we get y ab cl(@rb? Bee? erat) 3 +b +20 al ee Rea t apg) ay which is just the desired inequality. Equality holds if and only ifa=b = 1. | Problem 2.84. Ifa, b,c €[0, 1], then a(1— b*) + b(1 ~c?) +.e(1 - a2) < : (Ji Chen) | Proof. Let a=1-x,b=1—y andc=1~—z, where 0 abc(a? +b? + e). (Tran Quoc Anh) Proof. For any positive real numbers x and y, we have 2? —xy ty? (xt +y") =(x-y)*> 0. [tty xwaxyty?> >. Using this inequality, we get eb (a? — ab +b°)(b? ~ be + 2c? — ca +02) > yf OF H+ Act + a) = Xe a) and hence, it suffices to prove that It follows that 8 (a4 + b4)(b4 +c4)(c4 +a4) > gr brea? +53 +03)2, i aenmmennneees meme —tS 128 Vasile Cirtoaje - Vo Quoe Ba Can - ‘Tran Quoe Anh By applying the known inequalities 8 (x+y) +2)(e+x)> gh ty +2) + yz+zx) and x?y? + ye? 4 22x? > xya(x ty +2), we obtain (a* + bAY(bt +e4(c# +04) > stat + bt +c4)(atb* + b%c4 + c4a4y > perbtertat +b +ct)(a? +b? +c%), Thus, it is enough to prove that (at + bt +c8\(a? +b? +02) > (a3 +53 +.03)2, which is true according to the Cauchy-Schwarz Inequality. Equality holds if and only ifa=b=c. o ee 2.86, Let a, b, c be distinct real numbers such that a+ b +c = 3. Prove that P+ de? b+ ca? cP +025? ee os, (b= cF "(ea * (aby (Tran Quoc Anh) Proof. Since 2(a + bc?) = (a + bc)? + (a — be)?, the desired inequality is equivalent to a+be\? x( boc ) + Now, we notice that Spats btca be ¢ ~ -a and yuaek b=ca zi ye-4 b= tag =a btet jJ=- Since pate, bee abe b-ca boc @ Sy : =ae I-f = Three-variable inequalities 129 we can write the above inequality as a+bc\? a-bc\? at+be b+ca Do (SE) +x) ease. b—e b-e boc cma which is equivalent to atbe\? a—be)* a+be b+ca oe) & . x( ve) +(= i) 23 b-c ca It suffices to show that y a+be ae a+be b+ca a i re which is true according to the well-known inequality x? + y? +2? >xy+yz+ 2x. a Problem 2.87. Let a, b, c be nonnegative real numbers such that a <1 abe+2; (b) Ifab+ bc +ca =3, then a2b+b2e+c2a>3. (Vasile Ctrtoaje) Proof. Since a < b < c, we have a(b — a)(b —c) <0 and thus, it follows that ab +b*c+c%a > ab +b +c7a +a(b—a)(b—c) = b(a+c)+acla+c—b). We will use this inequality to prove (a) and (b). (a) It suffices to prove that B'(a+c)+acla+c—b)> abe+2. This inequality is equivalent to b’(a+¢)-2> ac(2b-a—c), 6°(3 — b)—2> ac(3b—3). From the inequality (b — a)(b —c) < 0, we deduce that ac < b(a+¢—b) = b(3 — 2b). Therefore, since b > 1, we have ac(3b — 3) $ b(3 — 2b)(3b — 3). 130 Vasile Cirtoaje - Vo Quoc Ba Can - Tran Quoc Anh Thus, it suffices to show that b°(3 ~ b)—2> b(3- 2b)(3b-3), which is equivalent to (Sb-2)(b-1)?>0, The last one is obviously true, so our proof is completed. Equality holds if and only ifa = b=c=1ora=0,b=1andc=2 (b) It suffices to prove that b(a+c)+acla+c—b)>3. =a +c, From the given conditions, we have 1 < x < 3. Now, replacing — bx into the above inequality, we can write it as ac= b?x +:(3 — bx)(x — b) > 3. This is equivalent to 2b?x — (x? +3)b+3x-3>0, We have 2b?x — (x? + 3)b-+3x—3 = 2(67— 2b + 1)x +2(2b— x — (x? +3)b 43-3 = 2(b-1)’x + (3-x)(bx —b-1) 2(3-x)(bx-b-1), and it is enough to prove that bx-b-120. From the inequality (b — a)(b — c) < 0, we get bx > b? +ac = b? +3 — bx, +3 bx = ——, and hence bx-—b-1> b?+3 (b-1)* b > 2 gee ‘The proof is completed. Equality holds if and only if a = b=landc=3, Three-variable inequalities 131 Problem 2.88. Let a, b, c and k be positive real numbers. Prove that ab cy kate kb+a ke+b otc ta kote ke+a! katb’ (Adapted after a Mathlinks Contest problem) First proof. Let ka kb ke 1+— 1+— 14+— ss b ‘a ¢ a =i) ys 1+k’ 1+k’ According to Hélder’s Inequality, we get (8) (+8) (8) = (ef EBZ) -co0n and thus, it follows that XYZ > 1. Now, we rewrite the desired inequality into the following form a kate b kb+a) | c_Kth) 5 B kb+e) ‘lc ke+a a katb)~ Itis equivalent to cla-b) , alb—c) , Blea) b(kb+c) * eke Fa) * alka by =” or a m2 1+ Since a_, (k+0x Ce Waa ve (k+ DY Yk the last inequality can be written as X-1 Y- or equivalently, 132 Vasile Cirtoaje - Vo Quoc Ba Can - Tran Quoc Anh By the AM-GM Inequality, we have Lele FF) LVR eT tos, so the above inequality is true. The proof is completed. Note that equality holds if and only ifa=b =c. o Second proof. Similar to the above proof, we need to prove that cab) | a(b-c) b(e~a) | b(kb +0) * clkeFa) * a(katb) =" Due to cyclicity, we may assume without loss of generality that b is between a and ¢, ie. (b ~ a)(b~ c) $0. Since b(c ~ a) = ~c(a — b)— a(b —c), we can write the above inequality as 1 i 1 1 a a ec Ligeti ot 7. ela ees arm] ta ‘laa warn] 2% or clk(a— b)%(a + b) + ba ~ bYfa =O] , [kO= Maca te) aloe) ab(ka+b)(kb +c) c(ke +a)(ka +b) Be Since (a— bla ~c)=(a~b)*-(b-a)(b-c)>0, and (b= ca-c)=(b-c)~(b-a)(b—c)>0, we can see that the last inequality is obviously true. a Problem 2.89. Let a, b, c be distinct real numbers such that a+ b-+¢ =0. Prove that the following inequality holds al 1 33 G-o * Ga te] b+be+ca? a (ab +bc oa} | 22 (Tran Quoc Anh) Proof. From the given condition, we have (a + b +c)? =0. This implies that —6(ab + be +ca) =(a— b)? +(b-c)? + (c—a)? Three-variable inequalities 133 Thus, our inequality is equivalent to 1 (Pty? +27? (= 4 pines where x =b—c, y=c-aandz=a-b. Without loss of generality, we may assume that a > b > c. Then, we have x > 0, 220, y =—x—z, and the above inequality becomes 1 1 297 # Gta] 4 [x? +2 +(x +2)? [= By the Cauchy-Schwarz Inequality and the AM-GM Inequality, we have and Therefore, Fl > (ete? a) y_32 1 |= 2 [‘e +(x +2) last 4. This completes our proof. Under the assumption a > b > c, equality holds if and only ifa+b+e=Oanda—b=b—c,thatiswhena=-candb=0. (x? +224 (x+2)?]? [= Problem 2.90. Ifa, b,c are real numbers, then 4abe + ¥ 2(a? + b*)(b? +c?)(c? +a”) > (a+ b)(b+c)(c +a). (Vo Quoc Ba Can) Proof. Since 2(a? + b*) = (a+b) +(a—b)? and : (b? +c?)(c? +a?) = (c? + ab)? +c%(a — by’, the Cauchy-Schwarz Inequality gives V2(a? + b?)(b? +c)(c? +a) > (a+ bc? +.ab) +c(a — by. 134 Vasile Cirtoaje - Vo Quoc Ba Can - Tran Quoc Anh ‘Then, it follows that 4abe + Wel EMDR Eee at ) 2 dabe + (a+ b)(c? +. ab) + c(a — b)2 =(a+b)(b+c)(c+a). Equality holds if and only ifa=b>0, orb=c>0, orc=a>0. Qa Problem 2.91. If a, b,c are real numbers, then (a? +ab+b?)(b? +be+c)(c2+ca+a2) > 3(a7b+ b*c-+.c7a)(ab? + be? +ca?), (Gabriel Dospinescu) Proof. Firstly, we note that A(a? + ac +07)(b? + be +02) = (2ab +.ae4 be+ 2c??? + 3¢2(a — b)? and A(a? +.ab +b?) = 3(a+ b)? + (a — by? Therefore, by applying the Cauchy-Schwarz Inequality, we get 16(a? +ab + b)(b? + be + c2\(c?+.ca +02) > 2 [V3la+ b) 20d -+ac+ be +202) + VIela—b)"]? = 12[(@?b + b7c + c2a) + (ab? + be2 +ca*)]}*. It suffices to prove that [(a?b + bc +¢7a)-+ (ab? + be? +ca2)]? > 4(a?b + b?c +c2a)(ab? + be? +ca?), which is true according to the AM-GM Inequality, Equality holds if and only if a=b,orb=c,ore=a. a Problem 2.92. If a, b,c are nonnegative real numbers, then be 3b? + 3a? 2a? + b? +c? a, _3~ab 3 bec? + a2 © 2c? +a +b? = 2 Wasile Cirtoaje) Three-variable inequalities _ 135 First proof. Write the inequality as 2 xy i _ 3a’ be >0, 2 2a2+b? +c? 2(b? +c? — 2a?) ~(b ~ x 2a? +b? +c? or, equivalently, Since D+? 20? ese 8 Lowe 2a? +b? +c? ba? , ab? - Lepee Bb? te +L +c? +a 2 522 > lene Se 7 (eye - LaF +c? +.a)(2c? + a2 +52)’ the inequality becomes No -oFs, 20, where Sq =2(b-+c)*(2a? + b? +07) — (a? +26? + ¢2)(a2 + b? + 2c? Since Sq = 4bc(2a? + b? +c”) + 2(b? +.c2)(2a? + b? + c2) — (a2 + 262 + c7)(a? + b? + 2c?) = Abc(2a? + b? + ¢?) - (a? — b?)(a? —c?), we can write the desired inequality as follows 4) | be(2a?+ b?+02)(b— 6)? > Sa? — bya? —<2\(b — ec}. Without loss of generality, assume that ¢ < b (b—a)*(c—a)*. Indeed, 4bc(20 +b? +c7)—(b—a)*(c—a)? > 4be(b* +.c2) — bc? > Bb2e2 — pc? >0. Equality holds for a a=b, and again for a c, fora = 0 and ¢,b=Oande =a,c=Oand 0. a Second proof. Assume that a > b > c. As shown in the first solution, the desired inequality is equivalent to) \(b —c)*S, > 0, where = Abc(2a? +b? +c?) — (a? — b2)(a? — 2). Since Sq 2 ~(a? — b?)(a? = c?) = —a?(a? — b2) + c2(a? — b?) > -a(a? — b?), Sp 2 Aca(2b? +c? + a?) ~ (b? ~ c?)(b? — a2) > ae — (b2 — <2)(b? — a) =aPc(a~c)-+ bc? + b%(a? — b2) > b2(a2 — b), Se = 4ab(2c? + a? +b?) — (c? — a)(c? — b2) > gab(2c? +02 + b2) — ab? 1 2 > zabla® +b?) -a*b? = zevta bY >0, we have DO - )75, 2 (b- cPS, + (a e)S, > (0? — b2)f-a2(b — 6)? + b*(a— 0] = (a? — b*)(a — b)(2ab — ac — be) > 0. Third proof. We rewrite the inequality as zr 3 3a? — be 2 2a? +b +c This is equivalent to ys 3(b—c)? +8be “2a +b ER = % or (b-c? be =e eye pr aa )* Disa ae 22 Three-variable inequalities - 137 By the Cauchy-Schwarz Inequality, we have (b-<) be x 2a? +b? +c? +8 soa $b re) = [Mo-or 44S be]? > ~ Sb = 020? + b-+07)+6 9 | be(2a? + b? +02) _ 4(Sa)" Di? +c? + 4bc)(20? +b? +e)" On the other hand, SiG? +2 + 4be)(20 +b? +c?) = = (i) (er+e +4be)] + arb? +e +4bc) =2()a?) (Sia) +2(dae)? =2[(Ue) (Let+2 de) + (dee)"| =2(Sya?+2 ab) =2(S72)' Combining this with the above inequality, the conclusion follows immediately. Qo Problem 2.93. Ifa, b,c are the side-lengths of a triangle, then (a+b)*+(b+c)*+(c+a)* > 9(a4+b4 +4), (Vasile Cirtoaje) Proof. Assume that a > b > c, From the inequality b +c > a, we get a > b> ¢2a~b > 0. According to this result, we have (at + b4) + 4c(a? +b) + 6c2(a? + b*) +.4c3(a +b) +.2c4 > (a* + b*) + 4c(a? + BS) + 6c2(a? + b2) +94 = (a4 + b4) + 4(a — b)(a +.B) + 6(a — b)*(a? +b), (a+c)*+(b+c)* ‘Therefore, it suffices to prove that A(a — b)(a? + b3) + 6(a— b)?(a? +b?) + (a +-b)* > B(a* + b4), 138 Vasile Cirtoaje - Vo Quoc Ba Can This inequality is equivalent to (a — b)(3a° - 90?b + 9ab?-+5b3) > 0, which is true because 3_ op2 nen 2. 3\7 3, 3a* — 9a*b + 9ab? = 3a(a? —3ab +3?) = 3a a-5b} +707! >0. The proof is completed. Equality holds for a degenerate triangle having a side equal to zero. o Problem 2.94. Let a, b, c be nonnegative real numbers such thata+b+c=6. Prove that a? + 2b? 43c? + abe > 24, (Tran Quoc Luat) Proof. We will consider three cases Case 1. a > 4, In this case, we have 2b? + 3c? + abe > 2b? +2c? + 4be = 2b +c). Therefore, it suffices to prove that a +2(b+c)? > 24, Bur this inequality follows immediately from the Cauchy-Schwarz Inequality (a+b+c? Grrr Ly l+s 2 a4 Ab +c? > Case 2. b > 2. Since +3? tabe> a? +c? + 20c = (a+), itis enough to check the following inequality (+c)? +2b? > 24 By the Cauchy-Schwarz Inequality, we have 2 (atc) +20? > eter bP 94 ist *3 Three-variable inequalities 139 so the above inequality is true. Case 3. a < 4 and b < 2. In this case, we have (4— a)(2—b) > 0, ie. ab > 2a + 4b — 8. According to this result, we find that a? + 2b? +3c? +abc > a? + 2b? + 3c? + c(2a + 4b — 8) =(a +c)? +.2(b +c)? - 8c. On the other hand, the Cauchy-Schwarz Inequality gives us atc+b+ceP 2 (arc P+20b+ey 2 SAEFETN 264 6 1k Combining this with the above inequality, we get 2 2 2 a? + 2b? +3? +abe > z+) -8c= a +242 24, The proof is completed. It is easy to see that equality holds if and only if a = 4, b=2andc=0. a Problem 2.95. Let a, b, c be positive real numbers such that a + b +c = 3. Prove that 8 slab + be +ca)Vab + be +ca > (a+ be)(b+ca)(c +ab). 3v3, (Tran Quoc Anh) Proof. From the given hypothesis a + b-+c =3, we can easily check that Die + be)(b + ca) = 4(ab + be + ca) (xy + yz tex)? 3B(x+y +z) 16(ab + be + ca)? | | | | | ‘Therefore, using the well-known inequality xyz < , we get (a+ be)(b +ca)(e+ab) < 3@+abtbe+cay’ so it is enough to prove that 8 16(ab + be + ca)* lab + be+ca)Vab+betea> et ete gygrh tbe tea Vad + be bea ® ee a acay’ or ab+be+ca+3>2V3(ab + be +ca). Bur thisis clearly true according to the AM-GM. The proof is completed. Equality holds if and only if a= b = =, a 140 Vasile Cirtoaje - Vo Quoc Ba Can - Tran Quoe Anh Problem 2.96. Let a, b, c be positive real numbers, Prove that (l+a+b+c)(1+ab+be+ca) > 4/2(a + be)(b + eae + ab). (Tran Quoc Anh) Proof. Note that Cta+b+e\l+ab+be+ca) =(1+a)(1 +b +e) +(a+5\(b +e)(c-+a), therefore, by applying the AM-GM Inequality, we get (t+a+b+c)(1+ab+be+ea) > 2/1 + ald + bMI +cat bb ele da). It suffices to prove that 8(a + be\(b + calc + ab) < (1+ a) +b)1 +c)(a + b)(b + cle ta). By the AM-GM Inequality, we have 2 (a+ be)(b+ca) 16. Three-variable inequalities 141 Since ai4ii —+—+—-9=(atb+c) a'bic we can write this inequality as (a=by a=eP | (b=cP bc + +48(ab + be + ca) > 16. ab ac Without loss of generality, we may assume that a > b > c. Since b > ¢ and (a-b? _ (a-c)? , by Chebyshev’s Inequality, we have 2 py op +o [5 2 ] 22/2 a ae. =| b c = 2[(a—b)* + (a-c)?J. It follows that (a-b? (a-c?? _ 2[(a— bP +(a—c)?] ab ae abte) On the other hand, c _ 2Ab-c}? be a(b+c) Therefore, we get (a—bP (ac? (bc)? _ 2f(a— by? +(a—cP +(b—c)"] “ab * ac! be a(b +c) bP +( r+(b (a+b+c? =8[(a—b)? +(a-c)?+(b~c)*] = 16(a+b +c)? — 48(ab + be +a), 2. ] » 8c 1 Problem 2.98. If a, b, ¢ are the side-lengths of a triangle, then (2b + b?c+c7a)? > abcla +b +c)(a2 + 6? +¢2), (Vo Quoc Ba Can) 142 Vasile Cirtoaje - Vo Quoc Ba Can - Tran Quoc Anh Proof. Without loss of generality, assume that a is between b and c. We need to show that the quadratic function F(x) = (0? +b? +.c)x? — 2(0?b + bc + c2a)x +abc(a +b +c) has its discriminant nonnegative. Since a? + b? +c? > 0, if the discriminant is negative, then f(x) > 0 for any real x. But F(a) =ala- bYa-ca—b+e) <0, and this shows that the discriminant is not negative. Equality in the original inequality holds for an equilateral triangle, and for a degenerate triangle with b=a+canda®=ac? +03, o Remark. Under the assumption that a is between b and c, the desired inequality is a consequence of the identity (a?b + bc +c7a)* — abc(a+b+c)(a2 +b? +c2)= = [a?(a ~ b) + b(a~c)]? — ala — ba —cNa—b + ca? +b? +02). which can be obtained from ab +b'c+c7a a+b? 4c? (ab +bc +a? a? +b? +c? 2 f(x) =(a?-+b? +c?) (-- ) tabe(atb+c)— and f(a) =ala— by(a-c)(a-b +c). Problem 2.99. If a, b, ¢ are positive real numbers satisfying abc = 1, then 3/_ @ af_& af. b+26* e+2 + a+26 =) Proof. By Bernoulli’s Inequality, we get @[b#26 _ of PT b-1_ b +80 27 27 ~ 8181 1 27 1 27, 1,7 6 +80" Ya+26~ a+80" Then 3 z x SI o z 2 3 < 3 x Three-variable inequalities 143 and it suffices to show that _# b+80 c+80 a+26 x ea Substituting 2 for Ya, 5 for YB, and ~ for YE (x, y, = > 0), the inequality becomes yt at x 3: F 3 3 + 3 3 x(80y3 +23) — y(80z3 + x3) * 2(80x3+ y3) 4 By the Cauchy-Schwarz, Inequality, we have y yh OP 4st ext x(0y* +29) ~ Sx(Boy? +23)" Therefore, it is enough to show that 27(x? ty? +22)? > BO(xy3 + yz3 + 2x9) + (x3y +y3e 29x). This inequality immediately follows from Cirtoaje’s inequalities (see problem 2.52) (P+ y? +272 > 3(xy? + yx? + 2x3), and (P+ y? +22)? > 3(x3y + y3z425x), Equality occurs if and only ifa = b =c = 1. ao Remark, Actually, the following more general statement holds: Let a, b, ¢ be positive real numbers such that abc = 1. If < m<5andk>0 then nf_@ af b af ©, 3 b+k” Ve+k” Vark= Yite Problem 2.100. Let a,b,c be nonnegative real numbers, no all are zero. Prove that 2 a b SS -_ SS + — 14+V2~ a4 Jab +2) b+ fle? +a2) SS 1 c+ 2(a? +b?) (Tran Van Luan) 144 Vasile Cirtoaje - Vo Quoc Ba Can - Tran Quoc Anh Proof. The right inequality follows immediately from the well-known inequality V(x? + y?) = x + y for any x, y > 0. Let us prove now the left inequality. By the Cauchy-Schwarz Inequality, we have ys a . (a+b+e? ; at Yabe+e) Sra fat 26? +-7)] It suffices to prove that Mes 2, ave? +07 < Br ernsor We have two cases Case 1. a? +b? +c? > (ab + be + ca). Since Dev? +e < alb +e) = 2ab + be+ca), it suffices to show that v2+1 z (atb+c). (a? +b? +c?) + 2V2(ab + be +ca) < But this is equivalent to e Aab+be+ca) 8(a +b? +c). (Michael Rozenberg) Proof. From the given hypothesis, we have 11,1 (a+b+c)|-+>+-]29. abc Now, we write the inequality as x (Fe) 295 a 60s, (Xe) (=3) 293 a6) ab. ‘This inequality is equivalent to (de? -3abe) (= z) 293 1a? - 6) ab = sabe, (Se? - 3abc) (=) 29310? -9) ab. (Se'-sebe) (EZ) = (Ue-Le4) (Le) (Zi) it can be written as (X#-Les) (He) (Zz)-9] =o which is obviously true because a? +b? +c? > ab + be +ca and or Since yoiji tatbeg(S+542)29 a be +b b+ The proof is completed. Equality holds if and only if 77 +2 ~£ 4 a = ¢ a 146 Vasile Cirtoaje - Vo Quoc Ba Gan - ‘Tran Quoc Anh Problem 2.102. Let a, b, c be nonnegative real numbers such thata+b+c=3. Prove that a b iG 1 eee ere. 5 B8+16 c3+16 a? +16 (Tran Quoc Anh) Proof. The inequality is equivalent to ( a a ) + b b 4s 16 B+16)” (16 F416 (i ab? rs be3 ki ca? 1 eee pag Oe b3+16 c3+16° a? +16 By the AM-GM Inequality, we have or B+8+8 b°+16 <——= : b = 12 12 ‘Then ab? ab?-b _ ab? 416 b +16 ~ 12 Adding this and its analogous inequalities, we get abs be3 : ca® ab? + be? +.ca” +16 416 +16 ~ 12 , and so, it suffices to prove that ab? + be? + cq? < 4, This is true according to the well-known inequality ab? + be? + ca? +abe $4 (see Remark from the proof of problem 2.43). Equality holds if and only if (4, b,c) = (0, 1, 2) o any eyclic permutation. et Problem 2.103. Let a, b, be positive real numbers such that abe = 1. Prove that a b c S>+ +——— > V3. VbP+2 Ve2+2a Vartan (Tran Quoc Anh) Three-variable inequalities 147 Proof. By Hilder’s Inequality, we have 2 (= ae) [Mac? +20] 2@+b +09 pF + 2c ‘Thus, the desired inequality holds if (a+b+c)* > 3(ab* + be? +.ca”) + 6(ab + be + ca). This is equivalent to a +b +c3+3(a7b + bc +c7a) + 6 > 6(ab + be + ca). By the AM-GM Inequality, we get b3+a?b +a7b +a°b +1412 6Va°b° = bab. Similarly, + b'e+b?c+b2e+1+1 > 6be, @B+ca+catc7a+1+12 bea. Adding these three inequalities, we obtain the desired result. Equality holds if and only ifa=b=c=1. a Problem 2.104. Let a, b, c be positive real numbers. Prove that 1 1 1 2 nota t= 2atb © 2bte “ W+a~ Va+byb+cKe+a) (Tran Quoc Anh) Proof. Multiplying each side of the desired inequality by a+b +c, we can write itas i (anne at > c 2atb+c 2atb A42a+b~ Yar ob+eyet+a) By the Cauchy-Schwarz Inequality, we have a+b 3.1 b 3.1 (a+b+ce)?? Sam Sopot Vos ata *2°2Saeath 2a+b 148 Vasile Cirtoaje - Vo Quoc Ba Can - Tran Quoc Anh and (atb+c? (at b+c)? 2a+b~ Sc(2a+b) © 3(ab+be+ca)? Therefore, it suffices to prove that (at+b+e)* 2atbtc) ET ep) Hab + be +ca) V@+ Yb +eNc+a) Now, using the well-known inequality 8 (at bb +eNle+a) 2 S(a+b+c)(ab + be +a), it suffices to show that at+btc) 2 ( y hosat (a+b +c} . 3(ab + be+ca) 3(ab + be +ca) But this is clearly true according to the Ate GM Inequality, so our proof is com- pleted. Equality holds if and only ifa = b =c. a 1 Problem 2.105. Let x, y, 2 be positive real numbers satisfying x + y +2 == xk 1 = +: =. Prove that F (ry + yz-+ax) (Vay + vz + Vax)? > 27. (Tran Quoc Anh) Proof. Setting a = yz, b = zx, c = xy, which yields x = je = Vo a ‘ab = (2 we have to prove thata + b+c=ab+be+ca implies (a+b+0)(Ya+ Vb+ve) 227, or, equivalently, 3v3 va — +Vb+ dex vatb+e Three-variable inequalities 149 This inequality can be written in the following homogeneous form 3V3(ab + be +ca) (=~, bo c 3 atbt+c at+bt+c a+bte (a+b+c)?? Since 2(ab + be + ca) =(a +b +c)? — a? — b? —c?, it is equivalent to 23V3, Z a 4 Be +b? +02) 25 atbtet (@tbtee 3v3a? @ El aeor a| eave. By the AM-GM Inequality, we have 3V3a? +¥ot (mm 3V3a Grbtept at+b+c a+b+c a+b+c Therefore, 3V3a? @ @ ee oe —*__ 23 yee a+bt+e 233) ae 3¥3. ‘The proof is completed. Equality holds if and only if x = y =2 = 1. a or Problem 2.106. Let a, b, c be nonnegative real numbers. Schur’s inequalities of third and fourth degree state that (@) aa b (ac) + b(b ~c)(b — a) + e(c— alle — b) > 0, @) — a(a—ba—c)+ b*(b—c)(b-a)+e%(c—al(e—b) >0. Prove that (b) is sharper than (a) if Va+V6+¥e>2max{ va, vb, ve} (Vasile Cirtoaje) Proof. The inequality f(a, b, c) > g(a, b, c) is said to be sharper than the inequality f (a, b, c) > h(a, b, c) if g(a, b,c) > h(a, b, c). Let us rewrite the inequalities (a) and (b) as abc 2 g(p, q) and abc 2 h(p, q), 150 Vasile Cirtoaje - Vo Quoc Ba Can - Tran Quoc Anh respectively, where p = a+b +c and q = ab + be +ca. Then, we need to show that g(p, 4) $ hp, q) for Va + Vb + Ye > 2max { Va, VB, ve}. We write (a) as +b +°4+8abe> Y'ab(a +b), +b +3 +6abe > (a+b +c)(ab + be +ca), (a+b+c)?+9abe > 4(a +b +c)(ab+be +ca), abe > a(p, q), where a, D= Analogously, we write (b) as p(4q-p*) a. a+ b* +4 +abe(a+b+c)> Y abla? +52), a*+b4 +444 2abe(a t+ b+¢) > (ab+be +ca)(a? + b? +c), and from +b +c —2q and at + bY +c = (a2 +b? +02)? 2(0b? + bc? 4 2a) = (p? — 2g)? — 29? + apabe, we find that (b) is equivalent to abc 2 A(p, q), where 2 % le, q= @ Usa ) »p Since 4 5 Mp, a)~ a(p, y= 2 Sea) p?—3q>0and 4q— p? =2(ab + be +ca)— a? — b? — <2 = (Vo+ve-va) (ve+va-vo) (va+Ve- ve) (ov). the conclusion follows. o Three-variable inequalities 151 Remark. If a, b, c are the side-lengths of a triangle, then the fourth degree Schur's Inequality is sharper than the third degree Schur’s Inequality. Problem 2.107. Let a, b, c be positive real numbers. Prove that for any n > 1, the inequality holds arti yet ott (2 et) fare bte cta at+tb~ \b+c c+a' a+b 3 . (@oland 2009) Proof. The original inequality can be written as at gereae a" =(F+)- 3 La or equivalently, : wee a” _ Dat bt ter (o+4+< a > pre 2a tote. From the Cauchy-Schwarz Inequality, we have za™+b"+c", x a (a +b" +c"? bte~ at(b+c)+b(c+a)+c"(a+b) Therefore, it suffices to prove that 7+ b8 be gh a 2 atbte— ete”. atlbtc)+b%c+a) +e Corb) V3 a+ bh ec which is equivalent to hth potty cnet a+ bt + cr The last inequality can be written as atl prt 4. ont a at 4 bt pont 3 - 3 , which is true according to the Power Mean Inequality. This completes our proof, Note that equality holds if and only ifa=b =c. a 152 Vasile Cirtoaje - Vo Quoc Ba Can - Tran Quoc Anh Problem 2.108. Let a, b, c be distinct real numbers. Prove that the following inequality holds ab?+1 b’c2+1 ca? +1 _ 3 + + as (a—b) (b-c}®? (c~a? ~ 2 (Nguyen Van Thach) . . 1+ab ltbe l+ea ; First proof. Setting x = WE y= and z = Zant WE an easily check that xy + yz +x = 1. Therefore, by using the known inequality x? + y+e>xytyz+2x, we get ltab\? (1+be\? (1+ca $ + 21. @ a—b b-e c-a l-ab Similarly, setting u = ———-, —ca , we find that uv + a— c vw + wu = —1, and from the obvious inequality wy? +w? =(u+v+w)? —2(uv + vw + wu) > —2(uv + vw + wu), (=e) Ga) + Now, from (1) and (2), we deduce that 1+ab\? 1-ab 1+a?b? =D) “EG 5) = ea we obtain a ) >2 @ or Lkalb 1+b?c? 14ca? 3 (a-by? © (bc * (C-ay ~ 2 This is what we want to prove. o Second proof. From the AM-GM Inequality, we get a2b241 ae 1 Uae Law leap? [Ze On the other hand, it is easy to check that Dew ) an Lew (x): Three-variable inequalities 153 Therefore, we have Gao ?*(Dars) (Zsa) 28 - Yap) (Mars?- Ya Poe) (a—b)*(b=c)*(c—a)? and hence it suffices to prove that 12(S)a?— Sab) (Sas? - Yar be) 2 ofa b)*(b- c)*(c-a?. Since 2(S7a?- ab) = )(b—c)* and 6 (S)a2b* — Srabe) = S(2be - ca — ab}, we can apply the Cauchy-Schwarz Inequality to get 12(Y)e?— Yoav) (Ya?6*— Yrare) = [So- abe ~ca—ab)]” =9(Sat- Sav’)? = 9a b)*(b—c}*(e— a)’, as desired. The proof is completed. o Problem 2.109. Let x, y, 2 be real numbers. Prove that «(D8 Ds) (DF Det) = Dror aLee (i Chen) Proof. Notice that 25) Dy? = ot", 238-2 tye = Py, and DSP042)- 23 P92 = Vx03 +28 - ys - ya) = xy 22 +2), so the inequality can be written as [ho?-#¥] [Po-2] 2 [Sxo-2Fo +9]. 4 i 154 Vasile Cirtoaje - Vo Quoc Ba Can - Tran Quoe Anh In this form, we can see that it is a direct consequence of the Cauchy-Schwarz Inequality [~ot-#¥] [Ero -27] 2 [Eee -202-241]’ 2 [Sexo -vo +a]. Remark. Actually, the following identity holds (Le-Lye)(Dre-Lex) [Leora Denl = =3(x- yy -2)\@—x Pty +2) Problem 2.110. Let x, y, z be real numbers. Prove that 4(Sx¢- Dy) (Sy? -Yx*v2) >3 (Sy - Yvye) (Vasile Cirtoaje) Proof. We have 2Doxt- ay? = 107-2), xyz = Dry + yz — 22x), and Tey-Lewe Dey = Vises? y2)- ays yet He -) 3 1 a eDot ~ 2 )xy + yz — 2zx). Therefore, the inequality can be written in the form (Xo7-=¥] [ey +ys- rex}?] = [o?-2Gy tye- a} o which is a direct consequence of the Cauchy-Schwarz Inequality. Three-variable inequalities 155 Remark. Actually, the following identity holds 4(D- Dy") (Dre Lev) -3 (Ley De)’ = = [Mayetts?—ay)]”. Problem 2.111. Let a, b, c be nonnegative real numbers, no two of which are zero. Prove that 1 1 a 2v2 oe ee Vae+be Vb? +ca Ve? +ab Vab+be+ca (Pham Kim Hung) Proof. Without loss of generality, we may assume that a > b > c. By Hélder’s Inequality, we have 2 ) [(b+c)9(a? +be)+a°(b? +ca)+a5(c?+ab)] > (2a+b+c)’, Therefore, it suffices to prove that (2a+b+c)*(ab + be +ca) > 8(b +c)*(a? + be) + Ba2(b? +c? +.ab tac). Since this is a homogeneous inequality in a, b, c, we may suppose that b-+c = 1. Then, it becomes (2a + 1)8(a+ be) 2 8(a? + be) + 8a5(a + 1 —2be), or equivalently, [(2a + 1)° + 16a - 8] be + a(2a + 1)? — 8a? — 8a5(a +1) > 0. This is true because (2a + 1)? > (b+¢ +1)? =8 and a(2a+ 1)° - 8a? — 8a5(a +1) = a(a—1)? +302 > 0, The proof is completed. Equality does not hold. a Problem 2.112. Let a, b, c be nonnegative real numbers. Prove that S(a+b+e)>2( Vad? +be+ ‘4b? +cat Vac? +ab). (Vo Quoc Ba Can) 156 Vasile Cirtoaje - Vo Quoc Ba Can - Tran Quoc Anh Proof. Without loss of generality, we may assume that a > b > c. In the non. trivial case b > 0, applying the AM-GM Inequality, we have qatnbe saa ee t tana yey On eg Oe ae gap CO ave oe 2a+ec = 2a+e 4b? +ca c(a — 2b) Vab?4+ca< oo igh ae 2Vab?-+ca$2b+e+ = 4b be = (ab + 4c?) (8c -a) c , Aab+4e ©, (8c-a a 2a+c 2b+c™ This inequality is equivalent to 2a-b 14b-11¢e Qate 22b+e)~ which is obviously true because a > b > c. The proof is completed. Equality holds if and only ifa = b and ¢ = 0, or any cyclic permutation. a Problem 2.113. Let a, b, c be real numbers such that a? +b? +c? = 2(ab-+be+ ca) > 0. Prove that md, eval, ol va?+2be Vb? +2ca yc? + 2ab (Tran Quoc Anh) = 6a > 0, which is Proof. For a = b = c, the given hypothesis becomes 3a” impossible. So, we must have (a—b)2+(b—c)? > 0, (b—¢)? +(c—a)? > 0 and (a— by’ +(a~c)? > 0. Now, by using the given hypothesis, we have a” +2be = (a—b)+(a—c)?. Therefore, lb-c| var+2be VC ‘Three-variable inequalities 157 and thus, the inequality is equivalent to x y z + + 22, yts Vetx \xty where x = (b—c)*, y = (c—a)? and z = (a — b)’. inequality, which can be proved using the AM-GM Inequality x + (y +z) > 2y/x(y +s) written in the form 2x a xtyts" We have equality for a = b and c = 0, or any cyclic permutation, and again for a= 4b = 4c, or any cyclic permutation. a Problem 2.114. Let a, b, c be nonnegative real numbers such that a+2b+3c = 4. Prove that (a°b + bc +a + abc)(ab? + bc? +ca® + abc) <8. (Tran Quoc Anh, MIC 2009) Proof. Applying the AM-GM Inequality, we have 2 2(Yeo+ abc) (Slab? + abe) < [Citorete) +2(Zoat rete) Therefore, it suffices to prove that Mon +2) ab? + sabe <8. This is true because Yi +2) ab? +3abc < Ya? +2 Fab? + abe _ (a+2b)(b + 2c)(c + 2a) 2 _ (a@+2b)(4b + 8c)(c + 2a) = . and a+2b+4b+8ce+c+2a (a+2b)(4b+8e)(c+2a) < ( : 3 ) =(a+2b+3c)? = 64. The proof is completed. Equality holds if and only ifa = 2, b=1ande=0. 0 158 Vasile Cirtoaje - Vo Quoc Ba Can - Tran Quoc Anh Problem 2.115. Let a, b, ¢ be real numbers satisfying a + b +c = 2. Prove that (a+b—ab)(b+c— be)(e +a—ca) $1—abe. (Tran Minh Vuong, Vo Quoc Ba Can) Proof. Setting A=(a—1)(b—1), B=(b—1)(c~1) and C=(c—1)(a-1), we can write the inequality as (1-4) -B)(1-C) <1—abe, or ABC —(AB+BC +CA)+A+B+C —abc>0. We have AB+BC+CA=(a—1)(b-1)(c — lat b+c-3)=—(a-1(b-1e-V, and A+B4+C—abe =D (a- 1b -1)— abe = jab-2) a +3—abe Mat Vab-abe=(—a)(1—b)-o). Therefore, —(AB+BC +CA)+A+B+C—abc =0, and hence, our inequality is equivalent to ABC > 0, which is true because ABC =(a—1)%(b —1)(e-1)? 20. Equality holds if and only if a =1, or b = 1, or ¢ = Problem 2.116. Let a, b, ¢ be nonnegative real numbers, no two of which are zero. Prove that b?- be +c? cata? | a ab Hb" , Mabie eee @+be * b?+ca ct+ab a+b? +c) (Bui Ngoc Anh) ie Three-variable inequalities : 159 Proof. By the AM-GM Inequality, we get (b? = be +c?)(a2 + bc) < E Then rbete?+a%t+be a2+b2 +c? 2 2 {- be+c? — be +c? 2(0? = be +c) Poe” Vb tbe) PD tee Adding this and its analogous inequalities, we obtain zy b? Bebete _ 2ab+ be + ca) ae +be @+R +e b= be +c? Clack beige) yy 24. athe @4hee This is what we want to prove. Equality holds if and only ifa = b and ¢ = 0, or any cyclic permutation. a Problem 2.117. Let a, b, ¢ be positive real numbers. Prove that i ¥ Hf + @tab+b? BP +be+e B+ b+c at+bt+e 2 cat (Phan Thanh Viet) Proof. Since a3 +63 +c? =3abe+(a+b +c)(a? +b? +c? —ab—be~ca), we can write the desired inequality in the following form . Babe a eee gay ob | (saan e «2 we xy abs Babe a@tab+b? ~atb+c Now, using the Cauchy-Schwarz Inequality, we get 4 (lathe) [Neta? +a0+0%] > (Svasx)” =abelatb +c), or equivalently, 160 Vasile Cirtoaje - Vo Quoc Ba Can - Tran Quoc Anh It follows that ab? abe(atb+c)? — abc(at+b +c) igre ~ Sela? +ab+ B23) ab+be+ca’ and hence, it suffices to prove the following inequality abc(a+b+c)_ 3abe ab+be+ca ~a+b+c’ which is true according to the well-known inequality (a + b +c)” > 3(ab+be+ ca). The proof is completed. Note that equality holds if and only if a = b = ¢. o Chapter 3 Four-variable inequalities “Mathematics is the door and key to the scienc Roger Bacon 3.1 Statements 3.1. Ifa, b,c, d are nonnegative real numbers, then a—b é b-c 4 c-d d-a 30 at+2b+c b+2ct+d ct+2d+a d+2a+b~ 3.2. Ifa, b, c, d are positive real numbers, then Vato +olatb+d)> Yact bd. 3.3. Leta, b,c, d be positive real numbers such that (a? +1)(b? + 1)(c? +1)(d? +1) = 16. Prove that abe + bed +cda + dab < 4. 3.4. Ifa, b,c, d are real numbers, then 6(a? +b? +c? +d?) + (a+b +e +d)’ > 12(ab + be +d), 161 162 Vasile Cirtoaje - Vo Quoc Ba Can - Tran Quoc Anh 3.5. Let a, b, ¢, d be positive real numbers such that 11,1,1 typtoty at+btctd= “ ab a Prove that (1 -abed) (f+ tee par 3.6. Let a, b, c, d be positive real numbers such that abcd = 1. Prove that (1407140042) +d) > (a+b +044), 3.7. Leta, b, c, d be nonnegative real numbers such that a2 +b? +c? 4d? =1, Prove that Vl-at+ V1—b+Vi-ct V1-d> a+ Vb4 Ve4 Va. 3.8. Let a, b, c,d be real numbers. Prove that (a~c)*(b—d}? +4(a- b)(b —c)(c- ad — a) > 0. 3.9. Leta, b, c, d be positive real numbers satisfying the condition a+b-+e+d = abe + bed + cda +dab. Prove that GFR odd ¢ et atl b+ 3.10. Let a, b, c, d be positive real numbers such that a-+ b +c + d=abe+ bed + cda + dab. Prove that 2 2 (varvi+ +1) +(ve+1+ Vea) S(a+b+c+dy 3.11, Let a, b,c, d be nonnegative real numbers such that a-+b-+¢+d=1and max{a, b, ¢, d} <1. Prove that (1+ 2a)(1 +.2b)(1 + 2c)(1 + 2d) s 125 Q-a)-b)a-cla-a) “38° Four-variable inequalities _ 163 3.12. Let a, b, c, d be nonnegative real numbers such that (a+b +c +d)? = 3(a? + b? +c? +d”). Prove that (a+b+c+d)> <27(abe + bed + cda + dab). 3.13. Let a, b, c, d be positive real numbers such that a+ b+¢c+d = 4, Prove that 1+2(abe + bed +cda + dab) = 9minfa, b, c, d}. 3.14. Let a, b,c, d be nonnegative real numbers such that a2+b?+c2+d? = 4. Prove that a+b+ce+d—4> (2-V2) (ab +be+cd+da-4). 3.15. Let a, b, c, d be positive real numbers such that a+b +c-4+d =4. Prove ha me Ljajaia ab" be cd’ da Sa 4b +2402. 3.16. Ifa > b> c > d>Osuch thata+b+c+d=2, then ab(b +c) +be(e +d) +ed(d+a)+da(a+b) <1. 3.17. Ifa, b,c, d are positive real numbers, then 1 1 1 1 16 —— +# +——— > ——___.. a(1+b) b(1+a) © c(1+d) © d(l+c) ~ 14.8Vabed 3.18. Ifa, b, c, d are positive real numbers, then (a—byla-c) (b-c)(b-d) (e~dye~a) , (d= aNd ~b) | a+b+c bt+et+d c+d+a dt+atb ~ 3.19. Ifa, b, c, d are positive real numbers, then (a+b\(b+e)(e+d)(d-+a) ( + Vabea)" > l6abed(1+a)(1+b)\(1+c\(1+4). 164 Vasile Cirtoaje - Vo Quoc Ba Can - Tran Quoc Anh 3.2 Solutions Problem 3.1. Ifa, b,c, d are nonnegative real numbers, then ab ob c-d d-a | a+2b+c"b+20+d c+2dta d+ b Proof. Write first the inequality as x a-b gi 1 Sa a+2b+e 2)" Bate Des a+2b+c By the Cauchy-Schwarz Inequality, we have yet [Neate]? TE a+ 2b+c™ S3a+cla+2b-+0) or Since (8a + c)(a+2b+c)=4(a+b +04 d)? and [Mea +o]°= 16la+b+c+dy’, ‘We get 3a+c Dashes at2bt+c~™” as desired. Equality holds if and only if a= and b =d, o Problem 3.2. Ifa, b, c, d are positive real numbers, then Vatb+c\a+b+d)> Yac+ Ved. (Russian Olympiad) Proof. By the AM-GM Inequality, we have a é a+b ac ——+ + 233 a+b atb+c atb+d (a+b+c)a+b+d)’ b +b ee a ae a b atb+c at+b+d (a+b+c)atb+d)" Adding these inequalities, the conclusion follows. Equality holds if and only if a c and bat Four-variable inequalities _ Problem 3.3. Let a, b, c, d be positive real numbers such that (a? + 1)(b? + 1c? + 1)(d? +1) = 16. Prove that abc + bed +cda + dab < 4. (Tran Minh Vuong) Proof. From the Cauchy-Schwara Inequality, we have 16=(a? +101 + bc? +0 +2) > (a+ b/c +d)’, and 16 =(a?-+1)(b? +11 +021 4a) > [ab +1) tea)? = (va + ved) * According to these two inequalities, we deduce that (a + b)(¢ +d) < 4 and Vab + Vcd < 2. Now, using the AM-GM Inequality, we get abe + bed +cda + dab = ab(c-+d)+ed(a+b) +b +d 12(ab+be+cd). (Vasile Cirtoaje) Proof. The original inequality can be written as 6(a? +b? +c? 4d?) + 12da+(a+b+c+d)’ > 12ab + be +d +da), or equivalently, 6(a +d)? + 6(b? +c?)+(at+b+ce+d)?>1A%a+cb+d). Using twice the Cauchy-Schwarz Inequality, we get (at+d+b+c)? 6(a+d)?+6(b?+c7) > (a+d)*+3(b+e)? > T= a(atb+c+d)? sHe 6 3 166 Vasile Cirtoaje - Vo Quoc Ba Can - Tran Quoc Anh Thus, it suffices to prove that 3(a+b+c+d)’ > 12a +c)(b +d). By the AM-GM Inequality, this is obviously true, and the proof is completed. Note that equality holds if and only if 2a = b =c = 2d. o Problem 3.5. Let a, b, c, d be positive real numbers such that 11141 atbt+c+d=-4—4-42, a bteta Prove that 1 1: 1 1 . Paptepeut 2 av qa abed) (a +b°+c? +d 2 pa z) 2° Proof. From the given hypothesis, we have that 11,1 arbdretar= (Siete a'’bve and thus, it follows that Le- The inequality becomes ab =?) (3 ~ 20, ym which is obviously true. a Problem 3.6. Let a, b,c, d be Positive real numbers such that abcd = 1. Prove that (1 +a?Q1 +7901 +02) +42) > (a+ b+ce+ay. (Pham Kim Hung) Four-variable inequalities 167 Proof. Among a, b, ¢, d there are two numbers which are less than or equal to 1, or greater than or equal to 1. Let b and d be these numbers; that is (b — 1)(d— 1) = 0. Applying the Cauchy-Schwarz Inequality, we get (14a? +b? +07b?)(c? +144? 407d?) 2(c+a+bd+abed)y’ =(at+e+1+bd)*. (1 407)(1 +b?) +62)(1 + d? Thus, it suffices to show that atc+1+bd2a+bt+c+d. This inequality is equivalent to (b — 1)(d — 1) > 0, which is true. Equality holds if and only ifa=b=c Problem 3.7. Let a, b,c, d be nonnegative real numbers such that a? + b? + c2 + @? = 1. Prove that Vi-at+ V1-6+ V1—c+ V1—d> vat Vb+ve+Va. (Vasile Circoaje) Proof. We can obtain the desired inequality by summing the inequalities vi-a+ V1-b> ve+ Via, Vi-c+V1-d>Vatyb. Vi-a+yV1-b22V(1—a)—b), 24 qe ver fa sa] *4 . Similarly, we can prove the second inequality. Equality holds if and only if a bae=d=5. o 168 Vasile Cirtoaje - Vo Quoc Ba Can - Tran Quoe Anh Problem 3.8. Let a, b,c, d be real numbers. Prove that (a -c}*(b—d)? + 4(a — b)(b — cc — dd — a) = 0. Proof. Let x =a—b, y = b—c,% =c~-d and t = d—a. Then, we have xt+ytzt+t=0,a-c=x+y and b—d=y +z, The original inequality becomes (xt yPly +2)? + 4xyzt > 0. (x+y +2), this inequality is equivalent to Replacing ¢ = (x+yPly +2 2 4xye(xt+y +2). By the AM-GM Inequality, we have (xt+yPOr +2) = [yety +2) 4x2}? > 4xya(x ty + so the last inequality is true. The proof is completed. Equality holds if and only if y(x + y +2) = xz, or (a—b)(c—d) +(b-e)(d a) =0. a Problem 3. b+c+d= . Let a, b, ¢, d be positive real numbers satisfying the condition a+ be + bed +cda + dab. Prove that atbtetd+— 4 ey te a1 b+1 c+1 d+1~ (Vo Quoc Ba Can) Proof. Put x =a+b+ce+d = abe + bed +cda +dab; then, by MacLaurin’s Inequality 8. (a+b +c+d)° > 16(abe + bed + eda + dab), it follows that x > 4. By Newton's Inequality, we have 29 ab > 3y (S72) (abe) =3, and by the Cauchy-Schwarz Inequality, we get y 2a 2x2 2x2 5 2? 2x x+x2-2) ab xP +x-3x Sm Thus, it remains to prove that 2x x+——>8, x27 which is obviously true, since it can be rewritten as (x — 4)? > 0. Note that equality holds if and only ifa = b =¢ a Four-variable inequalities 169 Problem 3.10. Let a, b,c, d be positive real numbers such thata+b+c+d= abc + bed + cda + dab. Prove that 2 2 (verra4 Versi) +(Vveri+ +1) <(a+b+e+a), (Vo Quoc Ba Can) Proof. The inequality is equivalent to 44+2V (a? + 1)(b? +1) +2V (c? + 1d? +1) < A(ab+ac+ad+be+bd+cd). Since a +b +c+d=abc + bed +cda + dab, > (a+ b)a+e)at+d) a +1 = ————____ atbt+c+d using the AM-GM Inequality, we have A(a + bP (a+ c)(b + dat d)(b +c) - (@tbte¢de at bY Heyl +d) + (a tab +e)]? ~ (a+b+c+dyP _ (btallb+c)(b+d) si il » bt a A(a? +1)(b?7 +) = which gives (a+ b[(a+c)(b+d)+(a+d\(b+c)] 24 1N(b2 Le eas a+b+c+d . Similarly, we get (ct+d)[(atc)b+d)+(a+d)(b+c)] z 2 pseee ee 2V(P +1 +s a+bierd . and hence 2V (a? + 1)(b? +1) +20 (c? + 1d? +1) S$ (a +c)(b+d)+(at+a)b+e). Therefore, it is enough to prove that 4+(atc\lb+d)+(at+d)(b +c) $2(ab+ac+ad +be + bd +cd), or equivalently, (a+bye+d)>4. Using the hypothesis, we can write this as (a+ bYct+d)a+b+c+d)> Alable+d)+cd(a+b)], which is equivalent to the obvious inequality (c+d)(a—b)? +(a+by(e-d)? > 0. 1 Equality holds if and only if a 170 Vasile Cirtoaje - Vo Quoc Ba Can - Tran Quoc Anh Remark. Note that this inequality is sharper than the following one due to Gabriel Dospinescu @+1 b?41 +1 a+1 atbtet+d> y—— 4 yf fot 2 2 * 2 +y 2° Problem 3.11. Let a, b, c, d be nonnegative real numbers such that a-+b+ce+d = Land maxfa, b, c, d} <1. Prove that (1+ 2a)(1 + 26) +.26)(1 +2d) 5s 125 (-aj1-b)a-cG=d) 38” (Wasile Cirtoaje) Proof. Without loss of generality, we may assume that a > b > c > d. From the hypothesis a+b +c +d = 1, we have 2(a+ b) > 1 > 2(c +d), Therefore, we have (1+2c)(1 + 2d) 1+2(c+d)+4cd 1-2(c+d)] (1—c)(1—d) 1-(c+d)+cd 1-(c+d)+ed pq 302+] _ 14+2(c+d) 1-(c+d) 1-(c+d)° and (+2a1+2b) _, 3[2(a+b)-1] 3[2(a + b)—1] G=agi—5) **T-@eby yap 24+ CES 1 (a+b) +> _ 4 +045? (2-a-by’ and so, it is enough to prove that 4 +a+b)* 14 2c+d) 125 (Q-a-bY 1=(c+d) ~~ x 1 Setting ¢ =a +b, we have 3 StS landc+d=1-~c. The above inequality becomes 4(1 + ¢)°(3 — 2t) 5 125 r2-1F eB Indeed, we have 4(1 + 2)?(3 - 2¢) _ 125 _ 3(2-3r)*(8- 72) e(2-¢2 8 8e(e-2)2 = Four-variable inequalities 171 and the proof is completed. Equality holds if and only if (a, b, c, d) is a permu- 4 1 La gh a 3 tation of | —, 3 Problem 3.12. Let a, b,c, d be nonnegative real numbers such that (a+b e+ d)’ = 3(a? + b? +c? + d?). Prove that (a+b+c+d)* < 27(abe + bed +cda + dab). (Pham Kim Hung) Proof. From the given hypothesis (a + b +c +d)? = 3(a? + b? +c? + d?), we deduce that Ya? =} ab, and hence oy oe -(Ze) (9). After expanding and simplifying, we get Neo =3} abe Therefore, the desired inequality can be written as 9(Xe") 2 (Xie) In the nontrivial case a+ b-+¢ +d > 0, we can rewrite this inequality as 9(Xe) (Sie) = (Sa) '=9(S2")’, ot, using the given hypothesis, (L°) (Gaye (Ley. The last inequality follows by the Cauchy-Schwarz Inequality. Therefore, our proof is completed. a Problem 3.13. Let a, b,c, d be positive real numbers such thata ++b+c+d=4. Prove that 1+ 2(abe + bed +cda+dab) > 9minfa, b, c, dj. (Vasile Ctrtoaje) 172 Vasile Cirtoaje - Vo Quoc Ba Can - Tran Quoc Anh Proof. Without loss of generality, we may assume that a = minfa, b, c, d}. From the condition a+ b +c +d = 4, we have a < 1, We need to prove that 1+ 2a(be + cd +db) + 2bed > 9a. since be+cd +db =(b—a)(c—a)+(c —a)(d —a) + (d —a)(b—a) + a[2(b +c +d) - 3a] > al2(b+c+d)—3a] = a[2(4—a) — 3a] = 8a —5a’, and bed =d(b —a)(c— a) +.a(d — a)(b +c 2a)+a°(b+e+d —2a) >a*(b+c+d—2a)=a"(4—3a), it suffices to prove that 1+ 2a(8a— 5a*) + 2a*(4—3a) > 9a, which is true because 14 2a(8a — 5a?) + 2a?(4 - 3a) — 9a =(1—a)(4a— 1)? = 0. =d=tl,or(a, b, ¢, d)isa Note that equality holds if and only if a = 11 Oo 1 tation of | 5, 5, 3 > perma (44,8 Problem 3.14. Let a, b,c, d be nonnegative real numbers such that a? + b? + co? +d? = 4. Prove that atb+c+d—4> (2-2) (ab+be+cd +da-4). (Vasile Cirtoaje) Proof. Lets =a+b-+c +d. From the Cauchy-Schwarz Inequality, we have 4=Vaatbet+c*+d)2s, and since a, b, c, d are nonnegative real numbers, sz Va?+b?+c? +d?=2, Applying the AM-GM Inequality, it follows that ab+bet+cd+da=(a+cibt+d)< 4 eS Four-variable inequalities _ 173 Thus, if va(* —4> (2-v2)(=-4), s-42 (2-v2) [5 then the desired inequality is true. Since s—4—(2-v2) (5-4) = 4-0 (e-209) 20 for s > 2V2, it remains to consider the case s < 2V2. We have ab+be+cd+da abed(a? + b? +c? +”). Since =(a-c)*20, (a+c)* — Bac(a? +c? and (b+d)* —8bd(b* +d?) =(b—d)* 20, 174 Vasile Cirtoaje - Vo Quoc Ba Can - Tran Quoc Anh we get bd(a +c)* +ac(b +d)* > Sabcd(a? + b? +c? +2). Therefore, it suffices to show that 8(a + c)[b+d) > bd(a+c)* +ac(b+4)*. Since 4bd <(b +d)? and 4ac < (a +c)?, it is enough to prove that 32(a +c)(b +d) >(b+dP(atc)*+(a+c)*(b +a). This inequality is equivalent to each of the following 322 (b+d)(atc)>+(atc)(b +a), (atb+c+d)'> 8(b+d)(a +c)? +8(a+c)(b +d), (a+c-b-d)*>0, the last inequality being clearly true. Equality holds if and only if a = b = d=1. Second proof. We write the inequality as f(a, b, c, d) = 0, where F(a, b, c, d)=(a+e)(b+d) —abed(a? + b? +c? + a2), and prove that f(a,b,c,d) > f(x,y,x,y)>0, ate b+d and y=, x+y =2. The left inequality is equivalent to where x 2x? y?(x? + y?) > abcd(a? +b? +c? + d?), that is bd(a? +c?) > ; a Without loss of generality, assume that z ee Pd a > ac(b? +d?), Since y? > bd, it suffices to show that 2x?(x?-+ y?) > ac(a? + b? +c? +42), On the other hand, since 4 a 2: 1 4 2x4 —acla? +02) = F(a—e)* 2 0, it suffices to show that 2x?y? > ac(b? +d”). Four-variable inequalities _ 175 This inequality follows by summing the inequalities bd(a? +c?) > ac(b? +d), and 4x?y? > bd(a? +c”) + ac(b? + d?). The last inequality is true, because 1 4x?y? — bd(a? +c?) —ac(b? +d?) = ga -cP(b-dP 20. The right inequality, f(x, y, x, y) 20, is true because F(x, yx, y)=4xy — 2x72? +?) xy = Zoet yy exyGt+ y= Bae-yytz 0. Problem 3.16. [fa > b 2c 2d 20 such thata+b+c+d=2, then ab(b-+e)+be(c +d) +ed(d +a) +da(a+b) <1. (Vasile Cirtoaje) First proof. The inequality is equivalent to 1+(a+c—b—d)(ab+cd)>(a+c)P(b +d) Since a+¢—b—d>Oand 2(ab +cd)—(a+c)(b+d)=(a-c)(b-d)>0, it suffices to show that 2+(atc—b—d)ate)(b+d) > Aatc)(b +d). This inequality is equivalent to 12 (a+c)(b+d), which follows immediately from the AM-GM Inequality (at+c+b+d)*>4(a+c)(b+d). Equality holds if and only ifa = b >c=danda+e=1. a 176 Vasile Cirtoaje - Vo Quoc Ba Can - Tran Quoc Anh Second proof. Since Mes - Sab? = = (ab? + be? + ca — ab ~ bc — ca) + (cd? + da? + ac? — c2d — da — ae) = (a~b)(b— ce - a) + (c-d)(d -a)(a—c) <0, it suffices to show that D+ Vad? +2 abe <2. We have Died + Sab? +2) abe = Sab + ab? + abe +aba) =(at+b+c+d)} ab =2a+c)(b +d) Problem 3.17. If a, b, ¢, d are positive real numbers, then 1 i, 1 1 16 ot oo tt or + > 22 . a(1+b) b(1+a) c(1+d) d(l+c)~ 14+8Vabed (Vasile Cirtoaje) Proof. We write the inequality as at+b+2ab c+d+2cd ss. 16 ae ree — ab(1+a)(1+b) cd(1+e)(1+d)~ 148Vabed We claim that a+b+2ab 1 ab(1+a)(1+b) ~ ab’ for ab > 1, and a+b+2ab ab(1+a)(1+b)~ Yab+ab for ab < 1, Indeed, the first inequality is clearly true, while the second inequality is equivalent to 2 (: - vah) (va- vo) 20. Let x = Vab and y = Vcd. Using this result for a and b, and the analogous result for c and d, there are four cases to consider Four-variable inequalities 177 Case 1. x > 1, y 2 1. It suffices to show that loi 16 = y? ~ 1+8xy Indeed, 1 1.2 16 = > . x?” y? “xy ~ 14 8xy Case 2. x <1, y <1. It suffices to show that at 1 8 3+ 2 xx? yty?~ 1+8xy This inequality is equivalent to s?+s5— 2p? 8 p'(s+p?+1)~ 1+8p?’ or (1+ 8p?)s? +s — 24p* — 10p? > 0, where s = x+y and p = /XY. Since s > 2p, we get (1+ 8p7)s? +s — 24p4 — 10p? > 4(1 + 8p”) p? + 2p — 24p4 — 10p? =2p(p+1)(2p-1)? 20. Case 3. x > 1, y < 1, We have to show that 1 16 yty 1+8ry" x This inequality is equivalent in succession to (14 8xy)(2x? + y? +y) = 16x*y((1+y), 1+ 8xy)(x-y)? +8x3y + Bry? — 16x3y + 2xy +x? +y 20, y (14 8xy)(x — y)? +8xy(x— 1)? +8xy?+x7+y > bxy. The last inequality is true because the AM-GM Inequality yields 8xy? +x? + y? > 3 Bx3y3 = bxy. Case 4. x <1, y 2 1. This case is similar to (c). 1 Equality n the original inequality occurs ifand only ifa=b=c=d=5. 0 ee 178 Vasile Cirtoaje - Vo Quoc Ba Can - Tran Quoc Anh Problem 3.18. If a, b, c,d are positive real numbers, then (a—byla-c) 4 @-9b~4) | €~dle~a) (d-a)(d-b) atb+c b+c+d c+d+a d+at+b (arij Grinberg, IMO Shortlist 2008) Proof. Let us denote by P(a, b, ¢, d) the left hand side of the original inequality. The key to solve this inequality is to notice that, without loss of generality, we may assume that (a~c)(b~d) > 0. Indeed, if (a—c)(b—d) < 0, then we can set a, = b, by =, ¢ =d, dy =a to have (a, — ¢)(b; — d}) = -(a—c)(b—d) =0 and P(ay, by, cy, d) = P(a, b, c, d). Now, we see that (a= bola~e) | (e~de~a)_ @=eP (a + 2e(a~c)b-d) atb+c ctdt+a a+bte (a+b+clat+c+d)’ and (b= b= a) | (d-ayld—b)_ (b= a? , (b+2d)(a~c)(b—a) beerd dt+at+b ~ bec+d* (b+c+d\dtatb) ‘Therefore, it is enough to show that (a-cP ( dP. (a+2cYa d) atbtc b+c+d~ (a+b+clate+d)™ According to the AM-GM Inequality, we have (@- | (b-ay 2(a-c)(b-d) atbte btetd™ Jatbrebte+d) Hence, it suffices to prove that +b+ 2at+e+d) [FS eat 20 a+b+c Ifa = d, then it is clear that brena= land 2(a+¢+d) 2 a+ 2c, so c+ the above inequality is true. Otherwise, if d > a, then we can easily check that arbre fate ly need sat Paerd = V oqa? 80 We only need to prove that Aatc+d)VaFe>(a+2c)Ve+d. Four-variable inequalities 179 Indeed, 2(at+ct+d)Vat+c=2Vatc+dy(atc+d)atc) 2Aatce)Vatce+d>Aatc)ve+d 2(a+2c)ve+d. Equality holds if a =c and b = Problem 3.19. If a, b, c, d are positive real numbers, then (at+b)(b+c)(et+d)(d-+a) (a+ Vebed)" > l6abed(1+a)(1+b)(1+c)(1+d). (Ukraine 2008) Proof. Firstly, we claim that for all x, y > 0, the following inequality holds xty V7 Q+x)0+y)~ (1+ yey)? Indeed, we have xty G+x04+y)_ (ze >) _ [ee | avy (a+ vay)’ vay (1+ vay)? _We- v9)? _ (ve- vy)? “25 (+ yey)? _ (VE- v¥)’ Gy +1) - yay (+ yey) This proves our claim. Now, by using the claim and the Cauchy-Schwarz In- equality, we have G+ byb+oye+ddta) . 4vabed(b +c)(d +a) (1+a)(1+b)(1+e)(1+d) ~ (1+ vab)’ (1+ ved)’ _, Wvabed (Vab + ved)” © (1+ ¥ab)’ (14 ved)? 2v vab- Ved 24Vabed | -————__—_, (14 VvaB- Yea) | 16abed (1+ Vabed)*” ee lL a sts—s—‘COwsisCCC(‘(‘(CNCOCOC(ONSC#iC 180 Vasile Cirtoaje - Vo Quoc Ba Can - Tran Quoc Anh 4 Multiplying each side of this inequality by [ ](1 +a) ( +V abed) swe get (a+b)(b+c)c-+d)(d+a) ( + Veved)” > l6abed(1+a)(1+b)(1+¢)(1+4), as desired. Note that equality holds if and only ifa =b =c=d. o Chapter 4 Five and six-variable inequalities “Inequality is the cause of all local movements." Leonardo da Vinci 4.1 Statements 4.1. Leta, b,c and x, y be positive real numbers such that x+y =2(a+b+c). Prove that ax? +by? +c 27abe. 4.2. Ifa>b2>c2>d2e, then (at+b+c+d+e)* > 8(ac + hd +ce). 4.3. Let a, b, c, d, e be positive real numbers such thata+b+c+d+e=5. Prove that abe + bed +cde+dea+eab <5. 4.4. Let a, b,c, d, e be nonnegative real numbers such thata+b+c+d+e Prove that ar.2 729 (a? + b?)(b? + c?)(c? + d?)(d? + e7)(e? +02) < > 181 182 Vasile Cirtoaje - Vo Quoc Ba Can - Tran Quoc Anh 4.5. Ifa, b,c, d, e are real numbers such thata+b+c+d+e=0, then a? +b? +c? +d? +e? > 3(ab + be +cd +de +ea). 4.6. Ifa, b, c,d, e are nonnegative real numbers such thata # b#c¢d Het a, then a b c d e pf gf os b-ad*e—d‘ da e—al e-bl= 3. 4.7. Leta, b,c, d, e be nonnegative real numbers such thata+b+c+d+e=5. Prove that 31(a? +b? +c? +d? +7) 2 a*+ bt +c +d* +04 +150. 4.8. Let a, b,c, d, e be nonnegative real numbers, no three of which are zero, and such that a? + b? +c? + d? +e? = 5. Prove that eo eas bt+et+d ctdt+te d+eta e+a+b a+b+ce 3° 4.9. Let a, b, c, d, e be positive real numbers. Prove that atbtetdte yo ad Dita - by? coorema tt — Vabede > =. 5 © 20 atb+ctd+e™ 4.10. Let a, b, c, d, e be nonnegative real numbers such that a? + b?+c?+4?+ e? =5, Prove that 1 1 7=2a + 7—2b 4.11. Let a, b, c, d, e be positive real numbers such that ee ee a pF 2d? Ife > 2, then (a—1)(b - 1) -1)(a- 1) 2 (e- 14 Five and six-variable inequalities 183 4.12. Let a, b, c, d, e be positive real numbers such thata+b+c+d+e=5. 1.11 Prove that ° + +i4 7 eee eee a ob ce at+b?4+c?+d2+e2 4.13. Ifa, b,c, d, e are positive real numbers such that a?+b?+c?+d? +e? then abede(at + b* +4 +d4 +e4) <5. 4.14, Let a, b,c, x, y, z be real numbers such that (at+b+c)(xty+z)=3, and (a?+b?+07)(x?+y? +27) =4. Prove that ax+by+cz20. 4.15. Leta, b, c, x, y, z be positive real numbers such that a-+-b+c=x-+y +2. Prove that a b ——+——+ b+x ety atz >1 4.16. Leta, b,c, x, ¥,t€RandA=ax+by +cz,B=ay+bz+ex,C = az+ bx +cy. If |A—B| > 1, |B —C| = 1 and |C —A| > 1, show that 4 (+b? +07)(x? + y? +27) > =: 4.17. Leta, b,c and x, y, z be real numbers such thata+x > b+y>c+z>0 anda+b+c=x+y +z. Prove that ay + bx >ac+xz 4.18. Ifa, b, c and x, y, 2 are real numbers, then A(a? + x?)(b? + y?)(c? +2?) > 3(bex + cay + abz)*. 184 Vasile Cirtoaje - Vo Quoc Ba Can - Tran Quoc Anh 4.19. Leta, b,c, x, y, 2 be positive real numbers such that ax -+by +cz= xyz. Prove the inequality xty+2>vVatb+Vb+c+VveFa. 4.20. Let a, b, c and x, y, 2 be positive real numbers such that x + y +2 = a+b-+c. Prove that ax" + by? + cz? 4+xyz > 4abe. 4.21. Ifa, b,c and x, y, 2 are nonnegative real numbers, then 2 2 2 9 at b\e+y) b+OG+s GHalets) ~ broxte+ayrarby 4.22, Let a, b, c be the side-lengths of a triangle. If x, y, z are real numbers such that x + y +2=0, then yza(b+c—a)+sxb(c +a—b)+xycla+b—c)<0. 4.23. Let a, b, c be the side-lengths of a triangle. If x, y, z are real numbers, then (ya? + 2b? +. xc?)(ea? + xb? + yc?) > (xy + yz +2x)(a2b? + b2c? + c2a?). 4.24, Let a, b,c and x, y, # be positive real numbers such that (a+ b+clxty +z) = (a? +b? +07)(x? +y? +2”) Prove that 1 abexyz< =. vF< 36 4.25. If x1, Xo, ..-,x6 €[0, 1], then iG deed xg 3 x§txhtxhtxht+o84+5 exp tas tagtaets 3 Five and six-variable inequalities 185 4.2 Solutions Problem 4.1. Let a, b, c and x, y be positive real numbers such that x + y = 2(a+b+c). Prove that ax? + by? +c > 27abe. Proof. By Hélder’s Inequality, we have ay be [2 b ay® thea AD. welts et ( 42 +4) (Vab + Vbe+ yea) va Ve Fe So, it suffices to show that [2(a+b +e)+1}? = 27 (veo + Vbe+ ve) Since a+b+c = Vab+ vbc + Va, it is enough to prove that [2a+b+ce)+1}§ > 27(a+b+c). 2 Setting s=a+b +c, the inequality becomes (25 +1)8 > 275, We can prove this either using AM-GM Inequality, s+s4+12 397, or writting it in the equivalent form (s — 1)°(8s + 1) > 0. Equality occurs if and only ifa=b =jadesyst a Problem 4.2. Ifa > b>c2d2>e, then (a+b+e+d +e)? > B8(ac + bd +ce). (Vasile Girtoaje) First proof. We have (at+b+c+d+e)—8(act+bd+ce)= (a+b+ctd+e—4c)+8(a+b+e+d +e)c—16c? — 8(ac + bd +ce) =(a+b+d+e— 3c)? +8(b—c)le—d)>0. From here the desired inequality follows. Equality occurs for either b = at+dte at+tb+e 2 186 Vasile Cirtoaje - Vo Quoc Ba Can - Tran Quoc Anh Second proof. For b =c =d, the inequality becomes (a+3b +e)? > 8(ab +b? + be), which is equivalent to the obvious inequality (a—b+e20. For b—d >0, let us denote E=(at+b+c+d +e) —8(ac + bd +ce). By the AM-GM Inequality, we have E>4(b+c)(atd+e)—8(ac+bd +ce)=4(b—c)(a—d te). From this inequality and the similar one, E>4(c+d)(a+b+e)—8(ac + bd +ce)=4(c—d)(b—a—e), we have (b-d)E=(b-CJE + (c— AE 2 4(b—c\(c—d)(b -a—e)+4(c—d)(b —c)(a-—d+e) =4(b—eNle- dba) > 0. Therefore, E > 0 for any real numbers b > c > d. a Remark 1. The following generalization holds: If a, > a) > +++ > d2,41, then (ay tag ters + Gabe) 2 ARC tepa + anMgy9 +--+ Ops 142K41)- For x = dy4;, we have ea a4 (% ~ ayy) + > (aj — x)(a ~ ayy) 2 0. a This inequality reduces to whe? ay tag tet dak4) 2 aydeys + daaey2 FF deg rake or ARP? + Ax (ay +g $+ + gus) 2 ARCA Ags + 2g +--+ degra)» Five and six-variable inequalities 187 Adding this inequality to the inequality (ay + ay +o- + aaug1 ~ kx)? 2 0, we get the desired inequality. Remark 2. Actually, the following more general statement holds (by Vasile Cir- toaje): 8 Way 2a 2a, and 2< 7 <4, then ay +g tert ag)? > mCayaeyy + andy ++ + dndeyn) 2 (where the subscripts are taken modulo n). For n = 4k and aa49 = dy43 = + = G4, = 0, we get the inequality from Remark 1. Problem 4.3. Let a, b,c, d, e be positive real numbers such thata+b+c+d+e= 5. Prove that abe +bed +cde+dea+eab <5. (Walther Janous) Proof. Due to cyclicity, we may assume that e = min{a, b, ¢, d, e}. Then, apply- ing the AM-GM Inequality, we get abc + bed +cde +dea + eab = bc(a+d —e) +e(a+c)(b+d) _— (aay < { ————_} +e( ——— 3 2 y 5—2e\$ +e = e 3 and thus, it is sufficient to prove that 5-2e\3 5-e\? ee pe (2ZE 3 "Va = This is true, since we have —2e\? fS-e\?_ 5 2s s-( 5 ) --() = zgle tae-1PE0 The proof is completed. It is easy to see that equality occurs if and only if a=b=c 188 Vasile Cirtoaje - Vo Quoc Ba Can - Tran Quoc Anh Problem 4.4. Let a, b, c, d, e be nonnegative real numbers such that a+b+e+ d+e=5. Prove that 729 a7 (Vasile Cirtoaje) (a? + b)(b? + 0?)(c? +d?)(d? + €?)(e? +a?) < Proof. Due to cyclicity, we may assume that e = minfa, b, c, d, e}. Setting e e xsat5 and y=d+5, we have : e +R —} and meee" wtvyt 2yv(y? + v2) < ( 3 — Therefore, it is sufficient to show that (ty 2 ~ 4 UM Five and six-variable inequalities 189 aeeys Indeed, by the AM-GM Inequality, we have uu ytyv ytv yt () ue 25 pee es ee 2, or, equivalently, 3 from which the conclusion follows. Equality occurs for (a, b,c, d, e) (3. 702 °) or any cyclic permutation thereof. o Problem 4.5. If a, b, c, d, e are real numbers such thata+b+c+d+e then a? +b? +02 +d? +e? > 3(ab + be +ed +de+ea). (Vasile Cirtoaje) First proof. By applying the AM-GM Inequality, we have Nalb-0) < Va? + (b-o*]=3 7 On the other hand, since 0= (Ya) =D et+2eb+2 ec, -2 ac = ya? +2) ab. 2d aco -)=Ya+4 >) ab. Combining this with the above inequality, we get Ne+4 dab <3 a?-2 5 ab, from which the desired result follows. Mao. we get It follows that Oo Second proof. We prove that sie +2(Sra)’ 215) ab, 190 Vasile Cirtoaje - Vo Quoc Ba Can - Tran Quoc Anh from which the desired inequality follows. Since (Sia) = Dats 2D ad +2 ec, the inequality becomes De+4 dace 11 ab. ‘We will show that there are p > 0 and q > 0 such that TYP +4> ac- 11) ab =p) la-a —q(b-c)]? 20. We have Nle-a-qb-o = =? (Yet Yad) 40 (Lev Lec) +2 et -2 Tee = 20? +1) Ya? + 2(2q - 1) ) ae ~ 2(q? + 24) Dab. Thus, the identity above holds if and only if 2p(q? +1) = 7, p(2q-1) = 2 and 2p(q? + 2q) = 11. All these equations are satisfied for p = 1- —= and vs _7+Vv5 a Remark. One can prove that the stronger inequality ab+be+cd+detea V5— a? +b? +c? +d? +e? 4 holds for any real numbers a, b, c, d, e such thata+b+c+d+e=0 and a? +b? +c +d? +e? > 0. This inequality follows by the identity Mar-(v5+ Yar (Xa)'= aL [2a-4)- (1+ 5) (@-«)]", which holds for any real numbers a, b, ¢, d, Problem 4.6. Ifa, b, c,d, e are nonnegative real numbers such that a #b #c # d#e#a, then (Vasile Cirtoaje) Five and six-variable inequalities 191 Proof. Let E be the left hand side of the inequality. Without loss of generality, assume that e = mina, b, c,d, e}, which implies a > 0 and d > 0. Since fx lea] |a—b| Id—e] we have a b cid E>——— 4 =+-. “ib-d te-al tata d By the AM-GM Inequality, ra +52 le-d] There are two cases to consider: b > c and b c. Write the above inequality as R>2 d bint ¢ 2 G-at ena t femal ta By the AM-GM Inequality, we have Boole ep f 24— > joel era? b=c [e-d le—dl ee le-d| and hence Ale lfc 3. Ifc > d, then $2n142 gS tas a7 tora 7124 Szaftst 23h ya £23 “at ed” This completes the proof. Equality holds for a = b = d and c =e = 0, or any cyclic permutation. a 192 Vasile Cirtoaje - Vo Quoc Ba Can - Tran Quoc Anh Problem 4.7. Let a, b, c, d, e be nonnegative real numbers such thata +b+c+ d+e=5. Prove that 31(a2 +b? +c7 +d? +e?) > at+bt+c44+d*+e4 +150. (Vasile Cirtoaje) Proof. We write the inequality as Seat- 31a? + 30a) <0, or Me-vs@ <0, where f (a) = a? +a?— 30a. Without loss of generality, assume that a > b > ¢> de. Since a+b <5, we have f(a) — f(b) = (a — b)[(a +b)? — ab +a +b — 30] <(a-b)[(a +b)? +a+b-30] =(a—b)a+b-S5a+b+6)<0. Similarly, we can prove that f (b)—f (c) < 0, f(c)—f(d) < 0 and f(d)—f(e) S 0. Since a-12>b-12>c~-12>d-12¢e-1 and F(a) S f(b) s f(c) S$ F(d) < fle), by Chebyshev’s Inequality it follows that sYe-vr@s [Ye-v] [Yo] On the assumption that a > b > c > d >e, equality occurs fora =b=c=d= e= 1, and again for a=5 and b =e=0. a 0. Problem 4.8. Let a, b, c, d, e be nonnegative real numbers, no three of which are zero, and such that a? + b* +c? +d? +e? = 5. Prove that a be 2 @ a 5 sta tt oo t eo te bt+ct+d c+d+e d+e+a e+atb atb+c™ 3 Five and six-variable inequalities 193 First proof. By the Cauchy-Schwarz Inequality, we have om @) 25 Lier Subsea Veer d = 25 _ 5v5 (EX) Meera] (Netore+aF 5v5 5v5 Bi Ne 3d 1a2(b? +0? + 42) 3 0(5- e*) =, /—_5 = 0 V¥3\ 25-Sat—S7are? V3) 50- S (a? +27 Therefore, it suffices to show that } (a? + ¢2}? > 20. Indeed, by the Cauchy- Schwarz Inequality we have Sa? +e? > [Sta? +e] =4 (Se?) = 100, from which the conclusion follows. Equality holds if and only if a = d= % Second proof. By the AM-GM Inequality, we get 2b+2c¢+2d <(b?+1)+(c?+1)+(d? 41) =8-a’ Therefore, it suffices to show that 2 c pierre By the Cauchy-Schwarz Inequality, we have : - Oey ts *Svte-a-2 ~yae 50 ge ne 80- Dwr+ey 80-5 [Set +ed)]* ° and the proof is completed. Qo eee 194 Vasile Cirtoaje - Vo Quoc Ba Can - Tran Quoc Anh Problem 4.9. Let a, b, c, d, e be positive real numbers. Prove that atbtctdte , 1 Y@-vy eee, abcde a 5 eee = 30 at btetdte (Vo Quoc Ba Can) Proof. Due to homogeneity, we may assume that a+b -+c +d -+e=5. Setting x = Vabede, the inequality becomes 100(1- x) = }(a- by. By the AM-GM Inequality, we have Sa- 5 =2 a? -2 ab =2(Soa?+2) ab +2 ac) - 6) ab-4 ac =2(Yia)’-6 ab -4 Yee 50-50 V.a2bc2d2e? = 50 — 50x?. Therefore, it suffices to prove that 100(1 — x) = 50 —50x?. This is equivalent to the obvious inequality 50(x-1/ 20, and the proof is completed. Equality holds if and only ifa=b=c=d=e. O Problem 4.10. Let a, b,c, d, e be nonnegative real numbers such that a? + b? + c? +d? +e? =5. Prove that tt gS ty te 7-2a 7=2b' 7-2" 7-2d 7-2e— (Vasile Cirtoaje) Proof. Without loss of generality, assume that V5 >a >b>c>d>e. Since Bete? V2b2+2e? — V(a2 +b?) +(c? +?) v5 OO BE = 2 a 6 Five and six-variable inequalities 195 and ain and 2 < 4 7-2 e+e | > (pte) +(2 +0 2 4 4 7-Vs-a If we show that 1 4 <1 -— ei, 7-20 5_fenae then we get the desired inequality by adding the above four inequalities. This inequality is equivalent to 7—3a 2 (3-a)V¥5—a’, By squaring, we get the obvious inequality (a-1)(a-2)? > 0. On our ordering hypothesis, equality holds for a 1, and again fora=2and b=c=d=e= 5. Lemma, Ix and y are nonnegative real numbers such that then 1 1 2 ie Jum * dy <7 A, aD 196 Vasile Cirtoaje - Vo Quoc Ba Can - Tran Quoc Anh Proof: For the nontrivial case x? + y? > 0, we have 7 =a lz Bers tO aos] sod@-yyf—_—2 +0 poo wT = TIP] + yx + 2y +25—7) (7 2x7 — 2y x +s)y +5) ; Ax+y)+25-7< 44 = +25-7=6s-7<0. Remark. The following more general statement holds: If a, a3, ..- dy are nonnegative real numbers such that a? + a3 +----+a2=n, then 0, since Oo for any p> 1+ Problem 4.11. Let a, b,c, d, ¢ be positive real numbers such that 1 1 1 1 x wrate 2 Ife > 2, then (a-1)(b — 1(e - 1d - 1) > (e- 194 (Vasile Girtoaje) Proof. We claim that Ha-b-2(e-1) (Fere-2), and we-Da-H2@-0(Fere-2). ab Five and six-variable inequalities 197 If these inequalities are true, then b d Ma~1Xb- led) (e- 17 (Gete-2) (Gere-2) ab cd =(e-1)? | 2e?-4e +44 (| — + — | e(e-2) a cd 2 (e- 1)? [2e? - 4e +4 + 2e(e — 2)] b and the conclusion follows. ‘To prove the first claimed inequality, we write it as b 2ab- 2a +b) +e(3~e)> ee. Since 22 ~0, which can be written in the obvious form [2ab —(a+b)e]}? + e(e — 2)(a—b)? > 0. Equality holds when a = b=c=d =e. a Problem 4.12. Let a, b,c, d, e be positive real numbers such thata+b+c+d+e = 5. Prove that 1,1,1,1 20 ~+o4+-4-4+5— 5 29. a boc e a?+b?+c*+d? +07 (Vasile Cirtoaje) Proof. Without loss of generality, assume that a 0. For the sake of contradiction, assume that £ is minimal for a < d. If we show that t . E(a, b,c, d, o>e(S% b, ¢; = ‘), then E is not minimal for a < d, and the conclusion follows. This inequality is equivalent to (a—d)® ™ 20(a — d)? ad(a+d)” (a? +b? +c? +0? +07)[(a+d)? + 2b? + 2c +207)" Since (a ~ d)? > 0 and 24 btactagaets t 2 a+b? +c7 +d? +e’ pgatbtcetd+eP=atbtctdte, it suffices to show that (atbt+ct+d+e)[(a+d)?+2b? + 2c? + 207] > 20ad(a +d). Since at+b+c+d+e>ata+a+d+d=3a+2d and (a+)? + 2b? + 2c? +20? > (a-+d)? +20? +20? + 2d? = Sa? +2ad +242, it is enough to prove that (3a + 2d)(Sa? + 2ad + 2d?) > 20ad(a + d), This is true since (3a + 2d)(Sa? + 2ad + 3d) — 20ad(a +d) = 15a3 — 4a2d — 7ad? + 6d? > Sa? — 4ad — 7ad? + 6d° =(a—d)*(Sa + 6d) >0. Thus, the proof is completed. Equality holds for a = b = 5 again fora =b =d=2 and e= 5, or any eyclic permutation, a 6 Five and six-variable inequalities 199 Remark. The following more general statement holds: If a, ay, ..., @, are positive real numbers such that a +a, +---+a, =n, then 2n+2¥n-1. Problem 4.13. If a, b, c, d, e are positive real numbers such that a? + b? +¢2+ d? +e? =5, then abcde(at+b*+c4+d*+e4) <5. (asile Cértoaje) Proof. Without loss of generality, assume thata < b 0. To prove this inequality, it suffices to show that b4 +c4+e4 > 20d”. Indeed, we have bi +c4+e4 — 207d? > b4 +.a* + d4 — 207d? = b* + (a? -d?)? > 0. To complete the proof, it suffices to show that the desired homogeneous inequal- ity a+betc2t+d2+e2)\? 4+ bt+ct+d4 tet pees > (abcde}* ——r 200 Vasile Cirtoaje - Vo Quoc Ba Can - Tran Quoc Anh holds for a = b =c =d = 1. Setting x = ?, we need to show that f(x) > 0 for x > 0, where 44x 2 FO) =9In= =Inx—2in 9 1 =e -2)7 prom fife) = 2-2 #8 _ Aor r= 29 44x x 443x7 x(44x)(44+x2) decreasing for 0 < x < 1 and increasing for x > 1. Therefore, f(x) > f (1 This completes the proof. Equality holds if and only if a =b=c=d 1 o , it follows that f (2x) is Remark. The following more general statement holds: If aj, a2, .-., dy are Positive real numbers such that a; +a, +---+a, =n, then (@ag++-n) F(a} taZ+---+02) glade tan)(br +b,) for any real numbers a1, @9, ++. dp aNd by, by, ..4 Bye Problem 4.15. Let a, b, c, x, y, % be positive real numbers such that a+b -+¢= x+y +z. Prove that a b i +—— >1 b+x cty atz (Ji Chen) Proof. Applying the Cauchy-Schwarz Inequality, we have a b c (atb+c) + + SS b+x cty atz~ alb+x)+b(e+y)+c(at+z) and thus, it is sufficient to prove that (a+b+c)>ab+be+catax+by +cz. Due to cyclicity, we may assume that a = max{a, b, c}. Then, we have ax + by +cz Sa(x+y+z)=a(at+b+c). It follows that (a+b+c)*—(ab+be+ca)—(ax+by +ez)> >(a+b+c)’-(ab+be+ca)—a(at+b+c) b+e)(at+b +c) —(ab + be + ca) =b? +c? + bce >0. The proof is completed. o Problem 4.16. Let a, b,c, x, y, t €IRand A=ax +by +s, B=ay + bz+cx, C=az+bx+cy. If |A—B| > 1, |B —C| > 1 and |C — Al > 1, show that 2 2 27-2 (2 2 4 (P+? +0) +y? +z 23 (Adrian Zahariuc) Ce eae eeseeeeesesesesesesesesesai‘i‘ie 202 Vasile Cirtoaje - Vo Quoc Ba Can - Tran Quoc Anh Proof. Without loss of generality, we may assume that A > B > C. Then, from the given hypothesis, we have |A-C| =A-C = (A-B)+(B-C)=|A—B|+|B-C|>2. This means that (A—C)? > 4, ie. [(a-b)x+(b-c)y +(c—a)z}? > 4. On the other hand, the Cauchy-Schwarz Inequality gives us that [a b)x+(b—c)y + (c- ale]? < (x? +y? +27)[(a— b)? + (b- ce}? +(€-a)"] = (x2 +y? +27)[3(a? +b? +07)—(at b+c)] £3(x? + y?4+27)(a? +b? +07). Combining this with the above inequality, we get 4 GP +? 42a? +b +°)> 5, as desired. The proof is completed. o } Problem 4.17. Let a, b, cand x, y, z be real numbers such that a+x 2 b+y2 c+s>Oanda+b+c=x+y +2. Prove that ay + bx > ac+x2. (Vasile Cirtoaje, Mircea Lascu) Proof. We have (Cy —c) +x(b-2) =a(a + b—x—s)+x(b—-2) =a(a-—x)+(a+x)(b—z) =ha-x)+ pet-)4+(e+ x)(b-#) ay + bx -ac—xz 2 Sa-xP+ Flat ala+2b— 2-22) =Ha-xP + Hat old +y-c-2)20, from which the required inequality follows. Equality occurs for a = x, b = 2, c=yand2x>y+s20. a Five and six-variable inequalities 203 Problem 4.18. if a, b,c and x, y, 2 are real numbers, then 4(a? + x?)(b? + y*)(c? +27) > 3(bex + cay +abz)*. (Vasile Cirtoaje) Proof. By the Cauchy-Schwarz Inequality, we have (a? +.x?)[(cy + bz)? + b?c?] > [a(cy + bz) + bex]?. Thus, we still have to show that A(b? + y*)(c? +27) > 3I(cy + bz)? + b?c?] This inequality reduces to (cy — bz)}* + (be — 2yz)’ 2 0, which is clearly true. In the case abc # 0, equality holds for ~ = % = = = a 2 a . Problem 4.19. Let a, b, c, x, y, 2 be positive real numbers such that ax-+by-+cx = xys. Prove the inequality xtytz>Vatb+Vb+c+Vve¥a. (MOSP 2007) ab First proof. The given hypothesis can be written as — + — +— = ye x” xy hence, we deduce that ab boc qc —+—<1l, ~—+—<1 —+—<1 ye 2x ax xy ye xy We thus have bic a a ee zy zx y Using this in combination with the AM-GM Inequality, we get boc a cab Aetytz)>—t+—t-+—+—+—4+xt+y4+2 zy B xX y x (2 ) ( ) (= ) = +2) +{(——+x])+ +y 2 x y 22Vat+b+2Vb+c+2Vve+a. Dividing each side of the last inequality by 2, the result follows immediately. 0 204 Vasile Cirtoaje - Vo Quoc Ba Can - Tran Quoc Anh Second proof. Write the inequality as 2 (eryseFertby tel), (Japp fires Mera). xyz Without loss of generality, we may assume that x is the largest number between x, y, 2. Then, by the AM-GM Inequality and the Cauchy-Schwarz Inequality, we have (xty +2)(ax + by +z) ss 4(y +2)(ax + by +c2) xyz yz 5 Ay +2)laly +2) + 2by + 2cz] = = a+2c a+2b =2y+2)( ye ) 22(Jar2e+ Vat2b) . It suffices to prove that a(Varae+ Var2) > (varb+Vbre+ vera)’, which is true because atb+Vb+c+veta< (Varb+ vo) + (vare+ Ve) < VXa+26)+ V2a+20), from the Cauchy-Schwarz Inequality. a Third proof. Without loss of generality, we may assume that a > b > c. Since 26+ 2a) + (2(b-+26) > (Vo+a+ va) + (Vo+e+ ve) > Vatb+Vb+ce+Vvera, it suffices to prove the stronger inequality xty+z2 f2(b+2a)+ V2(b+2c), which is equivalent to b (< al S)oty+e?za( b+2a+ bya). ye mx ey Five and six-variable inequalities 205 Now, using the AM-GM Inequality, we have (¢ tet Setyter= ya ax i eae i a yz ax? ay ax ax -DS+5(2+4 @) 423 a+2)(F+2) ax? ay y cz =S'—+ —+2 +2 = Deere TE aay (M+) =~, X(a+2b)+=(a+20)| +25) =F = ylat2e a > 2) Vas 2bya+20) +254 Therefore, it suffices to prove that 2 Met DY Var 20a +20) = (vo+20+ Vo+2c) After some simple computations, we may rewrite it as V(c+2a)(c+2b) — V(b +2a)(b+2c)>a+bt+e-yV(a+2b\la+2c), or equivalently, (b—c)(2a— b-c) s (b-c? V(c+2al(c+2b)+ f(b+2aNb+2e) atb+e+V(a+2bMa+ 2c) Because (b — c)(2a — b—c) = (b—c)* > 0, it is enough to check that atb+ce+V(a+2b)la+2c)> ¥(c+2a)(c+2b)+ V(b +2a)(b+2c), which is true since (a-+2b)(a-+2c)—(b +2a)(b +2c) = (a—b)(a+b—2c) > 0, and (a+b+c)*—(c+2a)c+2b) =(a—b)? >0. The proof is completed. a _ a bic Fourth proof. We write — + — + — = 1 and now the substitution a = yzu, ye ax xy b =2xv and c = xyw becomes natural. So, we need to prove that Va(yu+xv)+ /x(ev+yw)+ Vy(utxw) 4abe. (Vasile Cirtoaje) Proof. First, we see that (b+c—x)+(c+a—y)+(a+b—z)=at+b+c>0, Therefore, at least one of b+.¢—x,c+a—y,a+b—z is positive. Suppose that b+c—x > 0. On the other hand, if x? > 4bc then the inequality is trivial, so we will assume that 4bc > x?, Now, we write the inequality as 2 _ 32] > Babe, 2ax? + 2by? + 2c2? +x[(y +2)? - or 2ax? + (2b — x)y? + (2c — x)2” + x(y +2)" 2 Babe. Note that for any real numbers p, q, u, v with p +q > 0, we have 2 qv? put--qv?— Raut) — puny 3 pt+q pt+q Therefore, since (2b — x) + (2c — x) = 2(b +c — x) > 0, we have (2b —x)(2¢- x)(y +2)? mat? Be —acha? (2b —x)y?+ (2c x)? > hie w and then, it is sufficient to prove that 2, (2b—x)(2e— xy +z)? 2 +2) 2 Babe, 2ax? + Ab +e=x) +x(y +2)° 2 Babe, Five and six-variable inequalities 207 which is equivalent to (2b — x)(2c — x) x2 Wicos +x] > 20tabe x), (y+2)? [ or (y +2)*(4be — x?) a 5 20 4b - x?), Ab+e—x) ~ atabe 2") Since 4bc — x? > 0, this can be simplified to (y +z) 2 4a(b+e-x). This is true since, by the AM-GM Inequality, (y +2? =(a+b+ce-x) 2 4a(b+c-x). . ytz The proof is completed. Equality holds if and only if a = db x+y 2 -? Problem 4.21. Ifa, b, cand x, y, z are nonnegative real numbers, then 2 2 2 9 GtDe+y) Grog +H C+aG+D ~ bForte+aytatbhe Gi Chen, Vasile Cirtoaje) First proof. Due to homogeneity in a, b, ¢ and in x, y, 2, we may assume that a+b+c=x+y+z=1. Thus, the inequality becomes 2 4 3 +: 2 9 2 os > (1-e)-z) (1-a)(—x) * (1-6) = y) > 1-ax-by cz’ or 2 2 2 9 + + 1-(a+x)+ax "1-(b+y)+by © 1-(c+2)+ce~ 1-ax—by—cz atx b+ cts Let us denote p = as = where ptqtr =a Since ax S p?, by —, it becomes as follows abe BP +c cb +a*—b* a+b? <8 yz | —~—— +2 | tex | ———— +2 ] +xy | ———— +2] <0, be ca ab 2yzcosA+ 2zx cosB + 2xy cosC + Axy +yz+2x) <0, x? +y" +2? — 2yzcosA— 2zx cosB — 2xy cosC > 0, (x — y cos ~zcosB)’ +(y sinc —zsinB)? > 0. The last inequality is obviously true. Equality occurs if and only if x + y + = ¥ 2 Sind = dab = gags thats ifand only ifx = y =z =0 Q Second proof. Substituting x = —y — 2, the desired inequality becomes as fol- lows a(y +2)b(c +a-b)+y(y +2)c(a +b —c) > yza(b +e—a), ¥7c(a + b—c)+2"b(e+a—b) > yz[a(b+e—a)—cla+b—c)— b(e+a—b)], y*c(a+ bc) +2*b(c +a—b) > —yz(c +a—b)a+b—c). Since —yz(c +a— b)(a+b—c) < |yz|(c +a — b)(a+b—c) and yrcla+ b—c)+2*b(c +a—b) > 2lyz|be(c +a — ba +b—c), it suffices to prove that Abe > (c +a-b)(a+b—c). This inequality is equivalent to (b+c—a)(b+c+a)>0, which is obviously true. The proof is completed o Remark. For x -c,y=b-aandz b, we get the known inequality 7h? + bc? +070? Sab + b8e+C8a. Panat i 210 Vasile Cirtoaje - Vo Quoc Ba Can - Tran Quoc Anh Problem 4.23. Let a, b, ¢ be the side-lengths of a triangle. If x, y, z are real numbers, then (ya? +2b? + xc?)(za? + xb? + ye?) > (xy + yz +2x)(a2b? + b?c? + ¢7a), (Vasile Cirtoaje) Proof. We write the inequality as follows Ye? > yza?(b? +c? —a”) +2xb"(c? +a? — b*) + xyc*(a? + b? —c?), zx(c?+a?—b?) __ xy(a?+b?—c?) y2(b? +c? — a? Pa? ab? 2yzcosA 2excosB 2xycosC Beer oa eb * z 2 2 G -Xeosc- Fcoss) f. (Fsinc ~Esinz) =o a8 ¢ b ¢ Since the ls nequllry ig leary tre, the proofs completed. Equality occu if and only ce a 1 = and = =, we get again the well-known inequality ve Remark 1. For x = 5 Bb +b8c+c8a > ab? + b7c? +070? i i, Remark 2. For x=, y = = and = 55, we obtain the elegant asymmetric inequality of Walker (Math. whe. 43, iy 2 2 a(S+ s+ S) esse) ¢ Problem 4.24. Let a, b, c and x, y, be positive real numbers such that (atb+clxt+y+z)=(a? +b? +07)(x? +y? +27) = 4. Prove that 1 bi <= abexyz < (Vasile Cirtoaje, Mircea Lascu) Five and six-variable inequalities 211 Proof. Using the given relations and the AM-GM Inequality, we have A(ab + be+ca)(xy + yz+2zx)= =[(atb+cP—(a? +b? +0) [(x+y +2? -(CP+y? +27)] =20—(atb+ cP ty? +2%)—(xty +2? +b? +07) S20-2atbtelxty+z)V (a+b? +c2)(x2 + y2 +22) =4, and hence (ab + be +ca)(xy + yz+2x) <1. On the other hand, multiplying the well-known inequalities (ab+be+ca)? > 3abc(a+b+c), and (xy + yz +2x) > 3xy2(x+y +2), we get (ab + be +ca}*(xy + yz +2x)* > 36abexyz. Therefore, we get 12 (ab+be +ca)*(xy + yz +2x)* > 36xyz, 1 which implies abexys < 32. To have equality, its necessary to have (ab-+ be + ca)? = 3abc(a + b +c) and (xy + yz + 2x)? = 3xyz(x + y +2). But these relations imply a = b = c and x = y =z, which contradict the hypothesis (a+b+elxtytz)=(a? +b? +07)(x? +y? +27 1 Consequently, we have abexya < 3. Problem 4.25. If x1, Xp,.+.,X6€[0, 1], then xf x x8 +8438 + x8 42845 tah txh+xd+xb+5 (Ukraine 1999) 212 Vasile Cirtoaje - Vo Quoc Ba Can - Tran Quoc Anh Proof. We have xh tah tohtadtaf+52 x8 tah tah toh + bt t 4. Using this and its analogous inequalities, it suffices to show that Since 6 6 i 3 (oe ¥) = axe +2)25>)7x?, a fi the conclusion follows. Equality holds if and only if all x; are equalto1. Chapter 5 n-variable inequalities “Tris not enough to have a good mind, the main thing és to use it well.” Rene Descartes 5.1 Statements 5.1. If ay, aa, ..., dy (n 2 3) are positive real numbers, then 4; n ——_- <3, 1+ 2a; +i, 4 where ag = 5.2. Let ay, a2, -.., 4, € [0, 1] and S =a? +a} +--+: +a. Prove that a ay a, 1 —— + 9 +. cs Qn+1+S$—as In+1+S—a} 2n+1+S-a: 5.3. If x), Xg, ++ Xp ate positive real numbers such that x2+.x} +--+ then 1/< 1 i(Bae Ez} ee 213 214 Vasile Cirtoaje - Vo Quoc Ba Can - Tran Quoc Anh 5.4. Let x, Xp, ..., X, be positive real numbers such that x, x2 = 1. Prove that a n Sxra4x= = []a+x. i= 5.5. Prove that, for all real numbers aj, @,..., 4, €[1, 2], we have x 3 Gai + ais where the subscripts are taken modulo n. 5.6. Let a), da, ..., a, be positive real numbers such that a;a---a, = 1. Prove that 1 1 1 + she 1+ Y1+4n(n=Da, 1+ f1+4n(n= Day 1+ Y1+4n(n- 1a, 5.7. Let a), dg, ..., a, be positive real numbers. Prove that @ (ay tay +++ + aq)” “ (aj +143 +1)---(a2 +1) ry a, tay te tay (a2 + 1a} +1) (2 +1) 5.8. Let x1, xp, ..., Xq be positive real numbers such that xy +.) +--++%p Prove that n 2 =n) 5.9. Let a, 2, ..., dy be positive real numbers such that a,43-+-a,, = 1. Prove | that | - 1 1 1 ajtate-+a,+n + Speed aA Hi T+q, 1+a) +a, 4 n-variable inequalities 215 5.10. Let n > 2 be a fixed integer. Find the least constant C such that the inequality DY xxi? +37) $ Clr ty t+ x,)* astsjen holds for every x1, ..., x, 2 0. For this constant C, when does equality hold? 5.11, Let n be a positive integer and let x, < xp <+++ 3, then 3 3 Est ag x n + tet = : xgtxdtetat dt atte tx? xPtxdt+etx2 “1-2 5.13. Let a1, az, ..., dn (n 2 3) be positive real numbers such that aa---a, = 1. Prove that 1 1 1 i <1 244) +43 2+a,+a, ———— + n-2+a, +a, 7 5.14, Let x1, X3,..., x, be positive real numbers such that xy +x9+++-tay = 1. For X,41 = X;, prove that » [ish <2-—— 216 Vasile Cirtoaje - Vo Quoc Ba Can - Tran Quoc Anh 5.15. Forn€N, n> 2, q;,b;€R, 1 Si (3) Toit: i=l 3 3. il 5.20. Let a), dz, ..., d, be positive real numbers such that ay4j°+-a, = 1. If O 2a9. Prove that (Sn —1)(a? +a} +++ +42) > 5(ay +a, +++: + aq)” 5.22. Let a), a, ..., dy be positive real numbers belonging to the interval [a, b]. Prove that ay tate tan — (M ae “G, ~\2 = ny/Gay a b where M = 4/—+4/—. b a 5.23. If a,, ap ..., a, ate positive real numbers, then oye eee 218 Vasile Cirtoaje - Vo Quoc Ba Can - Tran Quoc Anh 5.2 Solutions Problem 5.1. If ay, 2, ..., dy (n> 3) are positive real numbers, then where a = ay and dyi1 = ay. (Vasile Cirtoaje) Proof. Applying the Cauchy-Schwarz Inequality, we have 8 i, a ij (Moldova 2007) Proof. For every 1 1a, +32 3(a, +a, +++ +4,). a iv Therefore a, q, an+1+S— Now, adding up the n inequalities for i ity. Equality holds if and only if a, =a) = ay tag+- +a, 3 2, 1 a 3 ,m, we get the desired inequal- =1. a n-variable inequalities 219 Problem 5.3. If x}, x2, ..., X, are positive real numbers such that x} + x3-++++ x? =n, then (Tran Tuan Anh) Proof. For all i=1, 2, ..., n, we have @- 1? 1 (x; - 1? +—=24-0 Tt . *t Xp txt tx Xi xi 22+ Therefore a from which the desired inequality follows. Equality holds if and only if x; — X= x, = 1 o Problem 5.4. Let x, xp, ..., Xp be positive real numbers such that xX)+*-Xq= 1. Prove that 8 a Dtd+xd2 sal Jats. fa =I (Arkady Alt) a’+1_a+1 7 Using this result, as well as the AM-GM Proof. By the Power Mean Inequality, for any a > 0 we have } (a+1)" Qe which is equivalent to a" +1 > 220 Vasile Cirtoaje - Vo Quoc Ba Can - Tran Quoc Anh | Inequality and the given condition x4, = 1, we obtain Dd+xd= ote tt Dot tag] [at S =| f a ist a i lo =) xptn= 7 +1) >—— " x T4n= Dor +> 5 Die +1) 27] []oi+ay = ral fe +x). This is what we want to prove, and so our proof is completed, Note that equality holds if and only if x, =x =-+-= x, =1. o Problem 5.5. Prove that, for all real numbers ay, a3, -.., a, €[1, 2], we have “3 a; + 20:41 where the subscripts are taken modulo n. (Vasile Cirtoaje) Proof. Rewrite the inequality as x i= A441 AG Fa. Ma + 2anp = ° Denote the left hand side of this inequality by L. Then, since x a GjQjny we have (4: — ai41) ina; + 2a;41) G = 41 P2ai41 ~ a;) SF iGi41 (a; + a4 (a; + 2a;41) = Gist Aging Since 1 < a; < 2, we have 2a; ~ a; > 0 for all i=1, 2, ..., n. Hence, L > 0, and equality holds if and only if all a; are equal. Qo n-variable inequalities 221 Problem 5.6. Let a, a5...» dy be positive real numbers such that «343 Prove that 1 1 ai + pret ay 1+ f144n(n—Da, 145/14 4n(n—1)az 1+ f1+4n(n—- 1a, (Vasile Cirtoaje) Proof. We write the inequality as follows 2, ¥1+4n(n—1)aj-1 x 2 2n(n—-1), =D 1 u ET Ss Stns mr By squaring, the inequality becomes 1 | 4n(n—1)] | 1 | 4n(n—-1) [+8] [5 | 2280—o+ ys Y a yj ij istejen UY isi n), and let q = for all i. Assume that x; < have C2) Thee vm—1 SX S15 xy) S++ < x,. By Bernoulli's Inequality, we = raGi+e Fxg tim km + k= nt x2) tet x2) Applying now the Cauchy-Schwarz. Inequality to the m-tuples Cover ke Lee TD) and (12, xeeny cos Xs we get (d+ Hag tm km kn tx beta) > ZO tebe tmnt xe be tty? = (m=ntxy $x ++ 4+x,)% and hence pel Ties [ee +02 ar == ( Equality occurs when a; =a) = (m= n+x, +x) 4+--+x,)3, or (a) Choosing m =n, we get the desired inequality, where equality holds if and only if a; =a, = (b) Since 2 5 Mm=n) +a; ta,+- = a +a2 +4) 2 = ay tay + stan), n-variable inequalities 223 we get 4m"?(m —n) Te +27 a tat tay). (m-1)"? Choosing now m = 2n, we get the desired inequality. Equality holds if and only if a, =a, =---= van Problem 5.8, Let x1, x2, ..., %q be positive real numbers such that x4-+23-++-+ Xn = 1. Prove that i 1-x)0 Asi (Es) , which follows from the Cauchy-Schwarz Inequality. The i=1 i= 1 Proof is completed. Equality holds ifand only if xj =x =-=x,=—. 0 Problem 5.9. Let ay, ag, ..., a, be positive real numbers such that ayy -~ Prove that a l+a, 1+a, (Gabriel Dospinescu) 1 1 1 3n (a; +2 +++++0n) 1- 1- +e+(1- > . (-ra)(-na)O-e) 2 or >— a4 +t 4 Now, using the Cauchy-Schwarz Inequality and the AM-GM Inequality, we get 2 > rath Sety Sinn Maj +n fa i= and thus, it suffices to prove that n-variable inequalities 225, But this is true, since by the AM-GM Inequality, we have (Se) (Be) seller) The proof is completed. It is easy to see that equality holds if and only if a dye A, M Problem 5.10. Let n > 2 be a fixed integer. Find the least constant C such that the inequality Hix 7 +7) S Cle Hg toe tx )t TIE J 1 Asi) 1 ‘ istjsn 2 2 s (=) Equality occurs if and only if x;xj(x? + x}) = xjxj(x? +x} +--+ +x2) for any i(2i—n—1) and n-variable inequalities __227 Problem 5.12. Let x1, x2, ..., Xq be positive real numbers such that x? +23 + s+xt=n. ifn > 3, then x? x3 xg n a5 2, fase gba ye z 2 . AZAAZt tx AFH AGH tHE xPtxzZte-+x2 1-2 (Maths and Youth Magazine) Proof. Denote by S the left hand side of the desired inequality. Setting xo = x,, we have $= Using Hélder’s Inequality, we get st [stoves | [ and thus, it is sufficient to prove that n(n—2)2> [Sa -x2, -»] [Ee -x2, -] : a Because ) '(n — x2. —x?) =n? —2n, and a a 2 2 42 ~Doxke?- a = n 4 x; +X; (ear et) isi i 2 ~ Feb ta this inequality is equivalent to 1 2 n(n - 2)>n?— seh 402%, or SGP +27)? 240. iI cr But this is true, since by the Cauchy-Schwarz Inequality, we have 2 a 1 fet 2 De +#aF 2 5 [Sets “| =4n ‘The proof is completed. It is easy to see that equality holds if and only if x, = xyes =x,el. a 228 Vasile Cirtoaje - Vo Quoc Ba Can - Tran Quoc Anh Problem 5.13. Let aj, a, ..., 4, (n 2 3) be positive real numbers such that 1a9°++Gy = 1. Prove that 1 1 1 ———_ <1, ———_—+ t-+—— n-2+a,+a, n-2+a,+a, n-2+4,+q (Vasile Cirtoaje) First proof. Let r = We can get the desired inequality by summing the following inequalities n-2 ayta,+--+ar n-2+a,+a)~ aj+ay+---+al’ ajta,te+ay aj +ah+---+ar’ a-2 n-2+a,+a,~ aj+a5+- ‘The first inequality is equivalent to (ay + a2)(aj +ah +-+++a7) > (n—2)(aj +43). By the AM-GM Inequality, we have n-2 ay tay +---+al > (n—2)(agaq---a,, Therefore, it suffices to show that a; +a > (aya2)3(a +), or, equivalently, agen Caraad (a* +a") This inequality is true, because we may rewrite it as (0-2?) b-2) 0 Similarly, we can prove that the other inequalities hold. Equality holds if and only if a, = a gS = dy n-variable inequalities 229 Second proof. Since a +aQ n=2+aq+a, | n-2+a,+ay’ one can write the desired inequality as ~ a +a) +1 >2, Fa; taj, +n-2° where @,,41 = a). Using now the Cauchy-Schwarz Inequality, we get (ge) Mea + a4. +2 -2) i=l Wate D) Mla Fala; Fae) 1sizjen 233g; +n(n- 2) a a+ Gin1 4a; +aj4, +n-2~ ‘Therefore, it suffices to prove that DS Ve Fanta; Far) 2 Daj + n(n -2). ist) Jajaisg = a 2» Ya, + n(n -2). iat 230 Vasile Cirtoaje - Vo Quoc Ba Can - Tran Quoc Anh Problem 5.14. Let x1, X2, ..., Xq be positive real numbers suck that xy + x2 + + Xp = 1. For Xpyx = X1, prove that DF <2 Proof. The original inequality can be written as 8 y 3 (x $x - v3? 4x24) = ia Since : 2xeXi41 sittin? +3 = ———" atx t MxF 4x2, txt the inequality is equivalent to ™ +x + x? Glan Xin ‘This is equivalent to n-variable inequalities 231 or Hig V5, yan VP xd +x; Xi t%. «, sg i Since yx? +x?7,; > +=") it is enough to check the following inequality Riis a-% fH (14+ V2) x) + x11 2 which is equivalent to xixiaa (3-2v2 | v2 = xi + 14+V2)xitxin FL 2 20. 2[ (1+ V2) xi +x] The last inequality is obviously true, so our proof is completed. Equality holds if 1 and only ffx; =x =" =x, = 7. o Problem 5.15. Forn €N, n> 2, a;, bj €R, 1 Si ata? + b?B? + 2a;b,AB) a =n (eda Derrawy: i) =n(4? +B?). ct i= fi 232 Vasile Cirtoaje - Vo Quoc Ba Can - Tran Quoc Anh From (A? +B)? < n(A? + B?), it follows that A” + B? < n, so the inequality is proved. a Second proof. Using the same notations as in the first proof, 0 D1 ~ aA ~ b:BY =) 10 + aPA? + b?B? ~ 2a, — 2b,B + 2a;)AB) ial isl 8 ft a 2 =nta? Sa? +B?) 1b? 24) 1a) —2B i+ 2AB Yai, a 7 a iat =n +A? +B? — 24? — 28? +0 =n—(42+B?). and the inequality follows directly from this identity. a Problem 5.16. Let dy, dz, ..., 4, be nonnegative real numbers such that a, +ay+ +a, =n. Prove that ai tas+e-tad-+n? < (n+ 1a? +43 +: +47). (Vasile Cirtoaje) Proof. Since n® = n(a, +a; +++: +4,), the inequality can be written as Ye + 1)a? — na; — a3] = 0, ft or , Sa; = (na; - a?) 2 0. i Notice now that for a; > a; and a; +a; a2 > +++ > aq, we can easily check that a,-12a)-12---2a,—1, and , 2 na, — a? > nay — a} > +--+ 2 na, — az, and so, we can apply Chebyshev’s Inequality to get Ya\= {na} 2) 2 4 bye - »] [Sy «| isi The proof is completed. For n = 2, the original inequality becomes equality, For n > 3, equality occurs when either a, = a) a,, = 1, or one of a; is equal to n and the other ones are equal to 0. o n-variable inequalities 233 Problem 5.17. Let x1, xp, ..., X, be nonnegative real numbers such that x; + Xgtess + X,_ = 2. Prove that 1 1 1. 3n-2 ary a 2 ; xP4+2 0 x42 x242 Proof. The case n = 1 is trivial, so let us consider now the case n > 2. We claim that 1 1 1 > 27 24a+bP fora > 0, b > 0 anda+b <2. Indeed, this inequality is equivalent in succession to +2 b+ 4+a+b? 5 Ate? +b? +2ab 4+ 2a? + 2b? +a7b? ~ 44 2a? + 2b? +4ab" 4+2q? +2b? +4ab 4+a7+b?+2ab 4420? +2b7 40°? ata? +h?” ab(4—ab) 2ab at+b 2 Since ab > 0 and 4—ab>4— ( ) > 3, it suffices to prove that 3(4 4a? +b?) > 2(4+ 2a? + 2b? +.a7b%), or 4207 +b? +20°b, But this is true because a? + b? + 2a°b? < a? +b? +2ab =(a +b)? <4. Now, by using the claim n — 1 times, we obtain, " 1 _ 3n-2 (xp txptentx, P42 6 Equality occurs when either one of x; is equal to 2 and the other ones are equal t0 0, or two of x; are equal to 1 and the other ones are equal to 0. Qo 234 Vasile Cirtoaje - Vo Quoc Ba Can - Tran Quoc Anh n-variable inequalities 235 1 25, then Problem 5.18. Let x1, x2, ..., x, be positive real numbers such that x1 + x2+ Problem 5.19. If xis Xoy +++» ++ bX_ = L. Prove that Xap Xn pve Os xix; (S44). HM (+)" >(3) [Jess jen iI (Gabriel Dospinescu) (lurie Boreico, Marcel Teleucd) Proof. The desired inequality can be written as Proof. Since 2x; > 1 for all i, by Bernoulli’s Inequality, we have fn n ny 2x, \ 2% 4x24 3 “Satz (1-Ex) (2; (+3) 214 St=3 (743) a aa | or equivalently, It follows that n 2x,\2 74\nt 3 oa) e(e Bante) LG) = (3) (+3) On the other hand, by the Cauchy-Schwarz Inequality and the AM-GM Inequal- ity, Iler3 Combining this with the above inequality, we get the desired result. Equality 1 Oo holds if and only if x4 = Problem 5.20. Let a), ay, ..., dq be positive real numbers such that a,a3-+ 1.ifO0(2+p)a;”, and hence (1+ pay)? =1+ pay(2+ pay)>1+p(2+p)a,*" , or 1 (1+paq)? >1+—, qx) ‘ _ my where q = ——— 2>n-1and x, = *? Therefore, it suffices to prove 1 p@+p) 7 , that x x Xa n pa mW l+qx,; 1+ qx 1+ qx, 1+q for q 2 n—1 and x1x9-++xq = 1. This is equivalent to the well-known inequality (by Vasile Cirtoaje) 1 fh 1 ue 1 n l+qx, 1+qxz 1+qx, ~ 14 (Qa ntU)xy + (qt Dlaz + +.x7") > nga} which follows from the weighed AM-GM Inequality as follows ‘ latte Gye ngx!. This completes the proof. Equality holds if and only if a, = ay = ++» =a, a a Remark. Similarly, we can prove the following more general inequality (by Vasile Cirtoaje) 1 1 ooo tao tt ee —, (+pa)y" (1 +pay) (1+ pany ~ (+p n-variable inequalities 237 =1Lkisa where a}, az, ..., dq are positive real numbers such that aya) positive integer and 0

2a. Prove that (Sn = 1)(a} +a} +++: +02) > 5S(ay tay +++ $4,) (Vasile Cirtoaje) Proof. Let a; = kay, k = 2. By the Cauchy-Schwarz Inequality, we have 3 k+ Lay +43 +++++a_]? @iadde bata Ql 4 tial dal gp. f ges Met Dae tart tan)? Gta i pas x (k+1)? ‘er tt? _ (tay te tay)? -Ope tay mate? Therefore, it suffices to show that Sn- 2k = tn-1 Sei." ‘This inequality is equivalent to (k — 2)(2k — 1) > 0, which is obviously true for 3a, +a. +a,)" k > 2. Equality holds if and only if Sa} +02 +----+a2 = Sess ay 24n- 6a3 _ that is, if and only if ay = 2ay = = Problem 5.22. Let a1, az, ..., a, be positive real numbers belonging to the interval [a, b]. Prove that ay+ag+ M\2-= zy ns/@ay [a 6 i where M = (Vasile Cirtoaje) 238 Vasile Cirtoaje - Vo Quoc Ba Can - Tran Quoc Anh Proof. By Lemma below, it suffices to prove the desired inequality for a, = a2 = = ay =a and y44 = Ogyy = ++ =a, = b, where k € {1, 2, ...,n—1}. The inequality becomes atb\2 2 na pit kat (n= Wb ; ate —. k-1 n k-1 ka+(n—k)b — id ¢ = ——— patqb+e p+q+1 Setting p = , We see that p 2 0, 1s P P n 2 q20,ptq=1- i anda+b= . Therefore, we can rewrite the inequality in the form +qb+c\Ptatt ao zaPbtc, ptaq+l which immediately follows by the weighted AM-GM Inequality. Thus, the proof is completed. For n = 2, equality holds when a; = a and a = b. Lemma. Let a), az, ..., dy be positive real numbers belonging to the interval [a, b). The expression is maximal when aj, dy, ..., dn € {a, b}. Proof, For the sake of contradiction, assume that E is maximal when one of a1, dy, ..., @y is different from a and b. Consider a < a; < b, and let _ ay tagte tay Fla) =" : From hypothesis, we have f(a,) > f(a) and f (a,) > f(b). Since f(a,) 2 f(a) is equivalent to a (vq - Ya) (F- a2 ~+++—a,) 20, where F = yams [( ya)" (vai)? (a) +--+ (va) ], we get Fa, tetany. Similarly, from f (a;) = f(b), we get ag +++ +a,26, where - 6 = Vbay | (ym) "2+ (ym)"™ (Vo) ++ (Ve) |: It follows that F > G, which is clearly false. Thus, Lemma is proved. oa n-variable inequalities 239 Problem 5.23. If a1, a... dy are positive real numbers, then ” (ne? } (32) sear at S aa; isi1" COSC aja; Sa " 2,2 Day 1si VB(a\ay +0203 +++ +.aq)). (Vo Quoc Ba Can) ee 240 Vasile Cirtoaje - Vo Quoc Ba Can - Tran Quoc Anh Proof. First, notice that _ (=1Gx+D v3x - Yax+1= m4 JETT? for any 0 < x $1. From the AM-GM Inequality, we have 24? ata? Beg a@t+ay ajt+a ata’ ate 253 @, 42 + paz +++++a_q, S z =a?tapte+a7=1 This allows us to apply the above inequality to get V3(aya2 + andy + +++ + ayy) S 1 + 2(ayag + agag +--+ anay) a? +b, 6.2. Let a and b be positive real numbers such that a + b = 2. Prove that ai +b) >2, 6.3. Ifa, b,c € [0, 1], then 2 2 3 Q-a+aby¥+(1~b+ beh +(e +ca)?2 5. 6.4, Leta, b, c be nonnegative real numbers, no two of which are zero. Prove that b+e c+a a+b 2a? + be 2b? +ca 2c? +ab 241 a rs Oa 242 Vasile Cirtoaje - Vo Quoc Ba Can - Tran Quoc Anh 6.5. Let a, b, c be positive real numbers. Prove that a* + ot + cf _atbte a +b3 © b+ © 3 403 20° 6.6. Let a, b, ¢ be positive real numbers. Prove that ab+be+ca 2a = 2b ig 2c 58 Vath Vore cta” V a?+b?+c? 6.7. Prove that for any positive real numbers a, b, c, the following inequality holds (a? — be) Va? + 4bc + (b? — ca) Vb? + 4ca + (c? — ab) Vc? +4ab 2 0. 6.8. Prove that if x, y, 2 are positive real numbers, then y Fe 2x —-32v2 -3 rye a(E (= 6.9. Prove that if a, b, c are positive real numbers such that abc = 1, then 1 1 1 1 1 i + + —— — l#bt+e | l+ct+a’ ltatb 24a 2+b 6.10. Let a, b, ¢ be positive real numbers. Prove that 1D ase aa ateee(i+ Yatb+or 6.11. Let a, b, c be nonnegative real numbers, no two of which are zero. Prove that 1 1 1 1 (a+ 2b)? + (b+ 2c)? ab+be+ca™ More difficult inequalities 243 6.12. Let a, b, c be nonnegative real numbers, no two of which are zero. Prove that afar+be. 4 afbitea | afch+ab | Vat eta? a+b? 6.13. Let a, b, ¢ be positive real numbers. Prove that (aaa VB y7aber aie VST 7abera = 6.14. Let a, b, c be positive real numbers such that abc = 1. Prove that, for any k2 0, we have be ca @+k b+k 6.15. Let a, b, ¢ be nonnegative real numbers such that a+b + ¢ = 6. Prove that —4< a°b+b'c+4ea? — Sabe < 128. 6.16. Let a, b, c be nonnegative real numbers satisfying ab*+bc? +ca? +abe = 4 VA. Prove that at tba tc4 3 6.17. Let a, b, c be positive real numbers. Prove that for any k > 2, we have Vive? +kab +0? < 41a? +(3k +2) Diab. 6.18. Let x, y, z be positive real numbers such that xyz = 8. Prove that 2 e y _ peepee 8 DOF+1) VOF+DESHD YS+DG+Hn 3 244 Vasile Cirtoaje - Vo Quoc Ba Can - Tran Quoc Anh 6.19. Let a, b, c be nonnegative real numbers. Prove that b co atb+e 2b? +ca © 2c? +ab +b? +2" 6.20. Let a, b,c be nonnegative real numbers, no two of them are zero. Prove that be ca bee * Vera * Vae5 (b+c? epait 6,21. Leta, b,c be positive real numbers, Prove that a + bt A a orbte @+abe+b3 ” be +abe+ tabe+a3 ~ a? +b? +c?" 6.22. Let a, b, c be positive real numbers. Prove that abc. Aab+be+ca) boc'a a+b? 4c? 6.23. Let a, b, c be nonnegative real numbers satisfying ab + be +ca = 1, Prove that 6.24. Let a, b, c be positive real numbers such that ab-+be-+ca = 3, Prove that, for any nonnegative real number k, the following inequality holds 1 3 aon tao tooo (a+kb) © (b+kc> * (c+ka)? ~ (k+ 18 6.25. Let a, b,c be positive real numbers such that abc = 1. If k > 0, then 7 +—1 — <1 ltatbk 14b+ck "1+c+at > More difficult inequalities _ _ 245 6.26. Let a, b, ¢ be positive real numbers. Prove that b fe fra V3 [Ca +b) b+ ee +a) Vorat? arb Voee* 4 at+btc . 6.27. Let a, b, c be positive real numbers such that a + b + ¢ = 3. Prove that, if k>0, then +1 ,fab+1 (+o ire +@ afore a+b) 6.28. Let a, b,c be positive real numbers. Prove that er ee ee ae 2 Grey * Grae * reps 6.29. Let a, b, c be nonnegative real numbers, no two of which are zero. Prove that vab vbe Vea 9 a+b 8+ ab+e* beta? cate 2 (a+b+ce)4" 6.30. Let a, b, ¢ be positive real numbers. Prove that {ort be + brtea | c+ab 10 3Vabe Yat+e)* Veta) * {cate VO arbre 6.31. Let a, b, c be nonnegative real numbers, no two of which are zero. Prove a b ¢ s4f/1 iy ) we Peet Bees lbtc ota tate 6.32. Let a, b, c be nonnegative real numbers such that a+b +c = 3. If k > 0, then ghtipk k akb+bke +e! asmm{3, ear}: 246 Vasile Cirtoaje - Vo Quoc Ba Can - Tran Quoc Anh 6.33. Let a, b, c, d be positive real numbers such that abcd = 1. Prove that 1 1 1 1 + a zai. Groat *G+Ha+e *Groas) *GsaGe 6.34. Ifa, b, c, d are nonnegative real numbers, then a* + b4 +c4+4d*+ 2abed > a*b* + a7c? +.a7d? + b?c? + b'd? + 07d”, 6.35. Let a, b, c, d be positive real numbers. Prove that 2 b 2 2 d 2 of aa) + (peta) +e) + (4) 28 atbt+c b+c+d ct+d+a d+at+b 9 6.36. Let a, b, c, d, e be positive real numbers. Prove that (+552) (aia) (aie) (253) (as) =) 6.37. Let a;, a2, ...,@, and by, ba, ..., by be positive real numbers. Prove that Vv 2 dob | 244 SY aay} | SY bb; mA iA isizjsn isisjen 6.38. For all positive real numbers a}, a2, ..., dy, the following inequality holds 6.39. Let x1, xp, . n> 3, then More difficult inequalities 6.2 Solutions Problem 6.1. [fa and b are positive real numbers, then a? + 578 > g2) 4 p24, Proof. Since the inequality is trivial for a = b, without loss of generality, assume 2landO b. We have two cases to consider: Case 1. a > 1, From a¢-®) > 520-5), we get 528 > Therefore, 25 p20 a?8 45?) — a? p> gt az a because a** > a?? and a > 574, a Case 2. 0< b d, p> 1), the inequality becomes co} — dP > To prove this inequality, we will show that 0? — dP > p(ed)P-'(c? — d2) > c? — a? The left inequality is homogeneous in c and d, and hence one may consider d = 1. Thus, we have to show that f(c) > 0 for ¢ > 1, where F(c) =e? = 1 = p(cP*! — cP“), We have f“(c) = pe?~2g(c), where B(C) = 2cP*1 — (p41)? 4 p—1 Since g/(c) = 2(p + 1)c(c?“! — 1) > 0, the function g(c) is strictly increasing for ¢ 2 1, hence g(c) > (1) =0 fore > 1, and f(c) > 0 for c > 1. Therefore, f(c) is strictly increasing for ¢ > 1, and f(c) > (1) =0 fore > 1. With regard to the right inequality above, we note that it is equivalent to each of the following inequalities pled)! > 1, (ab)? > & a 1ta-b, - Toaap na nb. 248 Vasile Cirtoaje - Vo Quoc Ba Can - Tran Quoc Anh We need to show that g(b) > 0, where 1 -—b +8 ng —Ind. 80)= Tate If g'(b) <0 forall b €(0, a), then the function f(b) is strictly decreasing, and hence g(b) > g(a) =0. Since 2 1 j= -_-— nes, 8O)=— Grae tb g’(b) < Oif h(a) > 0, where 1-a+b)? hla) =2Ina+ Ea Since 2_ 2-a+b) _ 2a-ba-V) 4 k@=—> b ab for 0 < b h(1) = b > 0. This completes the proof. Equality occurs if and only ifa=b. 0 Remark, Actually, for any positive numbers a, b and k < e, the inequality holds ak 4 bk > ak? + pi, Problem 6.2. Let a and b be positive real numbers such that a+b = 2. Prove that as+bs>2. (Vasile Cirtoaje) Proof. Without loss of generality, we may assume that a > 1 > b. Let us con- sider 2 cases T 34+ Vv21 The first case is when b > — =. In this case, we have —T7— Sa, bs 25-21 c Sa Using Lemma below, we get 7 a? 23a-2- (a1), and 7 23b-2-3(b-1° More difficult inequalities 249 Therefore ab +b 2 3(a+6)—4— 21a 19 +(6- 09] =2. 34721 The second case is when b < . In this case, we have 34+ V21 2-b>2- >14, and so, we deduce that 203. 3 a ac +b >as > 1.411 >2, The proof is completed. Equality holds if and only if a= 3+v21 25- J21 0, ie. xy 0, where 3 F(x) =F inx~ In(3— 15x + 21x? — 7x3) +In2. 3 273 = 15x + 21x §(x) =3— 15x + 26x? ~ 21x3 + 7x4 — (3 — 15x +21x? —7x3)Inx, We have f(x) = (x), where ‘, (x — 1)(28x3 — 28x? + 3x +3 gG@= eave meets) +3(7x? — 14x +5)Inx. +V21 25-21 <2 8 3 Because 7x? — 14x +5 <0 for , one may use the well-known inequality Inx 0 to get , (x = 1)(28x3 — 28x? + 3x +3) g(x)= Se ae eee) +3(7x? — 14x +5)(x — 1) x _ @~1)(49x? — 21x -3) 7 2 250 Vasile Cirtoaje - Vo Quoc Ba Can - Tran Quoc Anh More difficult inequalities _ 251 Since La W 4, oye. 4s Ate “Sd Now, let us prove the original inequality, Due to cyclicity, we may of course mee yWe lave 49x" — 214 —~3 3 0, ie. a) assume that b > a. From the Cauchy-Schwarz Inequality, we have 0 and hence, g(x) is increasing. Moreover, it is clear that g(1) = 0. Thus, 34+V21 7 24 72! a: g(x) <0 for 0 fort < es BOY ans (=a)? +67][- b+ be + (1c +ca)"] 2 2 shows that f (x) is strictly decreasing for x € [xp, 1] and strictly increasing for 2 [(1—a)(1— b + bc) + (1 —¢ +ca)] 25- V21 - =(1-a+aby xe | 1, —T]— | . According to this result, we find that f(x) > f(1) = 0 for 25—V21 . On the other hand, from the above lemma, we get any x € | x9, === |. This completes the proof of the lemma. a Problem 6.3. Ifa, b,c €[0, 1], then 3 (1-a+ab)?+(1—b + be)? + G-e+ea¥ 2 5 First proof. To solve this problem, we will need the following auxiliary lemma Lemma. If 1 = b>a>0, then ; 2 b?4+(-a) <2[1- Yaa—2)] Proof. After expanding, we can rewrite the inequality as a? +b? -2a+1< 2+ 2a-2ab- 4ya(1 — b), which is equivalent to 2[a+c-b)-2Vaa—8)| >a? +b? +2ab—2a-2b+1 This can be written as 2(va-Vi-b)'>(a+5-1F Since (V@— V1=b)” < 2(a +1~ 6), we have (atb-1=(Va-vi-8) (va+ vi-8)” <2(va- 1-8) (+15) <2(va- vi-b) This ends the proof of our lemma. (1a)? +6? s2[1- Yaaro]. Combining these two inequalities, we obtain 1-a+ab)? (1-b+ be)? +(1-c+ca}? > —_ ee _, 2 [a - yall 5] and thus, we see that it suffices to prove that (1-a+aby 2[1- vaa- BD) (’-a+ab}?+ — b+1-b 1 Setting x = y/a(1— 6) (note that x = y/b1 = 6) s —5— = 5), it becomes (1x22 OaaF 2 21-x)?~ 2 =x?) + which is equivalent to 20 -x?P +(1 4x)? >3. This is true, since by the Cauchy-Schwarz Inequality, we hi (24 -x?)+14x]? [3+x(1 21-2? P+ (+x > a Oe 241 3 The proof is completed. It is easy to see that equality holds if and only if a= b= Land ¢= 5, or any cyclic permutation. a 252 Vasile Cirtoaje - Vo Quoc Ba Can - Tran Quoc Anh Second proof. Let t = a(1—b)+b(1—c)+c(1—a) = a(1—c)+b(1—a)+c(1—b). Then, we have Da - a tab? = 711 - 2a(1 ~ 6) +02(1 — by") = 3 - 2 +e =3-20+ [Sati »)| -2d,a00 — p10) 23-2+2-2} alc) (==) 2 1 5 =3-2040— 5S la —c git? Therefore, it suffices to prove that 8 tn - 3- gtte > yo equivalently, (1—r)(3—2r)>0. ‘This is true because t=a+b+c—ab~be~ca=1-abe—(1—a)(1—b)(1—c) <1. The proof is completed. o Problem 6.4. Let a, b, c be nonnegative real numbers, no two of which are zero. Prove that b+e c+a a+b 6 so ts = — 2a*+be 2b?+ca © 2c? +ab~ a+bt+e (Vasile Cirtoaje) Proof. Firstly, we will prove the following auxiliary lemma Lemma, Ifa > b >c > 0, then (a+b? +e(vat vb)” 2ab(a +b) + cla? + b?) 2(a+b+2c) (a+ bYatb+c)’ Proof The inequality is equivalent to (a+b? +e(va+Vvb)” — 4ab(a+b)+2c(a? +52) atb+2 (a+b)at+b+c) ’ or 2 2: 2 2 +p — 10b(a+ b) +2c(a? +b?) path +e(va+ vb) | (a+b)a+b +e) a+b+2c More difficult inequalities 253 This can be simplified to (a—b*(a+b~c) , ¢(va- vB)" (a+b)a+b+c) at+b+2c which is true because (a—b)? = (a+ ¥b)* (Va vB)” > (a+b) (va- vB)", ¥ L a+b-c2c,and —_ = ———.. at+tb+c” a+b+2c Now, let us prove the original inequality: Due to symmetry, we can of course assume that a > b > c. Then, by the Cauchy-Schwarz Inequality, we have b a (b+a? (a+b? Da? tbe * 2 tea = Daa +e) +a2b™+ea) ~ 2adla t+ b)-+e(ae +b)’ and c c e(vb+ va) e(va+vb)" 2a + be * Db tea ~ Ba + be) Falab > +ea) 2ablatb)+e(a2 +b) Adding these two inequalities and then using the above lemma, we get bre | cha. (a+b) te (vat vb)’ a+b +2c) / 2a? +bc 2b?+ca~ 2ab(a+b)+c(a? +b?) ~ (a+b)atb+c) On the other hand, from the AM-GM Inequality, we have a+b ath 2c? +ab ~ +b)" gpg GED ‘Thus, it is sufficient to prove that 2Aa+b+2c) atb 6 (a+b)(a+b+c) a+b)? ~at+bt+c 2+ o) Letting ¢ = k(a + 6), it becomes le ee (a+ bP(k+1) (@+b\K +1) 1 2 (ope 4 > (a+b) ( "5 or equivalently, A2k+1) 4 k+l Bk? + 254 _ Vasile Cirtoaje - Vo Quoc Ba Can - Tran Quoc Anh After some simple computations, we can factorize 22k +1. 4 6 8k(2k - 1)? ¢ dy ( k+1 "B41 k+1~ (k4+1(841)’ which is obviously nonnegative, so that the last inequality is true. This completes our proof. It is easy to see that equality holds if and only if the triple (a, b, c) equals (t, ¢, ©), or (r, ¢, 0), or (0, ¢, ¢), or (¢, 0, £), where ¢ is an arbitrary positive real number. a Problem 6.5. Let a, b, ¢ be positive real numbers. Prove that ab 5. oF at+bte G+ Bre Sta 2 (Vasile Cirtoaje) First proof. Multiplying each side of the desired inequality by a3 + b3 +c3, we can rewrite it as UAE eh (D9) (Le) From the Cauchy-Schwarz Inequality, we find that ae eel Dette yes Now, notice that for any positive real numbers x, y, the AM-GM Inequality gives x us — + 2 > x. According to this inequality, we have y Vee 7 Eee! +0), -—— Sea? - pee +b), and hence, it suffices to prove that Lets Dee 7 Tae +o92 F(Ee) (Le). More difficult inequalities 255 After some simple computations, we can rewrite it as 2D at +45 076? > 3 abla? +52), or equivalently, Silat + b4 +4076? - 3ab(a? + b2)] > 0, which is true because at + b4 + 4a7b? - 3ab(a? + b?) = (a? — ab + b*)(a — bY > 0. The proof is completed. Note that equality holds if and only ifa=b=c. 0 ab3 a+ b8 x ab? a+bte a? + b> 2 Now, using the AM-GM Inequality, we get a +b? = V (a + 65) + 20963 > y/ 2y/2a5b3(a5 + 65) = ¥/Ba3b3(a5 + 65), and thus, it follows that ¥ ab? ——_ ab? Ly Vee a +63 /ea3b3(a5 + b5) vw ae On the other hand, the Cauchy-Schwarz Inequality and Cirtoaje’s Inequality (see problem 2.52) gives us that Tver ee] < (Liver) (S a 3 (D2) [Sie Combining this with the above inequality, we get abs a bs Lore 75 (h9) (Lape Second proof. Since — 5) one may write the desired inequal- a ity as 256 Vasile Cirtoaje - Vo Quoc Ba Can - Tran Quoc Anh More difficult inequalities 257 In order to prove the desired inequality, we see that it suffices to prove that 3 Ly ware We can prove this using the Cauchy-Schwarz Inequality as follows 2 »s (= | = [Lier+<5] [= Ta _ A(aS + b° +c8)(aSbS + boe8 + c8a8 (a6 + b9)(b° + c®)(c8 + a®) where the last inequality is valid according to the well-known inequality (x + 9 ytelxytystex)s gn +y)(y +2)(@ +x). The proof is completed. O Remark. In the same manner with the first solution, we can also prove that the inequality att) pet cH gt hte kT ae ak+ pk pkack ck+a 2 holds for k= 2+ ¥3 and a, b,c > 0. Problem 6.6. Let a, b, c be positive real numbers. Prove that 2a 2b # 2c 33 ab+be+ca a+b Vb+e cta~V at4be4c?" (Nguyen Van Thach) Proof. Since 2a? Peet a +c? +e Lax yi ae +h ate =) ate +Ue-9-)) atc’ and sEe- (50) (EA) --de- (oy (OF) a ie _ wo _abla- by =liarours =” J it follows that oe Ma >= atc Combining this with Hélder’s Inequality, we get 2 Be = (xVa) [(Mata+oye+e] 2 ; 2 22(E) (= 5) [Data +sya+0)] 22(De), hence it suffices to prove that 4(Sia)’ 2 27(Slab) [Tatar oya+o], or equivalently, 4 (Xe)? 227 (Stas) [(e) (Xe) +3abc] : Now, assume (without loss of generality) that a+b +c = 1, and furthermore put q= ab + be +ca. The inequality becomes 42> 27q(1-2q+3abe). From (a+b +c)? > 3(ab + be + ca) and (ab-+ be +ca)* > 3abc(a +b +c), we getO 3abe. Therefore, 4—27q(1 —2q + 3abc) = 4- 27q(1 — 29 +q*) =(4-3q)(1 - 34)? = 0, which proves our inequality, Note that equality holds if and only if a = b = c. Oo Problem 6.7. Prove that for any positive real numbers a, b, c, the following in- equality holds (a? — be) Va? + 4be + (b? — ca) Vb? +.4ca + (c? — ab) Vc? + 4ab > 0. (Vasile Cirtoaje) ee 258 Vasile Cirtoaje - Vo Quoc Ba Can - Tran Quoc Anh Proof. Without loss of generality, assume that a > b > c. We have (=) (o5-)- NE and Va3 +4abe > V b3 + 4abe. Since — be) Va? + 4be + (b? — ca) Vb? + 4ca = ) Va +4abe + (° b -=) V8 +4abe, vb we can apply Chebyshev’s Inequality to get (a? — bc) Va? +.4bc + (b? — ca) Vb? +.4ca > be ca (avarnve-% =) (Va 4abe + V0? +4abe (aya+bvb) (Vab—c) ( 3 +4abc + Vb? + 4abc) 2Vab Since aVa+bvb > 2VabVab, Vab—c =O and Va? +4abe+ Vb? +4abe > 2VabVab + 4cVab, it follows that (a?— be) Va? + 4b +(b2—ca) VB? +4ca > 2V/ab ( Vab—c) Vab+acVab, and hence, it is enough to check the following inequality avab (Vasc) yab+4cVab + (c?- ab) Vc? +4ab > 0, which is equivalent to (vab-e) [avo +4abe ab — (c+ Vab) Vera >0. This is true because 4 («8 +4abeVab) 7 (< + vee) (e +4ab) = =c (Vab ~c) (2ab + 2¢Vab +e? Jee Our proof is completed. Note that equality holds if and only if a= More difficult inequalities 259 Problem 6.8. Prove that if x, y, 2 are positive real numbers, then F z Sy -322/ Ny -3). 2x ytz 1 y= and Y= [arbi re" 2b4c4 DY oe 3242) abr *|" or equivalently, Ya Vea WEY ER —9(2- vA are bite For any positive real numbers m, n, we have (Vasile Cirtoaje) Proof. Setting the inequality becomes Ss 23459 LR 2 = Vm@an_™ +2(V2-1)mn+n _2(2 A) (mn- Vn ve) mtn m+n _ 2(2-Vv2)mn 2mn m+n m+n+ ym? +n? 2(2- v2) mn 2mn a aS 7) m+n 7 mtn+ v2 =0. According to this inequality, we obtain x sores re Let 42 (Y=) whe te!) bP +c? = D1 2c*(b? +c) - 2 (2- v2) abc? On the other hand, the AM-GM Inequality gives us 22>) Thus, we only need to prove that 7e7(b?-+c)-2 (2- V2) abc?) b? +c b*c* Vote Pre 22>) b%c9—3 (2 - v2) a?b%e?, 260 Vasile Cirtoaje - Vo Quoc Ba Can - Tran Quoc Anh which is equivalent to 2 2.27.2 Pi: 3.3 ‘pei 2 R262 @ Did?e2(b? +2) 29 b%c? > (2- v2) a?b?e (Sate Because | bc?(b? +c?) 2) b%c3 = )b%e(b —c)* and s @ yee (b? 02)? x (a? +b? +02)(b? 2)? b? +c? (a? + b?)(a? + c2) (a? + b7)(b? +c? )(c? +a?) it is sufficient to check that Sqb?c*(b —c)? + S,c7a?(c — a)? +S.ab*(a — b)? > 0, where (2— v2) a2(a? +b? +.07)(b +e (a? + B2)(b? + c2)(c? +a) and $), S, are similar, Without loss of generality, we may assume that a > b >. Then, it is easy to check that S, < S, = it suffices to prove that iF +b? +c*)laX(b+cP+b%(c+a)?] _ PO EFI ENE +a) z Applying the Cauchy-Schwarz Inequality, we have (b+¢)? < 2 (0 (+ +3¢ 2) and 3 (atcP< 3 3(« + 3°) . Therefore 5(a? + b? +c?) [# (+ 3.2) +52 (a24 32 gg A Nh) ~ 3(a? + b?)(b? + c7)(c2 + a?) _ 10 Sc? [(a? — 6?) +c*(a? + b?)] 3 6a? + BAG FCA +02) ~ 3 From S, +5, > 0 and S, > Sz, we have S, > Sy > 0. Using this and the obvious inequality a2c*(a ~c)? > b*c?(b —c)?, we get Yis.07e2(b-c)? > 54b2c*(b—cP°+8yc2a%(c—a}? = (S, +5, )b2c2(b—c)? = 0, as desired. Note that equality holds if and only if x = a More difficult inequalities 261 Problem 6.9. Prove that if a, b, ¢ are positive real numbers such that abc = 1, et 1 1 1 1 14 + + = + + a T+bt+e *Iteta1T+atb~ 24a 246 246 (Bulgaria 1997) First proof. From 2 b 1 a(b 1)? a+2 ab+b+1 a+b+1 = @saab+b+ arb” 1 b 1 2igea Daeaei* Uae b since 7 Spa bai 7 b this inequality yields 1 1 1 ate aS | SS ae Dares? Dare and thus, it suffices to prove that 1 1 1 12 be hay -at2 b+2 ct2 or equivalently, flay ss a+2° b+2°>c+2~ ~ By the AM-GM Inequality, we have a @2l3 a3 a a+2> at2abe}— a2/3 + 2(be)V3 ~ a2/3 + BAF Adding this and the two analogous inequalities, we get the desired result. It is easy to see that equality holds if and only ifa=b =¢=1 o Second proof. Without loss of generality, we may assume that a > b > c, From the hypothesis abc = 1, we have a > 1. Now, we write the inequality as 1 1) (2 1 i, 1 c+2 atb+l b+2 atc+1) at2~ btc+1 or equivalently, 1 b+c4+1" a+b-c-1 . atc—b-1 (c+2)a+b+1) (b+2)ate+1) Iv 4 2 262 Vasile Cirtoaje - Vo Quoc Ba Can - Tran Quoc Anh Since a2 6 2c anda>1,wehavea+b—c—1>a+c—b—1, and 1 1 2 (a~-1)b~0) Oe (c+2)a+b+1) (b+2)a+e+1) FAct2H@+b+iNates] =?) From this, applying Chebyshev’s Inequality together with the Cauchy-Schwarz Inequality, we get at+b-c-1 atc—b-1 (c+2)a+b+1) (b+2)ate+1)~ 1 1 = (a-1) | ———___,____4 =a fem (b eexsau| 5 4(a-1) * (c+ 2Ma+b+1)+(b+2\ate+1) : 4(a-1) © (a+3\b+e41)+3a42be+1 Thus, it suffices to prove that Aa 1)(b+e+1) (a+3)(b+e+41)+3a+2be+1 or equivalently, 4(a-1) btetl 3a+2b = a+34 c+] a+2 b+e41 Since b+c +13 2Vbce+1 anda > 1, it is enough to show that 4(a-1) 4 2vbE+1 , 3 = ag 4 et 2be+T "a F2 2Vbe+1 . 1 Setting t = Vbe <1, then a = — and it becomes 1 4(5-1 (3 Jaren 2t+1 2 1 3 1 (ara)ens Sear at > 1. This can be written as 20-241) | e(2e4+1) (28 FFE At GAG i) i More difficult inequalities 263 or (1-02? +241) 241 (t+2)(c3 + e241) Since 1 > t > 0, it is equivalent to 21 + fet 1(2e? +1) > (e+ 2 +02 + 122 +04 DY. This is true because 2t?+12> 3+ t? +1, and 21 + e)(2e+ 1) -—(¢ +2)(2e? +e 41) = e(1 - t)(2t +3) >0. The proof is completed a Problem 6.10. Let a, b, c be positive real numbers. Prove that (atb+e) (44442) 594 ar? ee OT OTT ETS 2arb+ oe 3 2 (Vo Quoc Ba Can) Proof. Without loss of generality, we may assume that a > b > c. Then, we have two cases Case I. a+c 2 2b, Since (ac)? = (a +c}? — 4ac and 1 :) (a+ b+oate) | a+btc 1 +b+e)(—+— —, (a+b o(z+z+t ae D we can write the inequality as (a+b+cYa+c) 54ac ate. 27(a +c)? ac (atb+eh” b * Xatbtae” By the AM-GM Inequality, we have (atb+eate) | S4ac > 6,/ Sat) ac (a+b+c? ~ Va+b+c’ and then, it suffices to prove that é CG CsLerw 27(a +c)? atb+c b ~" * 2(a+b+c)?" ate Setting +7 2t (¢ 2 1), it becomes 3t 277 6 +t244—— 2t+1 (2e+1)? 264 Vasile Cirtoaje - Vo Quoc Ba Can - Tran Quoc Anh 3t 3t 6t Since = —————— > ——, it is enough to check that 2+1 °° f3e(arz1) Stl 36 ayy 2707 ekg a 5St+1 ~ (2e+1)’ which is equivalent to (2007 = 7t —4)le (Se + 1)(2t +1)? Because t > 1, we have 20t? — 7t — 4 > 0 and hence, the above inequality is true, Case 2. 2b > a +c. In this case, applying the AM-GM Inequality, we get atbso(Z+542)-9= “el +2) e405 = a -8 2a+o/( a+) +2 +0 °( 2(va~ ve)? _(a- oe) a= oF ac vac ac Vac (fat fc)” 5 Ma=cF , a~c? 6(a—c)? 2 are? te? ate On the other hand, it is clear that 27(a-c)* 2xa-cP —_6(a~c)* Aatb+cr— ate 2 (ater 2(at—-te Combining these results, the conclusion follows. Equality holds if and only if a=b=c. ao Problem 6.11. Let a, b,c be nonnegative real numbers, no two of which are zero. Prove that 1 1 1 1 —— + + Ss 2 (a+2b)? © (b+2c)? © (c+2a)? ~ ab+be+ca (Pham Kim Hung) More difficult inequalities 265 First proof. ‘To prove this hard inequality, we need to consider the following two cases Case 1. 4(ab+be+ca) > a?+b? +c. In this case, applying the Cauchy-Schwarz Inequality, we get 9(S9)" x (a+2b)? ~ di@+20"a +2)?’ and henee, it suffices to prove that 9 (Xa)? (Sav) = Vat 26y%(a +207 Since Sa + 2b)%(a +20)? = (Xe)" +18 (Xav)’, this inequality can be ™“ 9 (Xa) (Sas) = (Xa)'+ 18 (Ma), (Xe? Yad) (4 Diab - Yee’) zo which is true according to the hypothesis 4(ab + be +ca) > a? +b? +c? and the well-known inequality a? + b? +c? > ab+be+ca. Case 2. a? +b?+c? > 4(ab+be+ca). Without loss of generality, we may assume that a=max{a, b, c}. Since or a(a—2b— 2c) =a? +b? +c? —4(ab + be +ca) + b(a—b)+c(a—c)+4be > 0, we get a > 2(b +c). Now, using the AM-GM Inequality, we have 1 1 2 aos too 2 oo’ (a+2b) ” (b+ 2c)? ~ (a +2b)(b + 2c) and thus, we see that it is enough to prove that the stronger inequality holds 2 1 ee (a+2b)(b+2c) ~ ab+be+ca’ or equivalently, b(a— 2b — 2c) (a+2b)\(b + 2c\ab + be tea) =" Because a > 2(b +c), this is clearly true. The proof is completed. Note that equality holds if and only ifa=b =c. a RB 266 Vasile Cirtoaje - Vo Quoc Ba Can - Tran Quoc Anh Second proof. Assume that a = max{a, b, c} and consider two cases Case 1. a < 3b +c. Settinga+2b=x+y,b+2c=y+zande+2a=zt+x yields gen tbe. iy yal ttere og : 2 ° 2 ° 2 a and Sxoyt22 4, Symz+2x 9° 9? Then, the inequality may be expressed as 1 1 1 27 — + SS + SS SS. (e+yP? "te? "Gtx? > x24 y2 4224 (xy t+ yetzx) Using the well-known Iran Inequality, along with the usual inequality x2 + y?-+ 2? >xy +yz+2x, we get roy ya, 9 (ety? (ytzP (@4+x)? 7 4xy t+ ys +sx) 27 — x? + y? +224 11 (xy + yz +2x) Case 2. a > 3b +c. Since c+ 2a < 3a < 3(2b +a), we have 1 1 — (c+2a)*~ 9a+2b)* Therefore, it suffices to show that 10 i 1 pa apnea a oliage ne Oa+2b)? (b+2c)* ~ ab+be+ca By the AM-GM Inequality, we get woo, 2v10 2.1 —~ +> Fe SO oS 9a+2b)* (b+2c)? ~ 3(a+2b)(b+2c) ~ (a+2b)(b + 2c) Thus, it is enough to prove that 2a s 1 (a+2b)(b+2c) ~ ab+be+ca’ that is, in succession, 2.1(ab + be +ca) 2 (a + 2b)(b + 2c), More difficult inequalities 267 a(1.1b+0.1c) > 2b? + 1.9be ‘We have a(1.1b + 0.1c) — 2b? - 1.9be > (3b + c)(1.1b + 0.1c) — 2b? — 2be 13b?—6be+c? _ 4b? +(3b—c)? | ~ 10 - 10 which completes the proof. a Problem 6.12. Let a, b, c be nonnegative real numbers, no two of which are zero. Prove that »[a2+be gan se 1 Vere Veta Vere" 72 (Vo Quoc Ba Can, Crux Mathematicorum) Proof. Without loss of generality, we may assume that a > b > c. Then, we will consider two cases Case 1. 4a2b? > (a? + b?)(a — b)’. We claim that c+ab (a? +02)(b? +07) a+b? at +b? +22” This inequality is equivalent to a2 +b? +2c?\? 5 tet y(bt +c? a+b? (ab +2 or ; 2 +b? + 2 2c? a+b? \ a+b? a? +b? +2? a+b? 44 | (a —b)? = @btey Because +12>2and (ab +c?)? > ab, it suffices to prove that 42 %Xa-by ere abe which is true according to the hypothesis 4a?b? > (a — b)*(a? +b). Using the claim above, the original inequality holds if 268 Vasile Cirtoaje - Vo Quoc Ba Can - Tran Quoc Anh where x = a? +c? and y = {/b? +2. To prove this, applying the AM-GM Inequality, we have ey fp 1 2 2 4 aye g++ a= (1- 5 5+% te eye yt” xP ty! 272) \y? ° x? + ae xo+4 y6 1 xityé 2a(i-ae)+2 fr 292 2YBx2y? | xo 4H ys 1 es) (tty4)8 =2(1-—~ }+2{/—+ ( mm) 16x3 y3(x® + y®)" On the other hand, the Power Mean Inequality and the AM-GM Inequality gives us that 1 Qty P= BOP +t 2 4x8y3 (x8 + 9), Combining this with the above inequality, we get 2 33 | of ey 1 [4 St 4 2 2(1--— —= Vattee ( wa)? 6? as desired. Case 2. (a? + ba —b)? > 4a*b*. In this case, we can find that a? > 8b, and hence a? +? > =| 507422), Therefore, 4/9 2 [5+ = s[eabe |» bP+ea sfaete2 fh? +2 be +2 c2+a? ~ V¥ p24 ae Because {2+ +(e >2+ # we see that our inequality is proved if the following inequality is valid 1 af_ab a+b ¥6 For the sake i contradiction, assume that a > (3+2v2) b, Then, a? > 3367, and hence a? +c? > 17(b? +c?). Thus » de. a<(3+2V2)d. z af@+be 3/b?+ca_ g/a2+c2 4/2 +02 epee Iq = V T+ b+ eta’ B+ ere Our proof is completed. Equality holds if and only if a = b and c = 0, or any cyclic permutation. a More difficult inequalities 269 Remark. In the same manner, we can prove that the following inequality is valid for any nonnegative real numbers a, b, c, + «/[b? [2 +ab a Be the ae ab 5 Bre Vere a+b? Problem 6.13. Let a, b, ¢ be positive real numbers. Prove that a Be 3 sooo tt ooo NV 2} a? +7abe +b? 3 + Zabe + c3 8+ 7abe +09 (Nguyen Van Thach) Proof. Multiplying each side of the original inequality by ‘a? + b? +c? + 7abc, we can write it as a’ 32 = 3+ 7abe. +B +7abe de ae By applying Minkowski’s Inequality, we have (Zve)'+ +(x = Bue a+ ——.—__ > +b +7abe and thus, it suffices to prove that , (26)'+ (SV aeptae) Demo or equivalently, 2 a3 2 byab+ = YS OZ Zarb. Darve (x @+b+7abe Now, using the AM-GM Inequality and the Cauchy-Schwarz Inequality, we have = a2b? a fi ab ? In addition, Hélder’s Inequality gives us that (= ae (Se)2 (Ta)*, J t——“i RBA 270 Vasile Cirtoaje - Vo Quoc Ba Can - Tran Quoc Anh a+b +7abe} ~ (Sa) (Se) (Stab) + 6abe Sa] According to the well-known inequality 3xy2z(x + y +z) < (xy tyz+zx)’, we have (Ze) (Tat) ete Les (Le) (Tet) +2 (Tat) -(S9)(E9) and hence, it follows that 2 3 (Xe) (x a8 aS (Xe) Combining (1) and (2), we see that it is enough to check that ae)" (eo) he Oey Multiplying each side of this inequality by —"*?*" we get RHE LY ab be bealt Q) = 7abe. ab+be+ca _ 7abc(a+b+c) (at+b+c}* ~ (ab+be+ca)?" Because 7 Tabclatb+e) 7 ,aXb-c)? 3 (ab+bet+cay 6(ab+be+cay” and abt+be+ca 1 Yb-c) (atb+cP 3 Oatbt+c)”’ the inequality becomes $,(b —¢)’ + $,(c — a)? + S,(a — b)? = 0, where 7a? 1 (ab+be+caP (a+b+c)?” A) a, More difficult inequalities 271 and S}, S, are similar. Due to symmetry, we may of course assume that a > b > c, and then S, > S, = S,. On the other hand, we have 7(b* +c?) 1 > __(b+e? 1 (abt+be+ca? (atb+cP~ (abt+be+eaP (atb+c}? _ bte 1 bee “\ab+be+ca atbt+c) \abt+bet+ca at+b+e be +be+c? bte ~ (ab+be+caya+b+c) \ab+bet+ca ' a+bt+e Sy+5, >0. So, it is clear that S, 2 S, > 0. Now, because S,(b —c)* > 0 and S,(a —c)* 2 S,(a— b)*, we deduce that Sa(b —c)? + Sy(c — a)? +S,(a — b)? = S,(a — b)? + S,(a — b)? =(S,+S)a- bP 20, as desired. This ends our proof. It is easy to see that equality holds if and only ifa= Problem 6.14. Let a, b, ¢ be positive real numbers such that abe = 1. Prove that, for any k > 0, we have be_ ca ab 3 O+k B+k +k k+l (Wo Quoc Ba Can) Proof. Since a, b, c > 0 and abc = 1, there exist some positive real numbers x, v zr XY seas a y, 5 such that a= 4/72, b= #/—, c= */>. Making these substitutions, the xe y z desired inequality becomes yr x ead Ga) 4 kx? + (ya 4] ~ KFT Using the Cauchy-Schwarz Inequality, we have : . [Scammer] Loomure ron] (Zs) and thus, it suffices to prove that ce Kip, ; > kCxyz)* P84 Soy. CO 272 Vasile Cirtoaje - Vo Quoc Ba Gan - Tran Quoc Anh Due to homogeneity, we may assume that x + y +% = 3. Then, the inequality becomes k(xyz)4 31x94 + Dixy 53k +. Using the well-known inequality 3xyz(x +y +z) < (xy +yz+zx)*, we deduce that fag 4] BD RET (xyz)4s ; 2 From this, applying the AM-GM Inequality, we get oot Des Hoe) le) 2 (Sees) a S42 +3) 2 81 IA =3. Furthermore, it is clear that +yt2? Combining these two inequalities, we get Kecyz)/4 Sx + Yi xy sak +0), | as desired. Equality holds if and only ifa =b=c=1 of Remark. In the same manner, one can prove that the inequality be ca ab 3 | G4k Bak Hk RT | holds for any positive real numbers a, b,c such that abe = 1 and for any positive | real number k. Problem 6.15. Let a, b, c be nonnegative real numbers such that a+b +¢ =6. Prove that —4 b¢ = = 2 pasB sg 6=a+b+c=at2 +3 3 z ab2c3 y <1 mae Using this inequality and the AM-GM Inequality, we get and hence, it follows that a°b + b*c-+4ca*—Sabe+4> abe? 23 =2 ab 6 ie 46 a \ abr Sab ~ 2 12 3 22.38 men 6 6 2 bc ca? ab?c3 =) (=) - | yas} — Sate 12 3 atog8 [abil =30' ge as desired. Note that equality holds if and only if a= 1, b= 2 andc=3, 2 a?b+b*c+4ca®—Sabe +4: 2b >2-15%/2". 5abc = 0, Next, we will prove the right hand side inequality. In order to prove it, we need to consider two cases ‘The first case is when 3a” + Sac > be. In this case, we have b*c — Sabc < 3a*b, and hence ab + b?c +.4ca? — Sabe < a*b +3a7b + 4ca? = 4a*(b +c) aoa =i6- 2.2. (pace 6 grater meng gS 3 3 ‘ be The second case is when bc > 3a? + Sac. In this case, we have a2 < gt and CC es— 3 274 Vasile Cirtoaje - Vo Quoc Ba Can - Tran Quoc Anh hence b a?b + bc +4ca? — Sabe = a2(b + 4c) + b2c — Sabe < S +4c) + be Sabe be 4 1 SZlbtacj+bee= gbelb +c) < 3b +c) < 1 +b+ch= . 7 syle tefas= 2 < 128. So, in both cases, we always have a?b + bc + 4ca? — Sabe < 128, as desired. It is easy to see that equality holds if and only if a = 4, b = 0 and Problem 6.16. Let a, b, c be nonnegative real numbers satisfying ab? + be? + ca? +abe= 44, Prove that (Tran Quoc Anh) Proof. The original inequality is equivalent to (al + ot 4ct)"= St (an? + be? +ca? +abe) 7 4V4 Setting a= x3, b= y3 and c =23, it becomes 81 5 , (xty+z)t> ai (\ x48 + Vyte8 + Votn8 + xiv). By applying the well-known inequality (see Remark from the proof of problem 2.43) 27 (x+y +2) > (xy? 4 yz? +2x? + xyz), we get 27 (xty+e Oy? tye? tax? + xyelix ty +2) = 2 [Mety? +94) 42272 P x] . More difficult inequalities 275 Furthermore, from the AM-GM Inequality, we have and Therefore 3 3 BE ey Yaya Yxtyet, xty? +xy3)+2xyz ) x > xtys+ xty tet. Dixty? + xy?) +2xy2) aD y Combining this with the above inequality, the conclusion follows. Equality holds if and only if (x, y, 2) = (0, 1, 2/2), or any cyclic permutation. Oo Problem 6.17. Let a, b, c be positive real numbers. Prove that for any k > 2, we have Dive? +kab +b? < 4a? +(3k +2) Pad. (Michael Rozenberg) Proof. By the Cauchy-Schwarz Inequality, we have (SverKst) <[Tlorn] (Te) =2(X9)(pe*), and henee, it suffices to prove that a +kab +b? 49 07 +(3k+2) > ab ra eee Since a? + kab + b? = (a+b)? + (k—2)ab and Met + Gkt+2) ab =4 (Xa) +3(k-2) Drab, this inequality can be written as 3(k-2) ab 2Yfetn+2K-aY <4 at y 276 Vasile Cirtoaje - Vo Quoc Ba Can - Tran Quoc Anh Since k 2 2, it is true when ab 3yiab 2 <=. > atb™ die Multiplying each side by a + b +, we may write this inequality as ab(at+b+c) 2a S38 or equivalently, 1 2ab aes 4 abe ap Sate Now, using the Cauchy-Schwarz Inequality, we get 1 1f1 1 2abe ) —— <2 =(-+-]= is Sage <2 Dg (G45) = Dee ‘This is what we want to prove. It is easy to see that equality holds if and only if k=2,ora=b=c oO Problem 6.18. Let x, y, z be positive real numbers such that xyz = 8. Prove that Pe yw? A pg J +0634) VO3+D@+D VeF+DG+) 3 (APMO 2005) First proof. By the AM-GM Inequality, we have = 2 1 1 +1 Vat Det—x4l) &FDFGI—xF1) xP+2" Using this result and the similar inequalities for y and z, we get m x? y 4x? VEEDGF+412) AO? +2)? +2)" Therefore, it suffices to prove that 4x? 4 au (x? +2)(y?+2) 3" rr More difficult inequalities or equivalently, 3x2? +2) > (x? +2)? + 2)? +2) After expanding, we can write it as Dix?y? +2) x72 72. This is true, since by the AM-GM Inequality, Sixty? 23 Vxtytet = 48, and 23x? 2 6 x2y22? = 24. ‘The proof is completed. Note that equality holds if and only if x = y a Second proof. We will use the following result. Lemma. If a, b, ¢ are positive real numbers, then (a? +b? 407)9 > 2c*(ab + be +ea)*. eer , we have Proof Putting ¢= yf a+b? +c 2(a? + b?) = 1? + 2tc. ab+be+cas Thus, it is enough to prove that (20? +0?)3 > 2c?(t? + 2tc), which is true because (20? +.c)3 — 2c?(¢? + 2tc}* = = 06 —2c4e? — 8c5r9 + 10074 +88 > c& — 2c4t? — 20742(c? + 4?) + 10c7e* + Br® = co 481? +2074 + 84° oo 6 4y2 é 254 4,2 = (24+ 5 +805 -2ctt! = +2c?t4 — 2c4t? | > 0. (S Zt et'— 20 Ja (G42 c ) ‘Turning back to our problem, since xyz = 8 and x, y, z > 0, we see that there 2a 2c 2b exist some positive real numbers a, b, ¢ such that x = —=, y = a Using these substitutions, the desired inequality becomes B a s b(8a" + b3)(8c* + a3) ~ 3 Vasile Cirtoaje - Vo Quoc Ba Can - Tran Quoc Anh Now, using Hélder’s Inequality, we get : [= TT [Diteer+ aFx6e°+0%] (Lad) = (Yet) | So it suffices to prove that 9(Le)" = (Mav) [Teea? + o%y(609 + «)). Since S7(8a° + b3)(8c3 +43) = 8yia® +7351 05b3 < 9308 +72) 03°, 2 (Sav) (Mek+s are’), it is enough to check that Because )a?— "ab = XG — by, and Det +800? — (Sa?) =8 Ya2b? 3 Y a2 (a2 +b?) — 602622 = Seas +be+ca)—3ab?](a — bY < Ve (ab + be + cay(a—b)*, it suffices to show that Va- oy? 7 Ser(ab + be + cayia ~ bY 2) fab ee y But this is true since from the above lemma, we have 20d es Ye)” ae ay The proof is completed. aye (Se ay More difficult inequalities Problem 6.19. Let a, b,c be nonnegative real numbers. Prove that atbtc a? +b? +07" a,b |e 2a?+be | 2b?+ca © 2c? +ab Wasile Cirtoaje) First proof. Due to symmetry, we may assume that a > b > c. Multiplying by a+b +c, the inequality becomes in succession as follows soe) (at+b+c)? 2a*+be ~ a? +b? +c?’ (atb+cP a(a+bt+c) es eee Paha 2 1 2a2+be |’ 23a wae) 2 (et rHr+ey ELMER 9 3a*-(b~c}? ae b\(a-c)20, 9 VEO 5 a yv- exe TB 2a? + be We see that it suffices to show that arm a%(a-b)la-<) x 2a2+bo c—b Larne 2° Since (c — a)(c — b) > 0, the first inequality is true if b%(b—c)(b—a) . 2b? +ca a(a—b)la-c) 2a? + be Indeed, a*(a — b)(a—c) b= c)b-a) , a(a-bYb-o) b*(b—c)(b—a) 2a? + be 2b°+ca 2a + be 2b? +ca _ (a? = DY = bb =e) ~~ (2a? + be)(2b? +ea) = 280 Vasile Cirtoaje - Vo Quoc Ba Can - Tran Quoc Anh The second inequality is also true because x e=b __em~b | (a@=b)t(b=e) ,_b~a 2a? + be 2a? + be 2b? + ca 2c? +ab 1 1 1 1 =0-0(sS-ae +0-0(sya- are) _(a=b\b=c) (2a4+2b-c 2b+2-a ~~ 26? 4a Gar EES) _ 2a = b)(b = cla = c)a? +b? +c? x ab~ be~ca)., (2a? + be)(2b? + ca)(2c? + ab) Equality holds for @ = b = c, and again for a = 0 and b =c, b= Oande =a, c=Oanda=b. a Second proof. On the assumption that ¢ = min{a, b, c}, we can get the desired inequality by summing the following inequalities @ b 4 2(a+b) 2a? +be 2b? +ca (2a? + be) + (2? + cay’ and 2(a+ b) ¢ atb+e 2(a? + b?)+c(a+b) © 2c? tab ~ a? +b? +07" Using the identity (x + y)(u+ v) — 2(xu+yv) =(y — x)(u—v), we get (2a? + be) + (20? +a) eq) Meth a a+ be * BPE _ 2 a 2 ie: a b U(2b"+ en) — (2a +-be)] (= tbe 2b + =) _ (a= bP a +2b ~e)(2ab ~ac~ be) (2a? + be)(2b? + ca) = and the first inequality is proved As to the second inequality, we write it as 2(a+b) 2(a2 +b) + c(a +B)" 2c? +ab a? +b? +c? ~ which is equivalent to c(a?+b?-ab—c?)_ c(a+b)(a+b—2c) ab + 2c? ~ 2(a? + b?)+c(a +b)” More difficult inequalities 281 Since a+b — 2c > 0 and 2(a? +b?) + cla +b) = (a+b)? +c(a +b) = (a+ b)(a+b +c), it suffices to prove that a’ +b? —ab— atb—2c ab + 2c? a+bte +b) since a? +b? —ab> @ a > picivenough wo check that (a+ bP 4c? | ala tb = 2c) ab+2c2 ~ atb+ce This is true because (a+b) ~4c? 4(atb~2c) (at b~20)[(a= bP +3c(a+b—2)] 4 ab+22 atbtc (ab + 2c2)(a +b +c) = The proof is completed. a Problem 6.20. Let a, b, ¢ be nonnegative real numbers, no two of them are zero. Prove that be ca Veet cea Vato + ae +oroF * cra (Tran Quoc Anh) Proof. From Hélder’s Inequality, we have (SG) (Heo ro]= (He) and hence, it follows that 282 Vasile Cirtoaje - Vo Quoc Ba Can - Tran Quoc Anh Combining (1) and (2), we deduce that Eire Date Re RE and thus, it suffices to prove the stronger inequality atbte ws (atb+c)? vab+be+ca 2(a? +b? +c?) ~ Denote by L the left hand side of this inequality. By the AM-GM Inequality, we get a+b+c | atbte pat bbe? 2vab+betca 2Vab+be+ca Aa? +b? +c") (a+b+c)4 (a+b+c)* \ a(ab + bc + calla? +b? +c GY Tey So, the proof is completed. Equality holds if and only if a = b and c = 0, or b=canda=0, ore =a and b=0. a Problem 6.21. Let a, b, ¢ be positive real numbers. Prove that f bf if B+ +03 a? +b? 407" po ng PtabctO Ptabete | Otabeta (Nguyen Van Thach) Proof. Multiplying each side of the desired inequality by a3 + b3 +c +abe, we can write it as at (Me 8) (Sia + abc) Le Lape Now, by the Cauchy-Schwarz Inequality, we have Sat (dee)! ¢ a Lastaer® Sica? +abe +63) (Dev) More difficult inequalities 283 ee) Lea) or equivalently, | Thus, it suffices to prove that + (ue) vy Be). (Ee) Leen) (Xe*) (Xie This inequality can be written as 242)? er) > 4) ab? +2abe Da. a since ))a‘(b? te) 2abe a? =D ya%(b-c)’, SY a26?(a?+b?)— 2 0b? = Sia?b?(a— by and 2(e) sayays va - 23 )09b3 =— we can rewrite the inequality in the form Ve-oF (« +b8c? — which is true because Dato? +c) + Da b(a? + 67) + 2abe ¥a°(b—c)? Sev 4 2a*be >0, ab+bc+ca 2a* 4 4 @este 2 og hep ab+bet+ca 5 ab+be+ca 2a*be 4 PR (5) aes The proof is completed. Note that equality holds if and only if a= b 284 Vasile Cirtoaje - Vo Quoc Ba Can - Tran Quoc Anh Problem 6.22. Let a, b,c be positive real numbers. Prove that abc. Hab i. eb ae (al +be+ca)_ 17 boc a at+b? +c? (Vo Quoe Ba Can) Proof. First we prove the following statement Lemma, If a, b, ¢ ate positive real numbers, then ai bec 28(ab + be +ca) | bic a’ (@tbece =” boca Proof. Expanding and combining like terms, the inequality turns into 3 xS 5 vaya, ab > 10a, or, equivalently, (af+Ee- 2b ab? ab? 2S) e(4 = Notice that a a?h ab? B8 a*b — 2ab? b(a— b)? +) —-2) —= _ =) —. LPL S ET LF) re and ab? 4ca? c(2a — 3b)? 4D -aeb=¥ (F-20498) =p Applying the AM-GM Inequality, we get a a’b ab? ab? (= hs ES) (e-s a= blab? ‘2a — 3b)? ->| (aby? | e(2a~3b) c b 22) (a -b)(2a-3b) =10))a?-10 Vad. The lemma is proved. Lee) 210} a?-10 ab. EEE More difficult inequalities _ 285 Returning to our problem, we see that it suffices to show that 28(ab+be+ca) 7ab+be+ca)_ 17 (atb+cP a? +b? +c? . This can be written in succession as follows 7 4 7(ab+be+ca) > 28(ab + be +ca) 2° a+b? +c? (at+b+cP 7atb+c)? _ 28(ab+be+ca) 2a? +b? +c?) (a+b+c)* (a+b +c) > 8(a? + b? +c7)(ab + be + ca). By the AM-GM Inequality, we have 8(a? + b* +c*)(ab + be +ca) = 4+ (a? + b? +c”) - 2(ab + be +ca) < [(a? +b? +c?) + 2(ab + be +ca)}? =(at+b+c)* So, the last inequality is true and our proof is completed. Equality holds if and b 2n v7 ~ tan tan = ¢ = ——— = or any cyclic permutation. V7 tan ¥ a only if —“— = 7—tan> V7 ~ tans a Problem 6.23. Let a, b, c be nonnegative real numbers satisfying ab+be-+ca = 1 Prove that to dig bead 22+—=: vote veta Vvat+b v2 (Le Trung Kien) Proof. Without loss of generality, we may assume that a > b > c. First we will prove that 1 1 1 rere pin eceeetcin Wifes ®) Vbe+1 vctt1 V(b+c}?+1 This inequality is equivalent to 1 1, 2 ed 1 1 2 +l +l Ys iyet1) (b+) +1 (b+cP+1 Notice that (b+ c)? +1—(b? +.1)(c? +1) = be(2 — be) 0. More difficult inequalities 287 286 Vasile Cirtoaje - Vo Quoc Ba Can - Tran Quoc Anh Therefore Since 2 % 2 a 7 av2 1 2v2 (b+1Kc? +1) Vb+cp t+ + -2 pt+—— 5 . a) 2va Jeag Va='*34e-3 and hence, it suffices to prove that 2v2 yl (¢=2)(v2e?-r=2) | =t+— -2-— = ——* 20, >14—_1 a v2 vat? ~) (b+c +1 . 5 P the last inequality is true and our proof is thus completed. Equality holds if and only ifa = b = 1 and c =0, or any cyclic permutation. oO This is equivalent to. bel2~ 2be—belb+e}2] @+ Nero +erriy => which is true because 2— abe ~ be(b +e}? = Zab +e)— be(b +c)? =(b +e)[2a— be(b +e)] 2 (b +c)[2a—a(b +c)] = ab +)(2-ab—ac)>0. Now, using the inequality (+), we get 1 1 + =¥, vatb Vate mel i 1 + V(b+clbt+a) VS(c+aye+b) = 1 1 — bre =Vbt+e +——— | > Vose4,f—7* (& =) Wine (b+cP +1 Thus, the original inequality is true if 14 Jones BE soph vb+e (tera te + Vb +c > 2, we have bte 1 1 I O+Ps1 1 Bo @+cP+1 Beko oe bte and the inequality becomes Problem 6.24. Let a, b, c be positive real numbers such that ab + be +ca = 3. Prove that, for any nonnegative real number k, the following inequality holds — _¢_—_1 41 5 3 (a+kbe (b+ke> (C+kay ~ (k+1 (iran Quoc Anh) Proof. By Hilder’s Inequality, we have 1 3 3/4) 4 [Saves] [Maw +ke)] > (Ne ) . It follows that 4 (Ste) (a3/4 4 3/4 4 9/494 x ee kee = [Zee +ke)] ~ (k+ 13 (ab + be + cae (03/4 + 3/4 $4) © WIE Dab be + cay” and so, it is sufficient to prove that (a9/* + B94 + 63/4)* > oV3(ab + be +ca)*? Let a’/# = x, b9/4 = y and c¥/4 =z. This inequality is equivalent to (x+y +2)* > 9V3[(xy)*4 + (y2)3 + ex BP, Because it is a homogeneous inequality, we may assume that x + y +z Then, we have to prove that (ey)? + (yz) +(ex)¥3 <3 288 ‘Vasile Cirtoaje - Vo Quoc Ba Can - Tran Quoc Anh for x + y +2 =3. Applying the AM-GM Inequality, we have @y)8s xty+1 4-2 wae ¥te+1 4-x epee (3, gE 3 +» Gays 3 3 (ax) g FERHI 3 Therefore, (xy) + (ya) + ex)? = xy-(xy)!8 tye (ye)!3 dex (ox) s zits —2) + y2(4—x)+2x(4-y)] = jlaxy +yz+sx)—3xyz]. To complete the proof, we need to show that Alxy + yz +2x) —3xyz <9, which is equivalent to the third degree Schur’s Inequality A(xy + yzt+ex\(x+y +2) <9xyzt (x+y +2). Equality holds if and only if a = b Problem 6.25. Let a, b, c be positive real numbers such that abe = 1. If k > 0, then is, a 1 lta+bk 14+b+e (Wasile Cirtoaje) Proof. We have two cases to consider. Case 1. k > 1. By expanding, we can write the inequality as Meto+ Yates Vak+ Ya. By the Weighted AM-GM Inequality, we get kab + Sako = Dako+e)2 k+ Na k+l ott =(k+1) atta, More difficult inequalities and R+k+1 Yiaty ek ab = Sst (a0-+5 et — ) enta 244) FT a(k+p ye fe pea? cM) Peeey) an2snen)) BT =e! (Wt ) =G4+D aa es =D Yaa | Summing up these two inequalities, it follows that Srattb+ Watt > Yaa at + Sattar Moreover, by Chebyshev’s Inequality we have | ade 2 (Le) (Lat) asemae (Ze) and ayiatat =a alter > (Sa) (Sat) > x(abe}*8°5 (Sa) =3 0a, | and hence a | Yet + Valter > Yate Va. | Therefore, | Matty + Vath > Pat+ a, which is what we want to prove. 1 Case 2.0 1. Since zk = 1, from the preceding case we have 1 1 x <1 a + ltxtyt Llty+st l4tstxt for any positive reals x, y, z with product 1. Setting x =a*, y = b‘, obtain the original inequality. Note that equality occurs if and only if c=1 290 Vasile Cirtoaje - Vo Quoc Ba Can - Tran Quoc Anh Problem 6.26. Let a, b,c be positive real numbers. Prove that b € a _3¥3 [(a+byb+elce+a) Voag tote yee Vera Vara tVore = 4 atbte LL Proof. Replacing a, b,c by ~, =, —, respectively, the inequality can be written as x 3v3 [@+bb+oKe+a) Vath 4 ab+be+ca Using the Cauchy-Schwarz Inequality, we have a \ a a(4a + 4b +c) (yi 2et _S eat abohe), (Cas) ab +gabe, it is enough to check the following inequality ) (Sta) + Yoatoe 5 Yat? + Zabe. FE+ ON +eN(e+a)>4(Ta 291 More difficult inequalities After some simple computations, we can write it as Nato +13 Dab? > a2abe, which is obviously true according to the AM-GM Inequality. Equality holds if and only ifa=b=c. o Problem 6.27. Let a, b, c be positive real numbers such that a+b +c = 3. Prove that, if k > 0, then be+1 «feat (b+c) aa Tero Pe (iran Quoc Anh) Proof. Write the inequality as feat | fab+1) , *\ Vorra Veer J * Applying the AM-GM Inequality, we have faby1 Bb? pice tay pad H2 5 be +1 p24 5 een, B41 +1 ‘ab+l cat rfeat+1 , sfab+1 | Ak+1) 1 b?4] etl bri Veer abi‘ cavd or Therefore, Sie .fca+l 4 4fsbt 2 te isi a(b? +1) a(c? +1) we Ver k ab+1 ca+1 _6(k+1) 1 a(b? +1) | b(a? +1) ~ k k ab+1 ab+1 6k+1) 1 _ (k+1) 6 Te -7 e+) = ke k Equality holds if and only if a= b = | Vasile Cirtoaje - Vo Quoc Ba Can - Tran Quoc Anh 292 Problem 6.28. Let a, b, c be positive real numbers. Prove that abte pete cath 3 +ep trap Gabe =4 (Vo Quoc Ba Can) Proof. Without loss of generality, we may assume that a > b > c. Then, we will consider the following two cases. Case 1. b +c > 1. We can easily prove that 1 for x > 0. Using this result, we have 4arte —gyeta 4cotb He peta cath bree * Grae * toe ey eta) (atbye eS abt pete cate Torey * (= Feta * Fanbyre 7) (=) &) =( 2a yn 2b Der \ ere b+e c+a “+ a) 7 Since min{b +c, ¢+a, a+b}=b+c > 1, by Bernoulli’s Inequality, we get 2a \ ote 2a be (=) =[+(FE-))] z14(bte) (A -1) =1420-b-c Similarly, we have that 2b \‘t# Qc \atb > a 2 4 (4) 214+2b-c—a, (S) 21+2c-a-b, and therefore, aabts | ancts aett® 7 a0 yh 2b Yo" 26 0H +P eray gaa? a) +(&) (3) ae Case 2. b+c <1. Ifa>1, then abt 3 —— >a’ > 1s > Gree Ae More difficult inequalities and the desired inequality is clearly true. If a < 1, then 1 2 = 2b, Spee *a+by tary ~ ore (c+a? (b+cP (c+ay? This completes our proof. Note that equality holds if and only if a 1. Remark, Similarly, we can prove the following more general statement: For any positive real numbers a1, dg, ..., dn Satisfying a, +a) ++---+a, =5, the following inequality holds: sa, an n + Gay n- 1)?" Problem 6.29. Let a, b,c be nonnegative real numbers, no two of which are zero. Prove that . 5 | vab vbe va 9 +b +c ab+c2 be+a2 ca+b?~2 (a+b+c)" (Tran Quoc Anh) Proof. By the AM-GM Inequality, we have 6ab+2(a2—ab+b)]°>_ 8 ae Beato 6ab(a? — ab +b?) < [ 3 This implies that 9 zvanla? —ab+b*)< (a+b). According to this result, we find that Va. 2 Jab(a? — ab +62) vab vab- (a+b) ab+c? (a+b)(ab +c?) (a+b)*(ab +c?) 9 _ abla? +b ) >? ab(a? + b3) ~ 2° (a+b)i(ab+c2) ~ 2 (a+b+e)(ab +c?) Vasile Cirtoaje - Vo Quoc Ba Can - Tran Quoc Anh More difficult inequalities 295 294 Therefore, xy vab yr ab(a? +b’ ab+c? ~ %a+b+c)* ab+c? From this inequality, we see that the desired inequality is true if abla? +3) be(b? +03) ca(c3 +a) ab +c? be +a? ca+ b? 2 a +53 +03. To prove this inequality, we rewrite it as ye ab ca ab+c? * ca+b? which is equivalent to eee. @be(a? — be) (teal? +ab) ~ 20, or abc} }a*(a? ~ be(a? + be) > 0. Since Sera? — be)(a? + be) = Se*tat — 022) = a6 + 66 +6 —3a2b2c? > 0, the last inequality is obviously true. This completes our proof, Equality holds if . 74+3V5 and only ifa =O and b = ¢, or any cyclic permutation. a Problem 6.30. Let a, b, c be positive real numbers. Prove that sfa+be fb? +ca +t c2-+ab Seah 3Vabe a(b+c b(e+a. c(a+b) ~ a+btc (Pham Huu Duc) Proof. Squaring both sides, we can rewrite the desired inequality as xs @+be 3/(a2+be)(b2+ca) 3¥abe a(b +c) abla+c\(b+c) “at+b+c~ Since (a? + be)(b? +a) — ab(a +c)(b +e) = cla +b)(a — by? = 0, we deduce that x 4| (a? + be)(b? +.ca) ab(a+c)(b +c) and thus it suffices to prove that @+be | 3¥abe x a(b+c) atb+c™ Using the AM-GM Inequality, we get @+be a +be 2(a? + be) 2(a? + be) a+) Jets boatbaa — @ +b) abt @+baro" and 3Vabe 3abe > 9abe a+b Varer(atb+c) (ab+be+cala+b +c) 9abe ~ (a+ b\b tele +a) babe 5 9abe (a+bXb+e)(e+a)+ fet orokere) Babe ~ (a+ bb teeta) Combining these two inequalities, we obtain a+be 3V¥abe 2(a? + be) 4 8abe Ly aero* atb+e~ 2 Lasher (a+d)(b+e)e+a) This is what we want to prove, and so our proof is completed. Note that equality holds if and only ifa=b =c. a Problem 6.31. Let a, b, c be nonnegative real numbers, no two of which are zero. 471 14 s\b+ccta ath)" (ham Kim Hung) Prove that a b? +c? Proof. Firstly, we will prove the following auxiliary statement. Lemma. If a, b, ¢ are nonnegative real numbers, then 2 be e 296 Vasile Cirtoaje - Vo Quoc Ba Can - Tran Quoc Anh Proof. We have the following identities a? a _ ab(a~b)+ac(a—c) +e? bre (Bb 4e2)b+o) a be(b ~ c) +ab(b — a) +a cra (+024) ce € _ cale—a)+be(c—b) @+e ath (@+b a+b) Thus, we get a a ab(a~ b) ab(a~b) Lisa Lin-L waa ereal ab(a— b? * (iets Net) FOOTE + EEA" and the proof is completed. Returning to our problem, multiplying each side of the desired inequality by 5(a+b +c), we can write it as afat+b+c) a+b aD bP +c? 240) b+c which is equivalent to 2 a a(b +c) a Sasa t5 7 24))— +12, Dipset x be+c? = uraet a Using the lemma above, it suffices to prove that 2 a a(b+c) Leet h ge?” Without loss of generality, we may assume that c= min{a, b, c}. We have a(b+c) _ b(c+a) _ alb+c) b? +c? +a? ~ be +be | cata? and c(at+b) | 2c? zs a +b? = a+b? It follows that RRC) 10c* 24y 8c? + * be +c? ~ a+b? a? +b? f More difficult inequalities 297 so it is enough to check the following inequality a e 9c2 soto t Bee” eye ate By the Cauchy-Schwarz Inequality, we have a b? + 9c? s (a? +b? +3c?)? a? +b? > a(b? +02) + bc? +a") Fea? +b) 4b? +32)? © 2(a?b? + bc? + 2a?) It +c? +a On the other hand, it is easy to see that (a? + b? + 3c?) — 4(a?b? + b?c? + c2a2) = (a2 — b?)? + 2c2(a? +2) 4.904 > 0. Thus, the conclusion follows. Equality holds if and only if a = b and c =0, or any cyclic permutation, a Problem 6.32. Let a, b,c be nonnegative real numbers such that a+b+c = 3. if k>0, then ‘esli-nshere 3 ee a*b + bke+cha < max} 3, ——__ } . v 4 (ke Masile Cirtoaje) Proof. We consider three cases Case 1. 0 < k < 1. Using Bernoulli's Inequality in combination with the well- . . (atb+c) known inequality ab + bc +ca s | —, we get akb+bkce + cha =b[1+(a-1]* +c[14(b— 1)‘ +al1+(c-1))* Sb[1+k(a- 1) +e[1+k(b-1)] +al1 + k(e-1)] (a+b+eP =k(ab + be +ca)+3-3k y2220and m > 1, then gktigk ato beteasasmax{s } xy tyMaeMx > xy" + yo" ax”, 298 Vasile Cirtoaje - Vo Quoc Ba Can - Tran Quoc Anh we can see that it is enough to prove the inequality for a > b > c > 0. Letting ght ge ke" inequality can be written as now K = max}3, femetsand b= c41 62 120) te Pn s(c)=K (c+ Sh ~(cts)Met (c+ fe —eX(e +5) 20. ‘We have #o=Kik+0 (c+ ay H R(t) e+ e(e+ Oetker) — [Cc +s) +(e +e) +e8] > 3(k+1) (c+ = =K(c +s! Mert (c+ tc+ ch 'e+s)J- —[e +s) +(e +e) +c] b =aken (A k ) —kak'b + be 164 ck 1a) — (a + bE + 8). We will prove that g’(c) > 0, ie. atb+e\* ace (2**) > k(a’'b + bee 4. ck1a) + (ak + BF + c%), Applying again the above result for x = = beg = ch! and m= Foy? bweset KT (BT ET ch1 4 (NTA (@k}) > pa Eye $b ¢ Mae, ak 1h + oT + ck la < abe) + bck! + cak Using this, we have g’(c) > 0 if at+bt+c k zl ) 22ak bk ech) tk Sa (b +0). ou+0( Due to homogeneity, we may assume that a + b +c = 3 (because the case a=b =c = (is trivial), when the inequality becomes 6k +1) 2 2€a* + BK +c8) +k a'(3 -a), More difficult inequalities 299 or equivalently, (2= bak + BF + ck) + kak? + BET +e!) < (kK +1). We need to prove that h(a) +h(b) +h(c) <0, where h(a) = (2— k)a* + 3ka*“! — k(2k — 1)(a — 1) — 2(k + 1). We have W(a) = k(2—k)a*? + 3k(k — 1a? - k(2k - 1), and h(a) = k(2—k)(k— Da*? + 3k(k - 1)(k— 2)a*> = k(k—1)(2- Kak 3(a - 3) <0. This shows that h’(a) is decreasing for a © [0, 3]. Since h’(1) = 0, we get h(a) > 0 for a € [0, 1) and h'(a) < 0 for a € (1, 3]. Therefore, h(a) is strictly increasing for a € [0, 1] and strictly decreasing for a € [1, 3]. From this, we get h(a) < h(1) = 0 for any a € [0, 3]. Using this and the analogous inequalities h(b) <0 and h(c) < 0, we get h(a) + h(b) + h(c) < 0. This proves the claim that g’(c) = 0. Therefore, g(c) is increasing for c 2 0, and we have eee rye k a2 =x (+) ~seak ($*) -«.(2) “t kn s see [egret 1 gk+igk . 2K = = K-———a tt 20. >«( 3 ) eee gen [K-@p ped [E+ = So, in this case, we also have ght ath +bkc +cka < max} 3, ——— }. «+ pet Case 3. k = 2. Due to cyclicity, we may assume that a = maxfa, b, c}. Then, applying Bernoulli’s Inequality, we have x k (ate =atb (1+ £) zatn(1+) a a k 2c ke k=1 k-2, zatb (14 =) =atb+al"be+a' abe >akb +b "be +c%2ac? =a*b + bke+cha. 300 Vasile Cirtoaje - Vo Quoc Ba Can - Tran Quoc Anh On the other hand, from the AM-GM Inequality, ate Ere k k-——+b (4) beck k ° k k+1 ght yk ght =———— b > ¢ > d. Then, we have and 1+a? Thus, by Chebyshev’s Inequality, 1 1 1 ee G+aata)” G+ b)0 +63)" (Feit +c) ~ sift, 2.47 1 3\ita 1+b 1te (= More difficult inequalities B01 | Now, since a > b = c 2d > 0 and abed = 1, according to problem 2.36, we have 1 11,3 | Tea tit 1+e> 1+k’ and 1 l+a where k = abc. Therefore, we get 1 1 1 3 5 at art! m2 = (+aja+a2) +b) +b) G+oate)~ +k) +k) and it remains to prove that 3 1 eee eee G+h0 +) (+3) ) After some simple computations, we find that it is equivalent to (k-1P OK + +k +2) | (k3 + 1)(ke +1) ° Of course, this is true and our proof is thus completed. It is easy to see that equality holds if and only ifa = b= c= o Problem 6.34. Ifa, b,c, d are nonnegative real numbers, then a 4 bt tc4 44+ 2abed > a2b? + a2c? +a2d? + bc? + b*d? +07d? (Turkevici) First proof, Due to symmetry, we may assume that a 2 b = ¢ > d. Write the inequality as E(a, b, ¢, d) 2 0, where Ela, b, ¢, d)= Sat + 2abed ~ Y\a"b? om = Dat + 2abed — (a? + 62)(b? + d?) — ac? — bd?, We will show that F(a, b, ¢, d)>E( vac, b, ¥ae, d) > 0. ee, | Ga 302 Vasile Cirtoaje - Vo Quoc Ba Can - Tran Quoc Anh ‘The left hand side inequality is equivalent to a* +04 — 2a7c? > (b? + d?)(a? +c? — 2ac), (a+c)(a-c)* > (b? +d?) —c). This is true because (a +c)? > a? +c? > b? +2, For the right hand side inequality, we can write it as follows a? + b4-4+d* + 2abcd > 2ach? + 2acd? + b7d?, (ac? + b4 — 2acb) +.d(d? + 2abc — b2d — 2acd) > 0, (ac — b?)? + d(b — d)(2ac — bd — d?) > 0. This is true because 2ac ~ bd ~ d? > 2bd — bd - d? = d(b—d) >0. The proof is completed. Equality holds if and only if a ,ora=b=c and d= 0, or any cyclic permutation thereof. a Second proof. Without loss of generality, we can assume that a > b >¢ >. Since Set —4abed = (a? - bP + (C2 x 2b? = Ya? - by, sym am 2 +2(ab — cd)? and we may rewrite our inequality as (a? ~c?)? + (b? = 2)? + (a? — d?)? + (b? — d?)? > 2(ab — cd. Now, applying the Cauchy-Schwarz Inequality and the AM-GM Inequality, we obtain (a? = d?)? + (6? 1 1 r= 3 +b? —2d?)? > > (2ab - 24°)? = 2Aab—d?)? > 2ab—cd)*, as desired. o Third proof. We will assume that a 2 b > c > d and rewrite our inequality in the form at + b* +c4—a7b? — bc? — 02a? 2d [d(a? + b? +c? — a?) — 2abc], that is (a? — b?)? + (b? — c+ (c? — a?)? > 2d [d(a? + b? +c?—d?)—2abc]. More difficult inequalities 303 Since (a? — b?)? > 2d(a + bY(a — b)?, (b* —c?)? = 2d(b + c)(b - c)’, and (c? — a)? > 2d(c +.a)(e — a), we have (a? — PP + (7 = P + (C2 - a P > 22d [(a+ bya bP? +(b +eN(b— cP +(c+aNle—a}*]. It suffices to prove that (a+ b)(a—b)? +(b+c)(b-c)? +(e +a)(c—a)? > d(a? +b? +c? — d?)—2abe, or equivalently, 2(a3 +3 +c +abe)—ab(a+b)— be(b-+e)— calc +a) > d(a? +b? +c? —d?). Now, using Schur’s Inequality ab(a +b) + be(b +e) +ca(e+a) Sa? +b3 +3 + 3abe, it is enough to show that 2(a3 +b? +03 + abe) — (a3 +b? +.c3 + 3abc) > d(a? +b? +c? — a”), or a? + b3 +c3 — abe > d(a? + b? +c? —d?). Applying Chebyshev’s Inequality, we have @O+P+3> 5+ b+c)(a? +b? +c), and it remains to show that ia +b+c)(a? + b? +c”) — abe > d(a* +b? +c?) —d°, or (a? +b? +.c?)(a+b +c —3d) > 3(abe - d°), which is true since by the AM-GM Inequality, we have a(abe ~ &°)=3( Yfabe—<) (Vor +43 abe +e) < 9Va2090 ( Vabe ~ (Sad), and therefore , a (Sa)? (ab +0s+ed ras)” (a+b toy ~ [Nac+b+0]° [ateP +O tae tater a” Using the Power Mean Inequality, we get atc aiecdza thus ‘ a3 atc\s b+d\5 PC) +2(44) | (atc? +(b+d)}?+(a+c)(b+d)]? 1 [@rot+@+as]’ 2 [ate tbrdPtataorae senting c= (22) the inequality b t= (S5)”, che inequality becomes x a sh (e441) (atb+c ~ 2 (t8+094+1)?" Thus, it remains to show that 92 +1) > 8h +08 +1), or More difficult inequalities : 305 1 F Setting u = ¢-+ =, u> 2, the above inequality becomes 9(u2 — 2)? = 8(u? -— 3u +1), or (u—2)?(u4 + 4u3 + 6u? — Su — 20) 2 0. Since ut + 4u3 + 6u2 — 8u — 20 = ut + 4u2(u — 2) +4u(u— 2) + 10(u? — 2) >0, the last inequality is true and the proof is completed. Equality holds if and only ifa= =d. o Problem 6.36. Let a, b,c, d, e be positive real numbers. Prove that Q+se) (1445) (+353) (+243) Q+sa)2 (3). Proof, Due to homogeneity, without loss of generality, we may assume that a+b+c+d+e=1. Then, we have a _a+bt+e_1-(+e) bte bte bte it and hence, the inequality can be written as faa roltt ero ter mii Grote}, (3 (b+eNe+ dd rene +ayatb) 3) , or equivalently, MAA) Now, note that for any x, y > 0, x+y < 1, we have (2-4) (2)-( 2 -1) =e x J x+y xyty) 306 Vasile Cirtoaje - Vo Quoc Ba Can - Tran Quoc Anh ( 1) 1 1)s 2 2 2 2 eta (i )? (aaa) =(4-3) . Multiplying these five inequalities and then, taking square root of each side of the obtained inequality, we get (=) (ea) (EMS VEN Thus, it is enough to prove that IMEEM i-a Nae \A-) (4-1) or equivalently, Xa(Z i= 2 Since the function f(x) = In (; a Iv — or oe 1) >s 3 25In5. -1) is convex for any x € (0, 1) (its oa de rx second derivative is Ta xay 7» We may apply Jensen's Inequality to get =x Fa) + f(b) + fC) + Fd) + fle) > 5F (ee) or equivalently, da(- Therefore, the last inequality is valid and our proof is completed. Note that equality holds if and only ifa = b =c=d=e. a 5h : =5in5. More difficult inequalities 307 Problem 6.37. Let a1, a2, -++5 dy and by, by, ..+4 By be positive real numbers. (Adapted after an Ukraine 2001 problem) Proof. ‘Taking square root of each side and using the identity al fa we can rewrite the desired inequality as Em) E3.0) Oe) ) 1sigjsn isi nn) icizjen isigjen ya? +2 aja, Y b2+2 bybj i Yj : = is isigjsn isigisn (EJ (:.5,0=) (25.0) <(S 1stejsn Isiejsn i=1 so it is clear that a a. ‘The proof is completed. Equality holds if and only if - = . = 1 be 308 Vasile Cirtoaje - Vo Quoc Ba Can - Tran Quoc Anh Remark. The special case n = 3 of the desired inequality is the well-known Ukraine 2001 Inequality. Beside that, this nice inequality has also many inter- esting applications and we will now present some of them here Application 1. Prove that for any positive real numbers x;, x, ..-,%, and yi, Ya, -+++ Yq Such that yo =(3): reigten the following inequality holds x1 "00 ti + yg) Fo} Xp be + Ky Oe Yn) Xt Fe (te + Yn) 2 (Vasile Cirtoaje, Gabriel Dopinescu) Proof. The inequality is homogenous in x,, x2, ..., X,, 80 we may suppose that xy +x) ++++-+2, = 1. Then, it can be written as Syn. im isl Applying the above inequality for (0,3) = (FE, En, ie" Tx and (Bt, Bay wees bn) = Os Yas vs Suds we get 2 dy} 24] ¥ Ee a a xy ow, (iA isigjen istajsa XiXj ee mend 2 aaa and thus, it suffices to prove that xix) a fhe, TIA) = 2 This inequality is true (see problem 5.8). o More difficult inequalities 309 Remark. The special case n = 3 of Application 1 is a well-known result of Titu Andreescu and Gabriel Dopinescu: If a, b, c, x, y, # are positive real numbers, then 7 G++ po+y2 ViGy tye te. (*) | This inequality has also many nice applications. b et Application 1.1. Let x, y, 2 be positive real numbers. Prove that | xy(xty—z)+ys(y +2—x)+2x(2 +x —y)2 V3G8y93 + 22? + y%29). Proof. Since x 3 yOe-by—2) = Bg = 2492 ys)= 3403 Sixy(ety-2) = Dox?+2)-axy2 = S742") Det +25), we can write the desired inequality as yo FZ V3OPy? +72? + y%29). In this form, we can see immediately that it is a direct consequence of (+). 0" +2)+ SSE +x) + > yt Application 1.2. Let a, b, c be positive real numbers such that ab+ be+ca = 3. Prove that a(b?+c2) b(c2+a2) cla? +b? st ot or? a? +be b+ca ce? tab (Pham Huu Duc) Proof, get x = a(b? +c), y = b(c? +a”) and z = c(a? + b?). Then, x a(b? +c?)(b+e) a(b? +c? yar *O= yeaa) tebe” ytz b(c? +4”) +c(a? +b’ a’ +be and the inequality can be written as x y z — ate hah +b)>3. FEO tote a) yay" )2 Using (*), we have 7H (b+e)+ oe +a)+~ gatne 3(ab + be +ca) = 3, and the proof is completed. BELIOIECA QUDETEANAT aq Panait Istrati BRAILA 310 Vasile Cirtoaje - Vo Quoc Ba Can - Tran Quoc Anh Application 1.3. Let a, b,c be positive real numbers. Prove that Bec eta? ath? _3f f@rbro@te +e —— + + >> | 4] ———_————-1 |. ab+e) beta) a+b) ~ 2 abe Proof. Note that Bee? be a+b abt+o) abtea\ b 7c Thus, by (*), we have a+ be +c? r be cha at s a(b+c) ab+ca b 5 ~ and hence b2 +c? (a+b)a+c) ers 2y900 be “3 Furthermore, from the Cauchy-Schwarz Inequality, we have (at+byate) _ 3(a+b+c)(a? +b? +c?) kre FP abe 33 1% (a+b+c)a? +b? +c") “2 abe Combining this with the above inequality, we deduce that pi 53 | [arb tena? +b? +c%) ab+e 2 abe a as desired. Note that equality holds if and only if a= b =<. Q Application 2. Let a, b, c be positive real numbers such that ab + be +ca =3. Prove that for all r > 1, we have a'(b+c)+b'(c+a)+c'(a+b)>6. (Vasile Cirtoaje, Walther Janous) More difficult inequalities 311 Proof. Applying the inequality [x(v +w) + yw +u) + 2(u + v))? > (xy +y2 +2x)(uv + vw + wu), (the special case n = 3 of the above inequality) for (x, y, 2)=(a", 6”, c) and (u, v, w) =(a, b, €), we get [a’(b +c) + b'(e +a) +e"(a+ b)I? > 4(a"b + b'c" +c7a" (ab + be +ca) =12(a"b’ + b'c’ +c'a"). On the other hand, from the Power Mean Inequality, we have abt+be+ca\" _, ab! + bic" +c'a" 23 —7-— ] = 3, Combining this with the above inequality, we deduce that [a"(b+c)+b'(c+a)+e(at b)]? > 36, and hence, it follows that a’(b+c)+b'(ct+a)+c"(at+b)>6, as desired. Note that equality holds if and only if a = b Application 3. Let n > 2 be a real number and let a, b, ¢ be positive real numbers. Prove that +b beet att bt te atb+e a"+b" bt +c" ctta"_ [3(a’ atb b+e cta Proof. Applying the inequality [xv +w) + y(w+u)+5(ut v)]}? > 4(xy + yz +2x)(uv + vw + wu), bre” ota” abt 1 for (x, ¥, 2) (a *) and (u, v, m=(3 ee (css) - DEG) Eater] Cre) -4(Sasgera] 2): and thus, it suffices to prove that ade Lgrobra 24 Ma . 312 : Vasile Cirtoaje - Vo Quoc Ba Can - Tran Quoc Anh Using now the Cauchy-Schwarz Inequality, we have ony a x (arene +e) ~ Vera +elb+e so that we only need to show that (Cs) Der)esLereroeta Ver are +e) = (Xa) (Se) + Pave, it can be written as Because (Sa) (Me™) 23 Vane, Sar(a? + ab +ac — 3be) > 0. Without loss of generality, we may assume that a > b > c. Since n > 2, we have a™? > p?? > ct? Also, it is clear that or equivalently, a’ +ab +ac—3be > b? + be-+ba—3ca Bc? +ca + cb — 3ab. Therefore, by Chebyshev’s Inequality, we have 3 a™%(a? + ab +ac~3bc)= (Sra) [a2 +ab +c 3be)| = (Sor) (Le Tes) e0 ‘The proof is completed. Note that equality holds if and only if @ Remark. For n = 2, we get the known inequality a+b? b2+c? | c? +a? + +} > 3(a? + b? +c?). a+b bt+e c+a Problem 6.38. For all positive real numbers a}, dg, ...5 dy, the following inequal- ity holds (Vo Quoc Ba Can) More difficult inequalities Proof, We begin with a preliminary result Lemma. If 1, if k= =! fee m=), (k=1) if ke \a+p then ak 1, the inequality can be written as : ms aj ( For the nontrivial case 0 < a, <1, this inequality is equivalent to i 1\2 (a aPt) 3, then 6 Berens? 2 + D(Rt) veoh . xixj% S 1. (Vasile Cirtoaje) More difficult inequalities Proof. For each i, we denote the set {1,2,..., n}\{i} by S;. We have te isi jxk ODES: 316 Vasile Cirtoaje - Vo Quoc Ba Can - Tran Quoc Anh where B; Dd x3x2. Therefore, Now, from (1) and (2), we get es )ixt+ [20-20m+ i=1 (m= 1(n~ 2? ] an Since 2(n —2)m + ene, we have e 13. (Vo Quoc Ba Gan) 317 318 Vasile Cirtoaje - Vo Quoc Ba Can - Tran Quoc Anh Problem 7.4. Let a, b, ¢ be real numbers in the interval [1, 2]. Prove or disprove that (Qa- a 2a) (2b —c)(2b-a) (2c — a)(2c — b) Pre e+e TY @ep 7a (Vo Quoc Ba Can) Problem 7.5. Let x, y,z€[-1, 1] andx+y+z=0. Prove that vi 7 7 1T+x+cy?+4/1 x22 oy xhey ty tet yltet oes 23. (Phan Thanh Nam, Phan Thanh Viet) Problem 7.6. Let a, b, c be positive real numbers. Prove that Feat waar ¢ weer?) ape +h te (Pham Sinh Tan) Problem 7.7. Let a, b,c be positive real numbers, and let n = 2 be a positive ’ : : inceger If isa positive real number such that k-+ > > (3) — 2, then nf _@ af _b c 3 / + eee kb+c ke+a ka+b~ Vk (asile Cirtoaje) Problem 7.8, Let x, y, z be nonnegative real numbers satisfying a+b +c=3. If O. (Vasile Cirtoaje) Problem 7.11. If a, b, ¢, d are positive real numbers such that then (1-abed) (artureettan for any positive integer n. (Vasile Cirtoaje) Problem 7.12. Ifa, b,c, d are positive real numbers such that (Xa) (=3) >10+4V6, ((S9) (Z2)22+ (Le) (Za) Problem 7.13. If a, b,c, d are positive real numbers such that (Xe) (x3) <10+4V6, then then (X9)( 1 a Wasile Ctrtoaje) ) -102 3(Si2?) (E3)-2 (Vasile Cirtoaje) Vasile Cirtoaje - Vo Quoc Ba Can - Tran Quoc Anh Problem 7.14. If a, b,c, d, e are positive real numbers, then =o. b+c ctd dt+e e+a atb (Vasile Cirtoaje) Problem 7.15. Ifa, b, c,d, ¢ are positive real numbers, then 4 end c+2d+e d-¢ e-a b-c ——"_+——__ 30. d+2e+a e+2a+b~ b+2c+d a-b a+2bt+c Problem 7.16. Let a, b, c,d, € [0, 1]. Prove that 1 1 1 1 1 6 + + + < - Ttatb*14¢b+c 1tc+d 1tdte 1teta~ 142¥abede Problem 7.17. If a, b,c, d, e are nonnegative real numbers such that a+b+e+ d+e=5, then a b € d # 5 —— + —— + —_ + —_ + 2S; @ Tarte ael aaa eee 2 a 6 ¢ d eo Ko Q+3 e431 M43 * 243 Dat3~ (Vasile Cirtoaje) Problem 7.18. If a,b, x, y,2 and p, q are positive real numbers, then 1 9 a LierpaTD = 30+ pyab + bet ca)’ 1 9 ® Lerner CF PITF gab + Be ea 1 | © Lamar (+p) +a) Db + ex t (Wasile Cirtoaje) Some open inequalities _ 321 | Problem 7.19. If a1, 42, --» y are positive real numbers, then 1 1 + —_— n? @+ajaq * a+ anes aga, ~ 2(djay + dgdy ++ + ayy) (Vasile Cirtoaje) Problem 7.20. If @;, a2, .--,4y are positive real numbers such that ay +ag+++°+ a, =n, then ay a, +(n—1)a5 nas EE a, +(n—= Maz (Vasile Cirtoaje) Problem 7.21. Let ay, ay, ...5 dy be positive real numbers such that a,a3° +4, 1. Prove that iu 1 + tt GE 2+a,+a2 n—2+a,+a3 (Vasile Cirtoaje) Problem 7.22. Let aj, a2, ..., @, (n > 4) be positive real numbers such that ya +d, = 1, and let p, q and r be nonnegative real numbers such that p+q+r = n-1. Then, 1 1 1 n Z at 2 3h + 2 3 2 : 1+pa,+qa?+ra3 1+ pag +qa2+ra3 1+pa,+qa2+ras~ 1+p+qtr Wasile Cirtoaje) Problem 7.23. Let aj, 2, ..., Gy (n = 3) be positive real numbers such that ,03+++d, = 1, and let p and q be nonnegative real numbers such that p +4q > n— 1. Then, 1-a l-a, 1 =a, 1+pa,+qaz 1+pa,+qa3 1+pa, +qa? ~ (Vasile Cirtoaje) 323 322 Vasile Cirtoaje - Vo Quoc Ba Can - Tran Quoc Anh Some open inequalities Problem 7.29. Let ay, dy, -+-) dy be positive real numbers. Prove that sh ee | 2 +agt--+a,\7 >. (ute: a, +a, a, ja?+ (FE) +--+ ater tt) i 2 i . Tq 2 aaa a? +a3 az +a3 2 undp 2+ n Problem 7.24. If x1, Xa, -..4Xq (m2 3) are positive numbers such that xy bg bee by SMX Xp, 2 p AX XQ--Xy then 1 tt T+( ig tae, = | (Wasile Cirtoaje) at < Problem 7.25. If ay, dg, ..., a, are positive real numbers and n is odd, then Je) ED (Vasile Cirtoaje) Problem 7.26. Is there a real number k > 1, which does not depend on n, such that a ay 4 8 @, +03 43404 a, +02 n 5f 2 for any natural n and any real numbers ay, day +++ On € (1, Kk]? (Vasile Cirtoaje) are positive real numbers, then Problem 7.27. If a1, a2, ay + a2 (n= 2)ay Fay tay (M—2)ay +g +04 + i) <1 (n= 2)a, +a; +427 ~ a, and x1, Xp, ..., Xq be positive real numbers Problem 7.28. Let a1, a2, ... such that (ay tag tes tay)(xy bgt b_) = (2 HaP p02 )P bad bx?) = (n-1P, ita my tg n and let P, = (aa2+*+dy)(x1X2**%p). Prove that (a) for n < 6, P, is maximal when a, = x, = -2 a, = X_ = 3 i (b) for n> 7, Py is maximal when a; = az and a) = x)=" = n= Land x) =x) = se Xqat: (Vasile Cirtoaje, Vo Quoc Ba Can) Vasile Cirtoaje - Vo Quoc Ba Can - Tran Quoc Anh Appendix A Glossary (1) Abel's Summation Formula Let a1, dp, ..., dy and by, bo, «-., By are real numbers. If $; = a, +++ +a, for any i=1,2,...,n, then | Said = Y Sibi ~ bias) + Sado: a a (2) AM-GM (Arithmetic Mean-Geometric Mean) Inequality If ay, dg, ..., dy are nonnegative real numbers, then ay tay +e $a, 2 n9/aqag- Gn, with equality if and only if @) =a) = (3) AM-HM (Arithmetic Mean-Harmonic Mean) Inequality if ay, dy, ..., dy are positive real numbers, then ay $a, +++, n Se with equality if and only if a, = a) =- (4) Bernoulli’s Inequality For any real number x > —1, we have a) (1+x)' >1+rx forr > land r <0; b) (14+x)' $1+rx ford (ayby + agby to band) with equality if and only if a; and b; are proportional for all i. (6) Chebyshev’s Inequality Let a, Sa, <+++ Sa, be real numbers. a) If by < by $+ Sb, then b) If by = by = +++ By, then (7) Convex functions A function f defined on an interval J of real numbers is said to be convex if for all x, y €Land any a, > 0 witha+f =1, flax + By) < af (x)+ Bf(y). qa If (1) is strict for all x # y and a, 6 > 0, then f is said to be strictly convex. If (1) is reversed, then f is said to be (strictly) concave. The inequality (1) is equivalent to f(a) F(x) (qq — 2) = 3) eg = 83)(22 - 1) where x, x2, x3 are distinct numbers on I. If f is differentiable on I, then f is (strictly) convex if and only if the derivative fis (strictly) increasing. If f is continuous on [a, b] and f” exists on (a, b), then f is convex on [a, 5] if and only if f” > 0. If f” > 0, then f is strictly convex, If f” exists on (a, b), then f is convex on (a, b) if and only if f” > 0. If f” > 0, then f is strictly convex. 327 Glossay (8) Cyclic sum If f is a function of n variables, define the cyclic sum assume Dla, 235-225 Oa) = Fas Mayo On EF Cay ay vy Ib AL ge Mts oor Mya In our book, the symbols x and }? are identical oe (9) Hélder’s Inequality Let xij (i= 1, 2,...,7, 7 =1, 2, ..., 2) be nonnegative real numbers. Then f(S«) =4 (C1). isl \ j=l j=l \i=1 More general, if p), ps ..s Px ate positive real numbers with py +p2++'-+Pn = 1, then Bi Il 0 =X (fhe). a1 \ = i= (10) Jensen’s Inequality Let w,, Wa, ..., Wy be positive real numbers. If f is convex on an interval I, then for any a}, dz, ..., dy € I, the inequality holds wf an) + wah (aa) toot wnf (Oa) , - (wit + waty ten two wy twote tw, = wy tWy te tw, If f is strictly convex, then equality occurs if and only if, = a) =---= ay. (11) Karamata’s Majorization Inequality We say that a vector A = (a), a, ...,@,) with a) > ay) > ++: > a, majorizes a verctor B = (bj, ba, ..., Bn) with by > by > «+» > b,, and write itas A > B., if a, 2b, ay +a2 = by + by, G+) +-+ + Og-y 2 dy + bg tet bn, ay tag +++ a, = by + byte + by. If f is a convex function on an interval I, and a vector A = (ay, a9, ...5@,) with a; €I majorizes a vector B = (b,, bo, ..., b,) with b; €I, then Flay) +f (ag) +--+ + fn) = f(b) + f (ba) +--+ Fn) 328 Vasile Cirtoaje - Vo Quoc Ba Can - Tran Quoc Anh (12) Lagrange’s Identity For any two sets fay, a2, ..., an} and {b,, by, ..., by} of real number, we have * EGG) Zs i=l Isi 1 and any positive real numbers @;, a2, ..., a, and by, by, ..., Ba, the inequality holds [Bem] = (Sx) (Es) (15) Newton’s Inequality If ay, dg, «+45 dq are nonnegative real numbers, then S$2S-iSe1 Glossay 329 (16) Power Mean Inequality For positive real numbers ay, @, ..., a, the power mean of order r is defined by Fe xgpeetxr\ it+% n M.={{-—, or r#0 YR for r=0 if not all q;’s are equal, then M, is strictly increasing for r © R. For instant, M, > M, = Mo implies (17) Rearrangement Inequality Let (a), dy, ..-5 G,) and (by, bg, ..., by) be two increasing sequences of ral numbers. Suppose that (iy, i, ..., i,) is an arbitrary permutation of (1, 2, n), then a,b, +agby +++ +Aqby 2 a,b), + agbi, + If the sequence (ay, dg, ..., dq) is increasing but the sequence (b;, by, ..., Bn) is decreasing then the sign of the above inequality is reversed. (18) Schur’s Inequality For any nonnegative real numbers a, b, c and any positive number r, the in- equality holds a"(a— bac) + b"(b —c)(b —a) +e" (c — alc — b) = 0, with equality if and only if a = b =c, or a =O and b=c, or b=Oande =a, or c=Oanda=b. For r = 1, we get the third degree Schur’s Inequality a +b? +c2 + 3abe > ab(a +b) + be(b +c) +ca(c +a), (a+b+c}+9abe > 4(a+b+c)(ab + be +ca), (b-c}*(b +c —a)+(c—a)*(c+a—b)+(a— b)*(at+b—c) 20, Ty 2 Aab-+ be +ca), c 4abe — + >2. a+b” (@tbyb+olera) 9abe 2. py. od +5 a +¢ tot 330 Vasile Cirtoaje - Vo Quoc Ba Can - Tran Quoc Anh For r = 2, we get the fourth degree Schur’s Inequality a*+b4+c4+abe(at+b +c) > ab(a? + b?) + be(b? +c2) + cafe? +a”). (19) Weighted AM-GM Inequality Let wj, W2, .-., W, be positive real numbers with w; +w2+---+w, =1. Ifa, dy, ..., dy are nonnegative real numbers, then Wydy + Woy ++ + Way 2 ay tay? +a a with equality if and only if a; = ay ay. (20) Weighted Power Mean Inequality Let P1, Pa, --- Pn be positive real numbers with py +py+---+p, = 1. For positive real numbers dy, a, ..., dq, the weighted power mean of order r is defined by im {ret neah tn tree? for r# ° ablay? ah" for r=0 If not all q;’s are equal, then M, is strictly increasing for r ER. Bibliography [ACDL] [BB] [CA] [can] [can] [cP] [cir] (cir] (cir) [cir] [Cir] [cir] (cir) Andreescu, T. Cirtoaje, V, Dospinescu, G., Lascu, M., Old and New Inequalities, GIL Publishing House, 2004. Beckenbach, E., Bellman, R., Inequalities, Springer Verlag, 1961 Can, VQ. B., Anh, T. Q., Bat Dang Thuc Va Nhung Loi Giai Hay, Hanoi Publishing House, 2009 (Vietnamese). Can, VQ. B., On a Class of Three-Variable Inequalities, Mathematical Reflections, Issue 2, 2007. Can, VQ. B., Su Dung Dang Thuc De Chung Minh Bat Dang Thuc, Maths and Youth magazine, 2008 (Vietnamese), Can, V. Q. B., Pohoat’, C., Old and New Inequalities, Volume 2, GIL Publishing House, 2008. Cirtoaje, V, On Popoviciu’s Inequality for Convex Functions, Gazeta Matematica A, Issue 4, 2002, 247 - 253. Cirtoaje, V, To Generalizations of Popoviciu’s Inequality, Crux Math- ematicorum, Issue 5, 2005, 313 - 318. Cirtoaje, V, The Equal Variable Method, Journal of Inequalities In Pure and Applied Mathematics, Volume 7, Issue 3, 2006. Cirtoaje, V, Equal Variable Theorem, Gazeta Matematica A, Issue 2, 2006. Cirtoaje, V, Arithmetic Compensation Method, Mathematical Reflec- tions, Issue 2, 2006, 1 - 5. : Old and New Methods, GIL Publish- Cirtoaje, V, Algebraic Inequaliti ing House, 2006. Cirtoaje, V, Four Applications of RGF and LCF Theorems, Mathematical Reflections, Issue 1, 2007. 331 332 [cir] [cir] [cir] [Cir] [cir] [Hun] (Hun) [LP] [Las] ([MBMR] [MPF] [PBL] [Ste] Vasile Cirtoaje - Vo Quoc Ba Can - Tran Quoc Anh Cirtoaje, V, On Jensen Type Inequalities with Ordered Variables, Jour- nal of Inequalities In Pure and Applied Mathematics, Volume 9, Issue 1, 2008. Cirtoaje, V, The Proof of Three Open Inequalities, Crux Mathematico- rum, Volume 34, Issue 4, 2008, 231 - 238. Cirtoaje, Vi, On Some Inequalities with Power-Exponential Functions, Journal of Inequalities In Pure and Applied Mathematics, Volume 10, Issue 1, 2009. Cirtoaje, V, On the Cyclic Homogeneous Polynomial Inequalities of De- gree Four, Journal of Inequalities in Pure and Applied Mathematics, Volume 10, Issue 3, 2009. Engel, A., Problem-Solving Strategies, Problem books in Mathematics, Springer, 1998. Hung, P K., Secrets in Inequalities, Volume 1: basic inequalities, GIL Publishing House, 2007. Hung, P K,, Secrets in Inequalities, Volume 2: advanced inequalities, GIL Publishing House, 2008. Littlewood, G. H., Polya, J. E., Inequalities, Cambridge University Press, 1967. Lascu, M., Inegalitdgi, GIL Publishing House, 1994. Mitrinovié, D. S., Barnes, E. S., Marsh, D. C. B., Radok, J. R. M., Ele- mentary Inequalities, Wolters-Noordhoff Publisher, Groningen, 1964. al and New In- Mitrinovié, D. S., Pecari@, J. E., Fink, A. M., Cl equalities in Analysis, Kluwer, 1993. Panaitopol, L., Bandila, V,, Lascu, M., Inegalitdgi, GIL Publishing House, 1995. Steele, J. M., The Cauchy-Schwarz Master Class, Cambridge Univer- sity Press, 2004. Som “Let solutions say the method!” is the way this book is writ Readers don’t find here the entire theory, strong theorems as as detailed explanation of the methods. But you can find here of beautiful problems with beautiful solutions. Most of tt solutions are simple and elementary, the authors try to avoid much as possible of using advanced methods of proving inequalities. The main weapons here are skilful technics of handlin« algebraic expressions and virtuous applications of cla inequalities. It makes the book more romantic rather academic. And evena student of 8th, 9th grade can read most of tl content of this book. Tran Nam D Ho Chi Minh city University of Scien COLiBRIs 1.S.B.N. 978-606-500-014-8

You might also like